Vous êtes sur la page 1sur 141

Usmle step 2 mcqs. With explanations.contains 250 mcqs in 6 part.

Part 1/6-50 mcqs 1> A normal, healthy, 25-year-old man lives at the beach. His twin brother has been living in a mountain cabin for the past 2 years. Which of the following indices would be expected to be higher in the man living at sea level? A. Diameter of pulmonary vessels B. Erythropoietin production C. Mitochondrial density in a muscle biopsy D. Renal bicarbonate (HCO3-) excretion E. Respiratory rate Answer ------------------------------------------------------------------------------2> A woman presents to a dermatologist because she has lost almost all the hair on her body, including scalp hair, eye brows, eye lashes, arm pit and groin hair, and the fine hairs on her body and extremities. She most likely has a variant of which of the following? A. Alopecia areata B. Androgenic alopecia C. Chronic cutaneous lupus erythematosus D. Lichen planopilaris E. Trichotillomania Answer ------------------------------------------------------------------------------3> The greater omentum is derived from which of the following embryonic structures? A. Dorsal mesoduodenum B. Dorsal mesogastrium C. Pericardioperitoneal canal D. Pleuropericardial membranes E. Ventral mesentery Answer -------------------------------------------------------------------------------4>A 34-year-old woman is prescribed an antidepressant, which she has taken for the past 3 months. She is on no other medications and is in generally

good health. After attending a party, at which she consumed wine and cheese, she is rushed to the emergency room with tachycardia, headache, and a blood pressure of 200/100. Which antidepressant is she most likely taking? A. Amitriptyline B. Fluoxetine C. Phenelzine D. Sertraline E. Trazodone Answer -------------------------------------------------------------------------------5>Which of the following hormones is secreted by anterior pituitary cells that stain with acidic dyes? A. ACTH B. FSH C. LH D. Prolactin E. TSH 6>A 23-year-old male graduate student gets into a serious argument with one of his college professors, making a physical threat to the professor, and necessitating a call to campus security. The argument was precipitated by an incident between the professor and the student's girlfriend; when the professor corrected the student's girlfriend in class, the student felt the professor was verbally abusive. Which of the following is the most likely diagnosis ? A. Dependent personality disorder B. Histrionic personality disorder C. Narcissistic personality disorder D. Paranoid personality disorder E. Passive aggressive personality disorder Answer -------------------------------------------------------------------------------7>Which of the following would shift the oxygen-hemoglobin dissociation curve to the right? A. Carbon monoxide poisoning B. Decreased PCO2 C. Decreased pH D. Decreased temperature E. Decreased 2,3-DPG Answer -------------------------------------------------------------------------------8>The wife of a 48year-old male patient brings him to the emergency room and says that his memory has progressively gotten worse over the last several years. She also says his personality has been changing. The physician notes

abnormal writhing movements of the man's limbs and hyperreactive reflexes . MRI reveals a loss of volume in the neostriatum and cortex. This disease is inherited via an A. autosomal dominant trait B. autosomal recessive trait C. x-linked dominant trait D. xlinked recessive trait Answer -------------------------------------------------------------------------------9>A man presents to a dermatologist because of a severe mucocutaneous rash that involves most of his body, including his palms and soles. Questioning reveals that he is a merchant marine who several months previously had an encounter with a prostitute in Southeast Asia. Which of the following is the most likely causative agent of this rash? A. Herpes simplex I B. Herpes simplex II C. HIV D. Neisseria gonorrhoeae E. Treponema pallidum Answer -------------------------------------------------------------------------------10>A patient with familial hypercholesterolemia undergoes a detailed serum lipid and lipoprotein analysis. Studies demonstrate elevated cholesterol in the form of increased LDL without elevation of other lipids. This patient's hyperlipidemia is best classified as which of the following types? A. Type 1 B. Type 2a C. Type 2b D. Type 3 E. Type 5 Answer -------------------------------------------------------------------------------11>A mutation affecting the development of the diencephalon could interfere with the secretion of which of the following hormones? A. Adrenocorticotrophic hormone (ACTH) B. Epinephrine C. Oxytocin D. Prolactin E. Thyroid stimulating hormone (TSH) Answer -------------------------------------------------------------------------------12>Which of the following organisms is the most common cause of community-acquired pneumonia? A. Chlamydia pneumoniae

B. Haemophilus influenzae C. Mycoplasma pneumoniae D. Staphylococcus aureus E. Streptococcus pneumoniae Answer -------------------------------------------------------------------------------13>A 54-year-old male with acute lymphocytic leukemia develops a blast crisis. He is treated with intensive systemic chemotherapy. Following treatment, the patient will be at increased risk for the development of A. bile pigment gallstones B. cholesterol gallstones C. cystine kidney stones D. struvite kidney stones E. uric acid kidney stones Answer -------------------------------------------------------------------------------14>A 65-year-old man with mild heart failure is treated with a loop diuretic . A few days later the man complains of muscle weakness. Laboratory results are shown below. Arterial PCO2: 48 mm Hg Arterial pH: 7.49 Plasma HCO3-: 35 mEq/L Which of the following is most likely decreased in this man? A. Plasma aldosterone B. Plasma potassium C. Potassium excretion D. Renin secretion E. Sodium excretion Answer -------------------------------------------------------------------------------15>A 27-year-old white male presents with a 3-week history of several swollen and painful toes and knees. He has a past history of conjunctivitis. He also describes some low back stiffness that is more severe in the morning. Which of the following is the most likely diagnosis? A. Gout B. Lyme disease C. Reiter's syndrome D. Rheumatoid arthritis E. Septic arthritis Answer -------------------------------------------------------------------------------16>A 37-year-old woman who was diagnosed with AIDS 3 years earlier is unable to work, is physically debilitated, and requests her physician to provide her with medications with which to take her own life. The most common emotional disorder that results in such requests by patients is

A. bipolar I disorder, manic type B. borderline personality disorder C. factitious disorder D. major depressive disorder E. schizophrenic disorder Answer -------------------------------------------------------------------------------17> A 25-year-old man presents with a chief complaint of persistent, high-pitched ringing noises in his ears. Questioning reveals that he has also been losing his balance lately. CT of the head demonstrates bilateral tumors involving the vestibulocochlear nerve. Which of the following chromosomes contains the tumor suppressor gene most likely to be involved in this case? A. 5q B. 13q C. 17q D. 18q E. 22q Answer -------------------------------------------------------------------------------18> An autopsy is performed on a man who suddenly began vomiting voluminous quantities of blood and exsanguinated. Which of the following organisms is most likely implicated in the pathogenesis of this disease? A. Cryptosporidium parvum B. Entamoeba histolytica C. Escherichia coli D. Helicobacter pylori E. Mycobacterium tuberculosis Answer -------------------------------------------------------------------------------19>During embryological development, hematopoiesis occurs in different organs at different times. Which of the following are the correct organs, in the correct sequence, at which hematopoiesis occurs embryologically? A. Amnion, yolk sac, placenta, bone marrow B. Placenta, liver and spleen, yolk sac, bone marrow C. Placenta, spleen and lymphatic organs, bone marrow D. Yolk sac, bone marrow, liver and spleen E. Yolk sac, liver, spleen and lymphatic organs, bone marrow Answer -------------------------------------------------------------------------------20>A 27-year-old male is brought into the emergency room by the police, who found him walking aimlessly, shouting the names of former Presidents. Urine toxicology is negative, and the man appears to be oriented with

respect to person, place, and time. He has had five similar admissions over the past year. Attempts to interview the patient are fruitless, as he seems easily derailed from his train of thought. A phone call to a friend listed in the chart provides the additional information that the man is homeless, and unable to care for himself. This patient is exhibiting the signs and symptoms of A. schizoaffective disorder B. schizoid personality disorder C. schizophrenia D. schizophreniform disorder E. schizotypal personality disorder Answer -------------------------------------------------------------------------------21>Molecular genetic studies are performed on a family with known familial hypercholesterolemia. In this particular family, the defect in the LDL receptor gene involves a messenger mutation near the 11th exon, in the region of homology with epidermal growth factor receptor precursor. A defect at this site would be most likely to produce which of the following effects? A. Decreased transcription of LDL receptor gene B. Poor internalization of LDL bound to LDL receptor C. Poor retention of the LDL receptor in the membrane D. Reduced binding of LDL E. Trapping of the LDL receptor in the endoplasmic reticulum Answer -------------------------------------------------------------------------------22>A 27-year-old woman is giving birth. During the birth, the placental membranes tear and amniotic fluid is expressed into a lacerated cervical vein. Which of the following is the woman most likely to experience immediately following this event? A. Hemiplegia B. Placental abruption C. Renal failure D. Respiratory distress E. Splinter hemorrhages Answer -------------------------------------------------------------------------------23>A 30-year-old pregnant woman complains to her physician of feeling very tired during her pregnancy. A complete blood count with differential reveals a hematocrit of 30%, with hypersegmented neutrophils and large, hypochromic red cells. Deficiency of which of the following would be most likely to produce these findings? A. Ascorbic acid B. Calcium C. Copper

D. Folate E. Iron Answer -------------------------------------------------------------------------------24>An otherwise healthy 3-year-old child is brought to the pediatrician with umbilicated, flesh-colored papules on his trunk. This condition is related to infection with which of the following viruses? A. Cytomegalovirus B. Herpesvirus 6 C. Parvovirus D. Poxvirus E. Variola Answer -------------------------------------------------------------------------------25>Which of the following renal structures is most medially located? A. Major calyx B. Minor calyx C. Renal cortex D. Renal pelvis E. Renal pyramid Answer -------------------------------------------------------------------------------Answers Now you can see what's hidden-------------------------------------------------------------------------------1>The correct answer is A. A number of physiologic changes occur in a person living at high altitude. The diminished barometric pressure at high altitude causes alveolar hypoxia and arterial hypoxia. Pulmonary vasoconstriction occurs in response to alveolar hypoxia; therefore, the diameter of the pulmonary vessels would be greater in the brother living at sea level. All the other choices describe physiologic processes that would be enhanced by living at high altitude. Increased erythropoietin production (choice B), caused by arterial hypoxia, leads to increases in hematocrit in people living at high altitude. Mitochondrial density increases (choice C) in people chronically exposed to the hypoxemia caused by living at high altitude. At high altitudes, the ventilation rate increases, causing a respiratory alkalosis. The kidney then compensates by increasing the excretion of HCO3(choice D). Increasing the rate of respiration (choice E) is a very useful adaptation to the hypoxic conditions of high altitude. The primary stimulus is the hypoxic stimulation of peripheral chemoreceptors. -------------------------------------------------------------------------------2>The correct answer is A. Alopecia areata is caused by an autoimmune attack on hair follicles. It has a wide range of clinical severity, with most cases involving a localized patch of hair (which regrows within 1 year in half of the

patients). The hair that does regrow may be gray or depigmented. More severe cases can involve the entire scalp (alopecia totalis) or, as in this patient, the entire body surface (alopecia universalis). These more severe cases are less likely to resolve adequately. Treatment of alopecia areata is often unsuccessful, but topical steroids are typically tried. Androgenic alopecia (choice B) is common male pattern baldness. Chronic cutaneous lupus erythematosus (choice C) can produce localized baldness. Lichen planopilaris (choice D) can produce localized baldness. Trichotillomania (choice E), also called traumatic alopecia, is alopecia due to trauma, such as hair pulling or tight braids. --------------------------------------------------------------------------------3>The correct answer is B. Both the omental bursa and the greater omentum are derived from the dorsal mesogastrium, which is the mesentery of the stomach region. The dorsal mesoduodenum (choice A) is the mesentery of the developing duodenum, which later disappears so that the duodenum and pancreas come to lie retroperitoneally. The pericardioperitoneal canal (choice C) embryologically connects the thoracic and peritoneal canals. The pleuropericardial membranes (choice D) become the pericardium and contribute to the diaphragm. The ventral mesentery (choice E) forms the falciform ligament, ligamentum teres, and lesser omentum. --------------------------------------------------------------------------------4>The correct answer is C. Wine and cheese (and many other fermented foods) contain tyramine, an indirect sympathomimetic that can trigger excess catecholamine release and lead to a hypertensive crisis when ingested by patients taking MAO inhibitors. The only such drug listed among the answer choices is phenelzine. Other MAO inhibitors with similar effects include tranylcypromine, isocarboxazid, and iproniazid. Whenever this particular drug class is mentioned in a question stem, consider the possibility of interactions with foods or other medications the patient may have taken. Amitriptyline (choice A) is a tricyclic antidepressant. Tricyclic antidepressant drugs (particularly amitriptyline) are known for their anticholinergic side effects. They also produce postural hypotension (because they block alphaadrenergic receptors) and are sedative. Fluoxetine and sertraline (choices B and D) are antidepressants that are selective serotonin reuptake inhibitors (SSRIs). Fluoxetine is also useful in treatment of obsessive-compulsive disorders. Trazodone (choice E) is an atypical antidepressant with substantial sedative side effects. Its most serious side effect is priapism, a medical emergency. --------------------------------------------------------------------------------5>The correct answer is D. The cells of the anterior pituitary can be classified as chromophils (love dyes) or chromophobes (do not stain with dyes). The chromophils can be further divided into acidophils (stain with acidic dyes) and basophils (stain with basic dyes). The acidophils include the somatotropes, which secrete growth hormone, and the mammotropes, which secrete prolactin. The basophils include the corticotropes, which secrete ACTH (choice A), the gonadotropes, which secrete FSH and LH (choices B and C), and the

thyrotropes, which secrete TSH (choice E). --------------------------------------------------------------------------------6>The correct answer is D. Persons with this condition often perceive attacks and danger in relatively innocuous situations. They are quick to respond with anger, and, because personality disorders are ego-syntonic, individuals with personality disorders do not believe themselves to be in error. The individual with dependent personality disorder (choice A) does not confront others but wants others to take care of him. The individual with histrionic personality disorder (choice B) is flamboyant and seductive, not confrontational and angry. The individual with narcissistic personality disorder (choice C) is characterized by feelings of entitlement because they are so "special." And the individual with passive aggressive personality disorder (choice E) expresses anger indirectly (e.g., always being late) rather than confronting --------------------------------------------------------------------------------7>The correct answer is C. The loading of O2 is facilitated when the oxygen dissociation curve shifts to the left, and the unloading of O2 is facilitated when the oxygen dissociation curve shifts to the right. A good way to remember the conditions that promote dissociation of O2 is to think of exercising muscle, which has decreased pH (choice C) because of the accumulation of lactic acid, increased PCO2 (compare with choice B) because of the increased rate of aerobic metabolism, increased temperature (compare with choice D), and increased 2,3DPG (2,3-diphosphoglycerate; compare with choice E) because of increased glycolysis. Carbon monoxide poisoning (choice A) left-shifts the oxygen dissociation curve, which interferes with the unloading of O2. Carbon monoxide also strongly binds to available sites on hemoglobin. --------------------------------------------------------------------------------8>The correct answer is A. This patient has Huntington's disease, which has autosomal dominant inheritance. It is characterized by severe degeneration of the caudate nucleus along with degenerative changes in the putamen and cortex. In addition to chorea, these patients frequently suffer from athetoid (writhing) movements, progressive dementia, and behavioral disorders. --------------------------------------------------------------------------------9>The correct answer is E. The rash described is that of secondary syphilis, caused by Treponema pallidum. Involvement of palms and soles by a rash is unusual, and secondary syphilis should come to mind. Not all patients with secondary syphilis have a severe form of the rash, and consequentially some cases are missed. Primary syphilis takes the form of a painless, button-like mass called chancres. Tertiary syphilis, which is now rare, has a propensity for involving the aorta and central nervous system and can also cause "gummas" (granulomatous-like lesions) in many sites, notably including liver and bone. Herpes simplex I (choice A) usually causes perioral vesicular lesions. Herpes simplex II (choice B) usually causes genital vesicular lesions. HIV (choice C) does not itself cause a rash, although co-infection with other organisms can result in a rash. Neisseria gonorrhoeae (choice D) does not typically cause a rash. -------------------------------------------------------------------------------10>The correct answer is B. Hyperlipidemia has been subclassified based on the lipid and lipoprotein profiles. Type 2a, which this patient has, can be seen

in a hereditary form, known as familial hypercholesterolemia, and also in secondary, acquired forms related to nephritic syndrome and hyperthyroidism. The root problem appears to be a deficiency of LDL receptors, which leads to a specific elevation of cholesterol in the form of increased LDL. Heterozygotes for the hereditary form generally develop cardiovascular disease from 30 to 50 years of age. Homozygotes may have cardiovascular disease in childhood. Type 1 (choice A) is characterized by isolated elevation of chylomicrons. Type 2b (choice C) is characterized by elevations of both cholesterol and triglycerides in the form of LDL and VLDL. Type 3 (choice D) is characterized by elevations of triglycerides and cholesterol in the form of chylomicron remnants and IDL. Type 5 (choice E) is characterized by elevations of triglycerides and cholesterol in the form of VLDL and chylomicrons. --------------------------------------------------------------------------------11The correct answer is C. The neurohypophysis (posterior pituitary) is derived from an evagination of diencephalic neurectoderm. This structure is responsible for releasing oxytocin and vasopressin to the general circulation. Both hormones are synthesized in cell bodies contained within the hypothalamus. ACTH (choice A), prolactin (choice D), and TSH (choice E) are all synthesized and released by the anterior pituitary, or adenohypophysis, which is derived from an evagination of the ectoderm of Rathke's pouch, a diverticulum of the primitive mouth. Remnants of this pouch may give rise to a craniopharyngioma in later life. Epinephrine (choice B) is synthesized and released into the circulation by the adrenal medulla, a neural crest derivative. -------------------------------------------------------------------------------12>The correct answer is E. The most common bacteria implicated in community-acquired pneumonia is the pneumococcus, Streptococcus pneumoniae. Other organisms frequently implicated in patients less than age 60 without comorbidity include Mycoplasma pneumoniae, respiratory viruses, Chlamydia pneumoniae, and Haemophilus influenzae. When communityacquired pneumonia occurs in elderly patients or patients with comorbidity, aerobic gram-negative bacilli and Staphylococcus aureus are added to the list. The organisms listed in choices A, B, and C are important causes of community-acquired pneumonia, but are not the most frequent causes. Staphylococcus aureus (choice D) is an important cause of communityacquired pneumonia (particularly in the elderly and in patients with comorbidity), but is not the most frequent cause. -------------------------------------------------------------------------------13>The correct answer is E. Uric acid kidney stones in patients with leukemia are secondary to increased production of uric acid from purine breakdown during periods of active cell proliferation, especially following treatment. Vigorous hydration and diuresis are generally instituted after the diagnosis of acute leukemia is made. Uric acid kidney stones are also associated with inborn errors of purine metabolism, such as gout. Pigment gallstones (choice A) are associated with hemolytic disease. The incidence of this type of gallstone is not increased in treated leukemias. Cholesterol gallstones (choice B) are associated with diabetes mellitus, obesity, pregnancy, birth control pills, and celiac disease. Cystine kidney stones (choice C) are rare; they are found in cystinuria.

Struvite kidney stones (choice D) are associated with infection by ureasplitting organisms, such as Proteus. -------------------------------------------------------------------------------14>The correct answer is B. The data shown in the table indicate that the man has developed metabolic alkalosis (increased PCO2, pH, and HCO3-), which occurs commonly with overuse of diuretics (thiazides and loop diuretics). The overuse of a loop diuretic increases the excretion of sodium (choice E) and potassium (choice C) by the kidneys. The increase in potassium excretion leads to a decrease in plasma potassium levels (choice B). The decrease in plasma potassium stimulates aldosterone secretion, which raises plasma aldosterone levels (choice A). The sodium depletion stimulates renin secretion (choice D), which in turn raises angiotensin II levels in the plasma (which also stimulates aldosterone secretion -------------------------------------------------------------------------------15>The correct answer is C. This is a case of Reiter's syndrome. Patients typically present with the acute onset of arthritis (usually asymmetric and additive), with involvement of new joints occurring over a period of a few days to 2 weeks. Joints of the lower extremities are the most commonly involved, but wrists and fingers can also be affected. Dactylitis (sausage digit), a diffuse swelling of a solitary finger or toe, is a distinctive feature of Reiter's arthritis and psoriatic arthritis. Tendonitis and fasciitis are common. Spinal pain and low back pain are common. Conjunctivitis, urethritis, diarrhea, and skin lesions are also associated with Reiter's syndrome. Up to 75% of patients are HLAB27 positive. Microorganisms which can trigger Reiter's syndrome include Shigella spp., Salmonella spp., Yersinia spp., Campylobacter jejuni, and Chlamydia trachomatis. Most patients are younger males. Gout (choice A) usually presents as an explosive attack of acute, very painful, monarticular inflammatory arthritis. Hyperuricemia is the cardinal feature and prerequisite for gout. The first metatarsophalangeal joint is involved in over 50% of first attacks. Lyme disease (choice B), caused by Borrelia burgdorferi, presents with a red macule or papule at the site of the tick bite. This lesion, called erythema chronicum migrans, slowly expands to form a large annular lesion with a red border and central clearing. The lesion is warm, but usually not painful. The patient also has severe headache, stiff neck, chills, arthralgias, and profound malaise and fatigue. Untreated infection is associated with development of arthritis. The large joints (e.g., knees) are usually involved with the arthritis lasting for weeks to months. Rheumatoid arthritis (choice D) begins insidiously with fatigue, anorexia, generalized weakness, and vague musculoskeletal symptoms leading up to the appearance of synovitis. Pain in the affected joints, aggravated by movement, is the most common manifestation of established rheumatoid arthritis. Generalized stiffness is frequent and is usually greatest after periods of inactivity. Morning stiffness of greater than 1 hour in duration is very characteristic. Rheumatoid arthritis is more common in females. The metacarpophalangeal and proximal interphalangeal joints of the hands are characteristically involved. Septic arthritis (choice E) is caused by a variety of microorganisms, including Neisseria gonorrhoeae and Staphylococcus aureus. Hematogenous spread is the most common route in all age groups. 90% of patients present with

involvement of a single joint, usually the knee. The usual presentation is moderate-tosevere pain, effusion, muscle spasm, and decreased range of motion. Peripheral leukocytosis and a left shift are common. Disseminated gonococcal infections present as fever, chills, rash, and articular symptoms. Papules progressing to hemorrhagic pustules develop on the trunk and extensor surfaces of the distal extremities. Migratory arthritis and tenosynovitis of multiple joints is common. -------------------------------------------------------------------------------16>The correct answer is D. Many people who request physician-assisted suicide have one of two conditions present: either a poorly controlled painful condition or severe depression. If the painful condition is adequately treated or the depression is brought under good medical control, the request for physician assistance in terminating the situation is typically withdrawn. It is important to note that bringing these conditions under control requires the intervention of caregivers who are specifically trained in the management of these two conditions; primary care physicians usually are not adequately trained to address these difficult presentations. While patients who are diagnosed as bipolar disorder (choice A), borderline personality disorder (choice B), and schizophrenic disorder (choice E) often make suicide attempts (and frequently complete those attempts), they do not generally ask their physician for assistance in the suicide. Persons with factitious disorder (choice C) are seeking primary gain, often for dependency needs, and are seeking to enter the "sick role" not the "dead role." -------------------------------------------------------------------------------17>The correct answer is E. The patient has bilateral acoustic neuromas, probably due to neurofibromatosis type II (over 90% of patients with NF-2 develop bilateral acoustic neuromas). This condition is a associated with the NF-2, gene, located on 22q (note all the 2's). Patients often develop meningiomas, gliomas, and schwannomas of cranial and spinal nerves. 5q (choice A) contains the APC tumor suppressor gene, which is associated with familial and sporadic colorectal cancers. 13q (choice B) contains the Rb tumor suppressor gene, which is associated with retinoblastoma and osteosarcoma. 17q (choice C) contains both the NF-1 tumor suppressor gene, which is associated with neurofibromatosis type I, and the p53 tumor suppressor gene, associated with many human cancers. 18q (choice D) contains both the DCC gene, which is associated with colon and gastric carcinomas and the DPC gene, associated with pancreatic cancer. --------------------------------------------------------------------------------18> The correct answer is D. Perforation of a peptic ulcer is potentially fatal, because of either peritonitis with sepsis or sudden exsanguination (if the perforation damages one of the many arteries of the stomach). Peptic ulcer disease, gastritis, and possibly gastric carcinoma and gastric lymphoma have been strongly associated with Helicobacter pylori colonization of the mucus layer covering the gastric mucosa. Colonization is associated with destruction of the mucus layer, thereby destroying its protective function. Cryptosporidium parvum (choice A) causes diarrhea that is severe in immunocompromised patients.

Entamoeba histolytica (choice B) produces dysentery-like symptoms or can cause liver abscess. Escherichia coli (choice C) causes a variety of diarrheal diseases and can infect the bladder and soft tissues. Mycobacterium tuberculosis (choice E) causes tuberculosis, characterized by granuloma formation, especially in the lungs. -------------------------------------------------------------------------------19>The correct answer is E. By the third week of development, hematopoiesis begins in the blood islands of the yolk sac. Beginning at 1 month of age and continuing until 7 months of age, blood elements are also formed in the liver. Hematopoiesis occurs in the spleen and lymphatic organs between 2 and 4 months, and in the bone marrow after 4 months. -------------------------------------------------------------------------------15>The correct answer is C. The patient is suffering from schizophrenia. The key to the diagnosis of psychosis is that there has been a marked decline in the level of functioning (i.e., the man is homeless and cannot care for himself). Although hallucinations or delusions are not mentioned in the case history, the presence of disorganized speech, grossly disorganized behavior, and the duration of symptoms (longer than six months) suggest a diagnosis of schizophrenia. In schizoaffective disorder (choice A), alterations in mood are present during a substantial portion of the illness. Although schizoid personality disorder (choice B) produces detachment from social relationships and is characterized by restriction of emotional expression, it is not accompanied by a marked decline in occupational functioning. Schizophreniform disorder (choice D) is characterized by schizophrenic-like symptoms, but the duration of symptoms is, by definition, less than six months. Schizotypal personality disorder (choice E) is characterized by eccentricities of behavior, odd beliefs or magical thinking, and difficulties with social and interpersonal relationships. Unlike schizophrenia, schizotypal personality disorder is not characterized by a formal thought disorder. --------------------------------------------------------------------------------------------------------------------------------------------------------------21>The correct answer is E. Familial hypercholesterolemia, which is due to defective function of the LDL receptor, is an area of intense research. The molecular basis of LDL receptor abnormalities is becoming better understood, and more than 200 mutations in the gene for the LDL receptor have been identified. The gene has 5 general domains and 18 exons. Defects near exons 7 to 14 (including this case) are in the region of homology with epidermal growth factor receptor precursor. This region of the molecule is needed for dissociation of LDL from the receptor in the endosome. Receptors with a defect in this area (sometimes called class II mutations) also have trouble being initially transported to the Golgi complex (transport-deficiency alleles) and become trapped in endoplasmic reticulum. Decreased transcription of the LDL receptor gene (choice A) is considered a class I mutation and involves the signal sequence domain near exon 1. Poor internalization of LDL bound to LDL receptor (choice B) is considered a

class IV mutation. Such mutations are associated with the membranespanning/cytoplasmic domain, specifically near exon 18. Poor retention of the LDL receptor in the membrane (choice C) is considered a class IV mutation and is associated with the membrane-spanning/cytoplasmic domain, specifically near exons 2-6. Reduced binding of LDL (choice D) is considered a class III mutation and involves the LDL binding domain near exons 2-6. -------------------------------------------------------------------------------22>The correct answer is D. Respiratory distress immediately follows amniotic fluid embolism as the emboli consisting of squamous cells, lanugo, and mucus deposit in the pulmonary microcirculation, producing numerous tiny pulmonary infarcts. The dramatic respiratory distress may also reflect the action of prostaglandins and other bioactive compounds present in high concentrations in the amniotic fluid embolus. Hemiplegia (choice A) would reflect an ischemic injury to one hemisphere of the cerebrum or the brainstem. A venous embolus would not produce such an insult. Placental abruption (choice B) is partial, premature separation of the placental disc from the endometrium. Although abruption may occur in this setting, it is not a result of an amniotic fluid embolism. There are numerous causes of renal failure (choice C); the most likely ones in the peripartum interval include eclampsia, hypovolemic shock, and ascending infections. Amniotic fluid embolism would be expected to produce severe dyspnea well before shock and renal failure might arise. Splinter hemorrhages (choice E) are small hemorrhages seen on toes and fingers due to a shower of microemboli arising in the arterial circulation. Amniotic fluid emboli arise in the veins and deposit in the lungs. -------------------------------------------------------------------------------23>The correct answer is D. The patient has a megaloblastic anemia, which can be due to deficiency of folate or B12. Pregnancy increases the need for folate and other nutrients used by both baby and mother, and may "unmask" a borderline dietary deficiency. For this reason, most obstetricians recommend vitamin supplements for pregnant women. Ascorbic acid (choice A) is vitamin C, and its deficiency predisposes for capillary fragility and oral lesions. Calcium deficiency (choice B) predisposes for osteoporosis/osteopenia. Copper deficiency (choice C) is rare; when it occurs, it may cause a hypochromic anemia, neutropenia, osteoporosis, or hypotonia. Iron deficiency (choice E) causes a microcytic, hypochromic anemia, with reduced mental and physical performance. -------------------------------------------------------------------------------24>The correct answer is D. The lesions are characteristic of molluscum contagiosum, which is a typically benign and self-limited condition caused by a poxvirus. The disease can be transmitted either venereally or through nonvenereal contact. The other viruses listed do not cause similar skin lesions. Patients with advanced HIV infection may develop a severe, generalized, and persistent eruption, often involving the face and upper body. Cytomegalovirus (choice A) causes congenital infections and disseminated infections in immunosuppressed patients. Herpesvirus 6 (choice B)causes roseola (exanthem subitum).

Parvovirus (choice C) causes aplastic crises in patients with hemolytic anemia. Variola (choice E) is the smallpox virus. -------------------------------------------------------------------------------25>The correct answer is D. This is a relatively simple question that requires you to visualize the relationship among the key parts of the kidney and to identify the one that lies most medially. If you think about it for a second, since the kidneys ultimately drain into the ureter at their medial poles, you are looking for the structure that is closest to the ureter. The correct answer is the renal pelvis. The renal pelvis is the dilated upper portion of the ureter that receives the major calyces. In terms of the other answer choices, the order from most lateral to most medial is: renal cortex (choice C), renal pyramid (choice E), minor calyx (choice B), major calyx (choice A), and then the renal pelvis (choice D). -------------------------------------------------------------------------------26>A 48-year-old female is being treated for breast carcinoma. Over the past few days, she has been complaining of dysuria and frequency. Laboratory examination revealed the presence of microscopic hematuria. The next day the patient developed gross hematuria. Which of the following agents most likely caused the development of these signs and symptoms? A. Cyclophosphamide B. Mitomycin C. Paclitaxel D. Tamoxifen E. Vincristine Answer -------------------------------------------------------------------------------27>A child who has had abnormal development of the membranous bones has a broad skull with associated facial and dental anomalies. Which other bones are most likely to also be affected? A. Clavicles B. Femurs C. Metatarsals D. Phalanges E. Tibias Answer -------------------------------------------------------------------------------28>A 3-year-old child is referred to a major medical center because of an abdominal mass arising from his right adrenal gland. Biopsy of the lesion demonstrates sheets of small cells with hyperchromatic nuclei containing occasional pseudorosettes composed of circles of tumor cells with central young nerve fibers arising from the tumor cells. Which of the following oncogenes is associated with this patient's tumor? A. erb-B2 B. c-myc C. Lmyc D. N-myc E. ret

Answer -------------------------------------------------------------------------------29>A 3-yearold child develops headaches and is brought to the family doctor. Funduscopic examination reveals papilledema; one retina also shows a very vascular tumor. CT of the head demonstrates a cystic tumor of the cerebellum. This child has a high likelihood of later developing which of the following? A. Berry aneurysm of the basilar system B. Bilateral renal cell carcinoma C. Cancer of a peripheral nerve D. Choreiform movements related to decreased GABA and acetylcholine E. Serum cholesterol of greater than 700 mg/dL Answer -------------------------------------------------------------------------------30>Administration of an experimental drug that acts on PNS myelin is shown to increase the space constant of an axon in a peripheral nerve. Action potentials traveling down the axon would be predicted to be A. faster B. larger C. slower D. smaller E. unchanged Answer -------------------------------------------------------------------------------31>A 26-year-old man presents to his physician with a chronic cough. The man is a smoker, and states that he also gets frequent headaches and aches in his legs when he exercises. Chest x-ray demonstrates notching of his ribs. Which of the following undiagnosed congenital defects may be responsible for these findings? A. Coarctation of the aorta B. Eisenmenger's syndrome C. Tetralogy of Fallot D. Transposition of great vessels E. Ventricular septal defect Answer -------------------------------------------------------------------------------32>A 36-year-old Asian male complains of difficulty swallowing. Esophagoscopy reveals a polypoid mass that is subsequently biopsied. In addition to tumor cells, the esophageal biopsy show normal smooth muscle and striated muscle in the same section. Which portion of the esophagus was the source of this biopsy? A. Lower esophageal sphincter B. Lower third of the esophagus C. Middle third of the esophagus D. Upper esophageal sphincter

E. Upper third of the esophagus Answer -------------------------------------------------------------------------------33> A 25-year-old woman with sickle cell anemia complains of steady pain in her right upper quadrant with radiation to the right shoulder, especially after large or fatty meals. Her physician diagnoses gallstones. Of which of the following compounds are these stones most likely composed? A. Calcium bilirubinate B. Calcium oxalate C. Cholesterol D. Cholesterol and calcium bilirubinate E. Cystine Answer -------------------------------------------------------------------------------34>A 54-year-old alcoholic presents with complaints of tremors and muscle twitching. Physical examination reveals the presence of Trousseau's sign. Laboratory data show that serum magnesium is < 1 mEq/L (normal, 1.4 - 2.2 mEq/L). Which of the following findings would be most consistent with this information? A. Decreased serum calcium B. Decreased serum phosphate C. Increased bone density D. Increased plasma parathyroid hormone concentration E. Increased urinary cAMP concentration Answer -------------------------------------------------------------------------------35>A young boy presents with failure to thrive. Biochemical analysis of a duodenal aspirate after a meal reveals a deficiency of enteropeptidase (enterokinase). The levels of which of the following digestive enzymes would be affected? A. Amylase B. Colipase C. Lactase D. Pepsin E. Trypsin Answer -------------------------------------------------------------------------------36>A Guatemalan child with a history of meconium ileus is brought to a clinic because of a chronic cough. The mother notes a history of respiratory tract infections and bulky, foul-smelling stools. After assessment of the respiratory tract illness, the physician should also look for signs of A. cystinuria B. hypoglycemia C. iron deficiency anemia

D. sphingomyelin accumulation E. vitamin A deficiency Answer -------------------------------------------------------------------------------37>An infant is born with an abnormally developed falciform ligament. The hepatogastric and hepatoduodenal ligaments are also malformed . These developmental anomalies are most likely due to abnormal development of the A. dorsal mesoduodenum B. dorsal mesogastrium C. pericardioperitoneal canal D. pleuropericardial membranes E. ventral mesentery Answer -------------------------------------------------------------------------------38>A 26-year-old man is admitted through the emergency department to the hospital for a heroin overdose. His heart rate is 45 beats/min, and his blood pressure is 75/40 mm Hg. Which of the following best depicts the results from an arterial blood sample ? pH PaCO2 (mm Hg) HCO3- (mEq/L) A. 7.22 66 26 B. 7.34 29 15 C. 7.40 40 24 D. 7.47 20 14 E. 7.49 48 35 Answer -------------------------------------------------------------------------------39>A 15-yearold boy presents to his physician with several weeks of slowly worsening pruritus of both of his feet. He is otherwise well and taking no medications. On examination, he has bilateral, erythematous, dry scaling lesions that are most obvious in the interdigital web spaces and on the soles. There is no bleeding or exudate. What would most likely be found in a potassium hydroxide (KOH) mount of a scrapin g of the affected skin? A. Branching hyphae with rosettes of conidia B. Branching, septate hyphae C. Budding yeasts D. Hyphae, arthroconidia, and blastoconidia E. Pigmented, septate hyphal fragments F. Short, curved hyphae and round yeasts Answer -------------------------------------------------------------------------------40>A 48-year-old white female secretary presents with progressive difficulty typing over the past month. She also notes that her hands begin to feel numb and weak after typing for long periods of time. Upon testing, which of the following deficits would be predicted?

A. Difficulty in abducting the fifth finger B. Difficulty in adducting the thumb C. Difficulty in flexing digits two and three at the metacarpophalangeal joints D. Loss of sensation over the lateral half of the dorsum of the hand E. Loss of sensation over the lateral half of the palm F. Loss of sensation over the medial half of the dorsum of the hand G. Loss of sensation over the medial half of the palm Answer -------------------------------------------------------------------------------41>A 15-year-old girl is brought into the emergency room with severe abdominal pain and a fever. Laboratory examination is remarkable for an elevated white blood cell count and a pregnancy test is positive. Upon questioning, it is determined that she attempted to terminate her pregnancy by inserting a sharp object into her vagina. The physician determines that the wall of the posterior fornix of the vagina has been penetrated. Into what region did the sharp object penetrate? A. Deep perineal pouch B. Ischioanal space C. Rectouterine space D. Rectovesical space E. Vesicouterine space Answer -------------------------------------------------------------------------------42>Which of the following is found in the respiratory zone of the lung? A. Goblet cells B. Main bronchi C. Mucous cells D. Terminal bronchioles E. Type I epithelial cells Answer -------------------------------------------------------------------------------43>A patient presents with a bulge in the groin, which his physician diagnoses as an inguinal hernia. To determine whether it is an indirect or direct inguinal hernia, the physician applies pressure over the deep inguinal ring while palpating over the superficial inguinal ring. The patient is then asked to cough. The physician notes that the hernia does not push out of the abdomen and concludes that it is an indirect inguinal hernia. Where did the physician apply pressure? A. Immediately lateral to the lacunar ligament B. Immediately lateral to the pubic tubercle C. Immediately medial to the femoral vein D. One half inch above the midpoint of the inguinal ligament E. One half inch below the midpoint of the inguinal ligament Answer --------------------------------------------------------------------------------

44>A 68-year-old woman presents for a routine check-up. Her only complaint is that she occasionally experiences a little swelling in her ankles. Her serum potassium is 3.5 mEq/L, so the physician wants to avoid unnecessary potassium losses. Which of the following diuretics would be most appropriate for this patient? A. Furosemide B. Hydrochlorothiazide C. Indapamide D. Metolazone E. Spironolactone Answer -------------------------------------------------------------------------------45>A 15-year-old boy is evaluated by a clinician for failure to develop normal male secondary sexual characteristics. Physical examination reveals small testes, a small penis, and gynecomastia. The boy has had some difficulties in school, and the parents say that the school psychometrist said he had an IQ of 90. This patient's condition is most likely to be related to which of the following? A. Deletion B. Nondisjunction of an autosomal chromosome C. Nondisjunction of a sex chromosome D. Non-Robertsonian translocation E. Robertsonian translocation Answer -------------------------------------------------------------------------------46>A 5-year-old boy falls off his bike and fractures his humerus. He is taken to the emergency room, and the bone is set by one of the emergency room physicians. Which of the following is responsible for producing the majority of the new bone that will reunite the two fragments? A. Cancellous bone B. Cartilage C. Compact bone D. Marrow E. Periosteum Answer -------------------------------------------------------------------------------47>A 25-year-old man presents with a 4-cm mass above the clavicle. Chest Xray demonstrates marked mediastinal widening. Excisional biopsy of the supraclavicular mass demonstrates areas with the appearance shown in the photomicrograph above. The large cell (indicated with the arrow) is most likely which of the following? A. Langhans cell B. LE cell C. Mott cell D. Reed-Sternberg cell

E. Touton cell Answer -------------------------------------------------------------------------------48>A 50 -year-old electrician reports pain in his left upper extremity and tingling and numbness in his 4th and 5th digits of his left hand. Ther e is mild swelling of the left hand. The man reports most of his pain and numbness occurs when he is doing electrical work with his arms overhead. X-ray reveals the presence of a cervical rib. Which of the following structures is most likely being compressed? A. Axillary artery B. Brachial artery C. Brachiocephalic artery D. Subclavian artery E. Subscapular artery Answer -------------------------------------------------------------------------------49>The left adrenal vein drains directly into which of the following veins? A. Hemiazygos vein B. Inferior vena cava C. Left renal vein D. Splenic vein E. Superior mesenteric vein Answer -------------------------------------------------------------------------------50>To which of the following diseases is pyruvate kinase deficiency most similar clinically? A. a-thalassemia B. b-thalassemia C. Glucose-6-phosphate dehydrogenase deficiency D. Hereditary spherocytosis E. Iron deficiency anemia Answer -------------------------------------------------------------------------------------------------------------------------------------------------------------Answers Now you can see what's hidden26> The correct answer is A. Cyclophosphamide is metabolized to acrolein, which is excreted in the urine. If the patient's urine is concentrated, the toxic metabolite may cause severe bladder damage. Early symptoms of bladder toxicity include dysuria and frequency. This can be distinguished from a urinary tract infection, since there is no bacteriuria with cyclophosphamideinduced bladder toxicity. However, microscopic hematuria is often present on urinalysis. In severe hemorrhagic cystitis, large segments of the bladder

mucosa may be shed which can lead to prolonged, gross hematuria. The incidence of cyclophosphamide-induced hemorrhagic cystitis can be decreased by ensuring that the patient maintains a high fluid intake. Cyclophosphamide is an alkylating agent used in the treatment of breast carcinoma, malignant lymphoma, multiple myeloma, and adenocarcinoma of the ovary, as well as various other forms of cancer. The major toxic reactions commonly seen with this agent include mucositis, nausea, hepatotoxicity, sterile hemorrhagic and non-hemorrhagic cystitis, leukopenia, neutropenia, and interstitial pulmonary fibrosis. Mitomycin (choice B) is an antibiotic antineoplastic agent used in the treatment of breast carcinoma, adenocarcinoma of the pancreas and stomach, as well as various other forms of cancer. The major toxic reactions commonly seen with this agent include bone marrow depression, nausea, hepatotoxicity, acute bronchospasm, thrombocytopenia, and interstitial pneumonitis. Paclitaxel (choice C) is an antineoplastic agent primarily used in the treatment of ovarian and breast cancer. The major toxic reactions commonly seen with this agent include bone marrow depression, nausea, hepatotoxicity, bronchospasm, thrombocytopenia, and neutropenia. Tamoxifen (choice D) is an antineoplastic hormone primarily used in the palliative treatment of estrogenreceptor positive breast cancer patients. The major toxic reactions commonly seen with this agent include depression, dizziness, thrombosis, mild leukopenia or thrombocytopenia. Vincristine (choice E) is a mitotic inhibitor antineoplastic agent used in the treatment of breast cancer, Hodgkin's disease, non-Hodgkin's lymphoma, advanced testicular cancer and various other types of cancer. The major toxic reactions commonly seen with this agent include mental depression, hemorrhagic enterocolitis, bone marrow depression, nausea, thrombocytopenia, and leukopenia -------------------------------------------------------------------------------27>The correct answer is A. In a syndrome called cleidocranial dysostosis, absence of part of the clavicles accompanies a broad skull, and facial and dental anomalies. Note that you could also have answered this question by noting that of the bones listed, only the clavicles form by intramembranous ossification. The femurs (choice B), metatarsals (choice C), phalanges (choice D), and tibias (choice E) are cartilaginous (formed by endochondral ossification) rather than membranous bones. -------------------------------------------------------------------------------28>The correct answer is D. The tumor is a neuroblastoma, which is one of the principal forms of cancer in children. Neuroblastoma typically occurs before age 5, with many presenting before age 2. Neuroblastoma can arise from neural crest cells throughout the body, but the adrenal medulla is the most common site. Homer-Wright pseudorosettes are circles of tumor cells with central young nerve fibers arising from the tumor cells. The oncogene associated with neuroblastoma is N-myc. erb-B2 (choice A) is associated with breast, ovarian, and gastric carcinomas. c-myc (choice B) is associated with Burkitt's lymphoma. L-myc (choice C) is associated with small cell carcinoma of the lung. ret (choice E) is associated with multiple endocrine neoplasia, types II and III. --------------------------------------------------------------------------------

29>The correct answer is B. The disease is von Hippel-Landau disease, which is associated with a deletion involving the VHL gene on chromosome 3 (3p). Affected individuals develop vascular tumors (hemangioblastomas) of the retina, cerebellum, and/or medulla. Roughly half of the affected individuals later develop multiple, bilateral renal cell carcinomas. Berry aneurysms (choice A) are unrelated to hemangioblastomas, but are instead associated with adult polycystic disease. Peripheral nerve cancers (choice C) are a feature of von Recklinghausen's disease (neurofibromatosis type I). Choreiform movements, related to decreased GABA and acetylcholine (choice D), are a feature of Huntington's disease. Extremely high serum cholesterol (choice E) suggests the homozygous form of familial hypercholesterolemia. -------------------------------------------------------------------------------30>The correct answer is A. The space constant of an axon reflects the amount of passive or electrotonic spread of current within an axon. The larger the space constant, the further the current can spread, allowing action potentials to propagate faster. This is why myelin increases the conduction velocity of action potentials down an axon. Conversely, demyelination decreases the space constant and slows action potential conduction. -------------------------------------------------------------------------------31>The correct answer is A. Coarctation of the aorta occurs in two patterns. In the infantile type, the stenosis is proximal to the insertion of the ductus arteriosus (preductal); this pattern is associated with Turner's syndrome. In the adult form, the stenosis is distal to the ductus arteriosus (postductal) and is associated with notching of the ribs (secondary to continued pressure from the aorta on them), hypertension in the upper extremities, and weak pulses in the lower extremities. Headache, cold extremities, and lower extremity claudication with exercise are typical if the patient is symptomatic (many adults with mild distal coarctation may remain asymptomatic for years). Upper extremity hypertension with weak pulses in the lower extremities, and a midsystolic (or continuous) murmur over the chest or back may be the only obvious signs in some. Note that the chronic cough is probably related to the man's smoking, and is not caused by the coarctation. Eisenmenger's syndrome (choice B) is a shift from a left-to-right shunt to a right-to-left shunt secondary to pulmonary hypertension. Tetralogy of Fallot (choice C) and transposition of great arteries (choice D) cause cyanosis and are usually diagnosed in infancy. Ventricular septal defect (choice E) might remain undiagnosed until adulthood, but would not cause notching of the ribs. -------------------------------------------------------------------------------32>The correct answer is C. The muscularis of the upper third of the esophagus (choice E) is composed entirely of striated muscle. The middle third (choice C) contains both striated and smooth muscle. The lower third (choice B) and lower esophageal sphincter (choice A) contain only smooth muscle. There is no such thing as the upper esophageal sphincter (choice D). -------------------------------------------------------------------------------33>The correct answer is A. Bilirubin is a degradative product of hemoglobin metabolism. Bilirubin (pigment) stones are specifically associated with excessive bilirubin production in hemolytic anemias, including sickle cell

anemia. Bilirubin stones can also be seen in hepatic cirrhosis and liver fluke infestation. Calcium oxalate stones (choice B) and cystine stones (choice E) are found in the kidney, rather than the gallbladder. Pure cholesterol stones (choice C) are less common than mixed gallstones, but have the same risk factors, including obesity and multiple pregnancies. Mixed stones (choice D) are the common "garden variety" gallstones, found especially in obese, middle aged patients, with a female predominance -------------------------------------------------------------------------------34> The correct answer is A. Malnutrition associated with chronic alcoholism can lead to a severe magnesium deficiency. The effect of low serum magnesium on parathyroid hormone secretion (PTH) depends on severity and duration. An acute decrease in serum magnesium will increase PTH secretion, while a prolonged severe deficiency results in decreased PTH secretion. There is also evidence that the action of PTH is decreased with chronic magnesium deficiency. Hence, this patient is suffering from "functional" hypoparathyroidism. The low serum calcium can produce weakness, tremors, muscle fasciculations, and seizures. A positive Trousseau's sign indicates the presence of latent tetany. It is observed by inflating a blood pressure cuff above systolic blood pressure for at least 2 minutes. A positive reaction consists of the development of carpal spasm, with relaxation occurring within seconds after deflating the cuff. In patients with magnesium deficiency, magnesium administration will produce a prompt rise in plasma PTH with subsequent restoration of serum calcium concentration to normal. With functional hypoparathyroidism bone density would be decreased (not increased, choice C). The combination of decreased PTH secretion (not increased, choice D) and decreased effectiveness of PTH produce hypocalcemia and hyperphosphatemia (not hypophosphatemia, choice B). Urinary cAMP would probably be decreased (not increased, choice E), given the low PTH. -------------------------------------------------------------------------------35>The correct answer is E. Enteropeptidase, formerly called enterokinase, activates trypsinogen by limited proteolytic digestion to give trypsin. Trypsin is itself capable of activating trypsinogen, which produces a positive feedback effect. Trypsin also activates chymotrypsinogen (and several other proteolytic enzymes), so deficiency of enteropeptidase results in a severe deficiency of enzymes that digest protein. Amylase (choice A) aids in the breakdown of starches to oligosaccharides, maltose, and maltotriose. Colipase (choice B), along with other lipases, functions to digest fats. Lactase (choice C) is a brush-border disaccharidase that hydrolyzes the bond between galactose and glucose in lactose. Pepsin (choice D) is a proteolytic enzyme secreted in an inactive form (pepsinogen) by the chief cells of the stomach. Pepsinogen is activated by stomach acid, and so is not dependent on enteropeptidase. Pepsin alone will not replace the activities of other proteolytic enzymes, partly because food does not remain in the stomach for an extended period of time. --------------------------------------------------------------------------------

36>The correct answer is E. The child is likely suffering from cystic fibrosis. In this disorder, an abnormality of chloride channels causes all exocrine secretions to be much thicker, and more viscous than normal. Pancreatic secretion of digestive enzymes is often severely impaired, with consequent steatorrhea and deficiency of fat-soluble vitamins, including vitamin A. Cystinuria (choice A) is a relatively common disorder in which a defective transporter for dibasic amino acids (cystine, ornithine, lysine, arginine; COLA) leads to saturation of the urine with cystine, which is not very soluble in urine, and precipitates out to form stones. Hypoglycemia (choice B) is not a prominent feature of children with cystic fibrosis who are on a normal diet. Hyperglycemia may occur late in the course of the disease. Iron deficiency anemia (choice C) is not found with any regularity in children with cystic fibrosis. Sphingomyelin accumulation (choice D) is generally associated with deficiency of sphingomyelinase, as seen in Niemann-Pick disease. Part 2/6-50 mcqs <1>A 37-year-old female presents to the emergency room with a fever. Chest x-ray shows multiple patchy infiltrates in both lungs. Echocardiography and blood cultures suggest a diagnosis of acute bacterial endocarditis limited to the tricuspid valve. Which of the following is the most probable etiology? A. Congenital heart disease B. Illicit drug use C. Rheumatic fever D. Rheumatoid arthritis E. Systemic lupus erythematosus Answer -------------------------------------------------------------------------------<2>Which of the following is the primary opsonin in the complement system? A. C1q B. C3b C. C5 D. C5a E. Factor B Answer -------------------------------------------------------------------------------<3>A patient is referred to a neurologist because of ataxia. Neurological examination reveals a loss of proprioception and a wide-based, slapping gate Magnetic resonance imaging reveals degeneration of the dorsal columns and dorsal roots of the spinal cord. Which of the following organisms is most likely to have caused this pattern of damage? A. Haemophilus influenzae

B. Herpes simplex I C. Neisseria gonorrhoeae D. Neisseria meningitidis E. Treponema pallidum Answer -------------------------------------------------------------------------------<4>A 24-year-old woman in her third trimester of pregnancy presents with urinary frequency and burning for the past few days. She denies fever, nausea, vomiting, or chills. She takes no medications besides prenatal vitamins and is generally in good health. Physical examination is remarkable for mild suprapubic tenderness, and a urine dipstick is positive for white blood cells, protein, and a small amount of blood. Culture produces greater than 100,000 colonies of gram-negative bacilli. Which of the following attributes of this uropathogenic organism is most strongly associated with its virulence? A. Bundleforming pili B. GVVPQ fimbriae C. Heat labile toxins D. Heat stable toxins E. P pili F. Type 1 pili Answer -------------------------------------------------------------------------------<5>A previously healthy 11-year-old girl develops a gastrointestinal infection with cramping and watery stools. After several days, she begins to pass blood per rectum, and is hospitalized for dehydration. In the hospital, she is noted to have decreasing urine output with rising blood urea nitrogen (BUN). Total blood count reveals anemia and thrombocytopenia, and the peripheral smear is remarkable for fragmented red cells (schistocytes). Infection with which of the following bacterial genera is most likely responsible fo r this syndrome? A. Campylobacter B. Clostridium C. Salmonella D. Shigella E. Vibrio Answer -------------------------------------------------------------------------------<6>Five days after returning to his military base in South Carolina after survival training in the nearby countryside, an 18-year-old recruit reports to the infirmary complaining of a headache. Physical examination reveals a fever, but no other abnormalities are noted. A few days later he returns

to the infirmary with a maculopapular rash involving the hands and feet. The rash then spreads centripetally to involve the trunk. Which of the following diseases should be suspected? A. Chickenpox B. German measles C. Measles D. Mumps E. Rocky Mountain spotted fever Answer -------------------------------------------------------------------------------<7>A 32-yearold, blood type A positive male receives a kidney transplant from a blood type B positive female donor with whom he had a 6-antigen HL A match. Once the kidney is anastomosed to the man's vasculature, the transplant team immediately begins to observe swelling and interstitial hemorrhage. After the surgery, the patient developed fever and leukocytosis and produced no urine. Which of the following is the most likely explanation? A. Acute rejection due to antibody-mediated immunity B. Acute rejection due to cell-mediated immunity C. Chronic rejection due to cell-mediated immunity to minor HLA antigens D. Hyperacute rejection due to lymphocyte and macrophage infiltration E. Hyperacute rejection due to preformed ABO blood group antibodies Answer -------------------------------------------------------------------------------<8>A sexually active 18-year-old woman presents with a fever of 102 F for the past 24 hours and lower abdominal pain and anorexia for the past 5 days. On physical examination, there is generalized tenderness of the abdomen, and the cervix is erythematous with motion tenderness. There is no rash nor any lesions on the external genitalia. A smear of the odorless cervical discharge contains sloughed epithelial cells and scant neutrophils. Which of the following would likely be found in the exudate? A. A naked, icosahedral double-stranded circular DNA virus B. Iodine-staining intraepithelial inclusion bodies C. Intraneutrophilic gram-negative diplococci D. Intranuclear "owl's eye" inclusion bodies E. Lactose-fermenting gram-negative bacilli F. Pear-shaped flagellated protozoa G. Pleomorphic, gram-negative rods H. Spirochetes on dark-field microscopy Answer -------------------------------------------------------------------------------<9>A 45-year-old homeless man has a chronic cough, a cavitary lesion of the

lung, and is sputum positive for acid-fast bacilli. Which of the following is the principle form of defense by which the patient's body fights this infection? A. Antibody-mediated phagocytosis B. Cell-mediated immunity C. IgA-mediated hypersensitivity D. IgEmediated hypersensitivity E. Neutrophil ingestion of bacteria Answer -------------------------------------------------------------------------------<10>A 15-year-old girl in a rural community has swollen, painful lymph nodes in her right axilla. Physical examination reveals multiple scratches on her right arm with a papule associated with one of the scratch marks. She states that the scratches occurred about 5 days ago. What type of animal is the most likely source of the infection? A. Cat or kitten B. Chicken C. Dog or puppy D. Horse E. Parrot Answer -------------------------------------------------------------------------------<11>A 38-year-old AIDS patient presents to the clinic complaining of nausea, occasional vomiting and "bumps" on his groin. On physical examination, multiple, nontender, pedunculated reddish purple nodules in the inguinal and perirectal areas are observed. The patient's liver is palpable 8 cm below the right costal margin. Routine laboratory tests are unremarkable except for an alanine aminotransferase level of 58 and alkaline phosphatase of 90. He denies any foreign travel, but has two pet cats . Which of the following is the most likely cause of this patient's infection? A. Bartonella henselae B. Human papillomavirus C. Molluscum contagiosum virus D. Rickettsia prowazekii E. Treponema pallidum Answer -------------------------------------------------------------------------------<12>A 49-year-old Vietnamese man is diagnosed with tuberculosis. On physical examination, large flocculent masses are noted over the lateral lumbar back , and a similar mass is located in the ipsilateral groin. This pattern of involvement strongly suggests an abscess tracking along the A. adductor longus

B. gluteus maximus C. gluteus minimus D. piriformis E. psoas major -------------------------------------------------------------------------------Answer <13>A 25year-old man presents with a high fever and generalized malaise. His condition deteriorates so rapidly that his friends decide to take him to the emergency department 24 hours after the onset of symptoms. He has a history of intravenous drug abuse. A test for anti-HIV antibodies is negative. Physical examination reveals a systolic murmur, and echocardiography shows bulky vegetations attached to the tricuspid valve leaflets. Which of the following microorganisms will be most likely be isolated from this patient's blood cultures? A. Candida albicans B. Hemophilus influenzae C. Staphylococcus aureus D. Staphylococcus epidermidis E. Viridans (a-hemolytic) streptococci Answer -------------------------------------------------------------------------------<14>A 23-year-old man develops explosive watery diarrhea with blood, fecal leukocytes, and mucus approximately 3 days after eating chicken that was improperly cooked. Comma-shaped organisms were found in the fecal smea r along with red blood cells and leukocytes. Which of the following pathogens is the most likely cause of these symptoms? A. Campylobacter jejuni B. Enterotoxigenic E. coli C. Shigella sonnei D. Staphylococcus aureus E. Vibrio cholera Answer -------------------------------------------------------------------------------<15>An otherwise healthy 3-year-old child is brought to the pediatrician with umbilicated, flesh-colored papules on his trunk. This condition is related to infection with which of the following viruses? A. Cytomegalovirus B. Herpesvirus 6 C. Parvovirus D. Poxvirus E. Variola Answer -------------------------------------------------------------------------------<16>A 47year-old male presents with declining renal function characterized by oliguria, elevated blood urea nitrogen and creatinine, and hematuria . He also complains of nasal congestion and epistaxis. Review of systems is notable for occasional cough and hemoptysis. Examination shows mucosal ulceration and nasal septal perforation, but no polyps. Which of the following serum markers would likely be present in this case? A. Anti-centromere antibody B. Anti-Ro C. Anti-SS-B D. c-ANCA (cytoplasmic antinuclear cytoplasmic antibody)

E. Decreased erythrocyte sedimentation rate (ESR) Answer -------------------------------------------------------------------------------<17>A child with sickle cell anemia is seen in a hematology clinic. Her mother states that she has been feeling very tired lately, and may have "come down with a virus." On physical examination, the girl is very pale, and a complete blood count shows severe anemia. A bone marrow aspirate contains no erythroid precursor cells. The girl was probably infected with which of the following viruses? A. Coxsackie virus B. Echovirus C. Hepadnavirus D. Herpes virus E. Parvovirus Answer -------------------------------------------------------------------------------<18>Zygomycosis, a destructive fungal infection of the sinuses, is likely to reach the brain by which of the following routes? A. Cavernous sinus B. External carotid artery C. Internal carotid artery D. Superior sagittal sinus E. Superior vena cava Answer -------------------------------------------------------------------------------<19>A newborn in the neonatal intensive care unit becomes tachypneic and irritable. Blood cultures grow a gram-positive coccus in chains that is presumed to be a streptococcus. Which of the following characteristics would help to differentiate Streptococcus agalactiae from Streptococcus pneumoniae? A. Alpha-hemolysis B. Carbohydrate capsule C. Cytochrome enzyme system D. Growth in bile E. Oxacillin sensitivity Answer -------------------------------------------------------------------------------<20>A 16-year-old girl presents with a painlessly enlarged lymph node in her right axilla. Peripheral blood counts are within normal limits. The lymph node is biopsied, and numerous granulomas filled with neutrophils and necrotic debris are observed. Which of the following organisms could produce this disease? A. Bartonella henselae B. Borrelia burgdorferi C. Chlamydia psittaci D. Coxiella burnetii E. Rickettsia prowazekii Answer -------------------------------------------------------------------------------<21>A 36-year-old mother of two children presents with a 4-day history of swollen, painful hands. Her wrists and metacarpophalangeal joints are boggy and inflamed bilaterally. Her 5-year-old son had been sent home from school approximately 3 weeks previously with red cheeks and a blotchy rash on his torso. What is the most likely diagnosis? A. Listeriosis B. Lyme disease

C. Mumps D. Parvovirus E. Reiter's syndrome Answer -------------------------------------------------------------------------------<22>A 37-year-old, intravenous drug-abusing male presents with fever and chills. Blood cultures are positive for Staphylococcus aureus. He develops central nervous system symptoms, and a cerebral abscess is suspected. Which part of the brain is most often affected by septic emboli in patients with infective endocarditis? A. Brainstem B. Cerebellum C. Frontal lobe D. Occipital lobe E. Parietal lobe Answer -------------------------------------------------------------------------------<23>Which of the following organisms is the most common cause of community-acquired pneumonia? A. Chlamydia pneumoniae B. Haemophilus influenzae C. Mycoplasma pneumoniae D. Staphylococcus aureus E. Streptococcus pneumoniae Answer -------------------------------------------------------------------------------<24>One week following a visit to the woods along an Eastern seaboard beach, a 50-year-old woman develops fever, headache, chills, and fatigue. A blood smear demonstrates protozoa within erythrocytes. Which of the following is the most likely pathogen? A. Babesia microti B. Leishmania donovania C. Plasmodium falciparum D. Plasmodium vivax E. Trypanosoma cruzi Answer -------------------------------------------------------------------------------<25>A 20-year-old female presents with a two day history of dysuria and increased urinary frequency. She states that she was recently married and was not sexually active prior to the marriage. Physical exam reveals a temperature of 100.7 F with normal vital signs. Gynecological exam reveals no evidence of discharge, vaginitis, or cervicitis. Urinalysis reveals 14 white blood cells per high-powered field with many gram-negative rods. The most appropriate therapy would be A. ampicillin B. ceftriaxone C. fluconazole D. gentamicin E. metronidazole Answer -------------------------------------------------------------------------------Answers Now you can see what's hidden1>The correct answer is B. The most probable etiology of bacterial endocarditis involving the tricuspid valve is illicit intravenous drug use, which can introduce skin organisms into the venous system that then attack the tricuspid valve. Staphylococcus aureus accounts for between 60% and 90% of cases of endocarditis in

intravenous drug users. The endocarditis associated with congenital heart disease (choice A) typically involves either damaged valves or atrial or ventricular septal defects. The tricuspid valve is not particularly vulnerable. Rheumatic fever (choice C) most commonly damages the mitral and aortic valves, and tricuspid damage is usually less severe and seen only when the mitral and aortic valves are heavily involved. Consequently, secondary bacterial endocarditis involving only the tricuspid valve in a patient with a history of rheumatic fever would be unusual. Rheumatoid arthritis (choice D) is not associated with bacterial endocarditis. Systemic lupus erythematosus (choice E) can produce small, aseptic vegetations on valves, but is not associated with bacterial endocarditis. -------------------------------------------------------------------------------2>The correct answer is B. C3b is the most critical molecule in both the classical and alternative complement pathways. C3, the most abundant protein of all the complement proteins, is cleaved into C3a and C3b. C3b attaches to bacterial surfaces for opsonization by phagocytes. C3a binds to mast cells and basophils, activating them and producing histamine release. C1q (choice A) is a complement component in the classical pathway. It binds to the constant heavy domain of an IgG molecule that has reacted with the bacterial surface epitope. C1q is not involved in opsonization as it simply functions as an enzyme in the early complement cascade. C5 (choice C) is a protein, that once split into C5b, initiates the assembly of the membrane attack complex. This complex consists of C5b, C6, C7, C8, and polymerization of C9, and is responsible for lysis of the bacteria. C5a is a strong chemotactic molecule. This fragment is the result of C5 being split by the C5 convertases of both pathways. This C5a (choice D) is a strong chemotactic factor for neutrophils and results in stimulating the inflammatory response. Factor B (choice E) is an activator protein of the alternative pathway. It combines with C3b to form C3bBb. This C3bBb is the C3 convertase of the alternative pathway. -------------------------------------------------------------------------------3>The correct answer is E. The findings described are those of tabes dorsalis, a form of tertiary syphilis caused by Treponema pallidum. Tabes dorsalis, and other forms of tertiary syphilis, are now uncommon in this country, possibly because the common use of antibiotics may "treat" many unsuspected cases of syphilis. Haemophilus influenzae (choice A) and Neisseria meningitidis (choice D) can cause meningitis. Neisseria gonorrhoeae (choice C) causes gonorrhea, which usually does not involve the CNS. Herpes simplex I (choice B) can cause an encephalitis that typically involves the frontal and temporal lobes. -------------------------------------------------------------------------------4>The correct answer is E. Urinary tract infections are the most common bacterial infections encountered during pregnancy, and Escherichia coli is the most commonly isolated organism. 70% of cases in the U.S. are caused by P pili-positive strains. Bundle-forming pili (choice A) are found in enteroaggregative E. coli (EAEC). GVVPQ fimbriae (choice B) are found in EAEC. Heat labile toxins (choice C) are pathogenic factors in enterotoxic strains (ETEC). Heat stable toxins (choice D) are pathogenic factors in ETEC or EAEC. Type 1 pili (choice F) are a major pathogenic factor in ETEC. -------------------------------------------------------------------------------5>The correct answer is D. This patient has developed hemolytic-uremic syndrome (HUS), a complication of the Shiga toxin or Shiga-like toxin: exotoxins released by Shigella species and the enterohemorrhagic E.coli. HUS in children usually develops after a gastrointestinal or flu-like illness, and is characterized by bleeding, oliguria, hematuria

and microangiopathic hemolytic anemia. Presumably the Shiga toxin is toxic to the microvasculature, producing microthrombi that consume platelets and RBCs, and may fragment the red cell membrane. The incorrect choices are all bacteria which may produce an enterocolitis, but do not elicit HUS. A long-term consequence of Campylobacter (choice A) infection is a reactive arthritis or full-blown Reiter's syndrome. Clostridial enterocolitis is produced by Clostridium difficile (choice B), a normal inhabitant of the gut that produces pseudomembranous colitis when other gut flora are suppressed by treatment with antibiotics. In the United States, Salmonella infections (choice C) are almost all non-typhoid inflammatory diarrhea, producing a simple enterocolitis that may proceed to sepsis in some cases. Typhoid fever (produced by Salmonella typhi and S. paratyphi) produces a protracted illness that progresses over several weeks and includes rash and very high fevers, but not HUS. Vibrio (choice E) infections produce copious amounts of watery diarrhea, and the major risk of cholera and other Vibrio enteritides is shock due to hypovolemia or electrolyte loss. -------------------------------------------------------------------------------6>The correct answer is E. Take rashes involving the palms and soles (otherwise unusual sites) very seriously: only a small number of infections can cause this pattern, including Rocky Mountain spotted fever, meningococcemia, and secondary syphilis. Rocky Mountain spotted fever is caused by the rickettsia R. rickettsii, and is found throughout the United States, particularly in south central and eastern portions (not the Rocky Mountains). 3-12 days after a tick bite, patients develop malaise, frontal headache, and fever. Several days later, the rash described in the question stem develops. Other manifestations can include hepatosplenomegaly, thrombocytopenia, and (potentially fatal) disseminated intravascular coagulation. Chickenpox (choice A), or varicella, is characterized by maculopapules that evolve into vesicles over hours to days, then eventually form crusts. Typical lesions first appear on the trunk and face and rapidly spread to involve other areas. The maculopapular rash of German measles (choice B), or rubella, usually begins on the face, then spreads down the body. Although the maculopapular rash of measles (choice C), or rubeola, can include the palms and soles, it typically begins along the hairline in frontal and temporal regions, then spreads down the trunk to the limbs. Mumps (choice D) is characterized by fever, malaise, parotitis, and orchitis, but not a rash. -------------------------------------------------------------------------------7>The correct answer is E. The patient is suffering from hyperacute rejection due to the preformed anti-B ABO blood group antibody found in all type A positive individuals. Hyperacute rejection occurs within minutes to a few hours of the time of transplantation, and is due to the destruction of the transplanted tissue by preformed antibodies reacting with antigens found on the transplanted tissue that activate complement and destroy the target tissue. Preformed antibodies can also be due to presensitization to a previous graft, blood transfusion, or pregnancy. Acute rejection due to antibody-mediated immunity (choice A) is incorrect because this patient suffered from hyperacute rejection (immediate) occurring within minutes to hours, rather than days. Acute rejection due to cell-mediated immunity (choice B) will not occur until several days or a week following transplantation. Acute rejection is due to type II and type IV reactions. Chronic rejection, due to the presence of cell-mediated immunity to minor HLA antigens (choice C), occurs in allograft transplantation months to even years after the transplant. Chronic rejection is generally caused by both humoral and cell-mediated immunity. An accelerated acute rejection,

occurring in 3-5 days, can be caused by tissue infiltration and destruction by presensitized T lymphocytes and macrophages (choice D) and/or

antibody-dependent, cell-mediated cytotoxicity (ADCC). Note that this is not a hyperacute reaction. -------------------------------------------------------------------------------8>The correct answer is B. The presentation is typical for pelvic inflammatory disease (PID). Chlamydia trachomata (serotypes D-K) is the most common bacterial cause of sexually transmitted disease (STD) in this country and is the most likely agent on the list to produce the symptoms described. It is an ATP-defective organism that must therefore live intracellularly in the human host and can be visualized inside epithelial cells with iodine, Giemsa, or fluorescent-antibody stains. The remainder of the answer choices refer to other agents that could be found in the female genital tract, either by sexual transmission or by contamination with fecal flora, but they are not the best choices. A naked, icosahedral double-stranded circular DNA virus (choice A) refers to human papilloma virus, which is the most common cause of STDs in the U.S., but presents with anogenital warts. Intraneutrophilic gram-negative diplococci (choice C) refers to Neisseria gonorrhoeae, which would be expected to present with dysuria and neutrophilic exudate. Intranuclear "owl's eye" inclusion bodies (choice D) refers to cytomegalovirus, a common STD in the United States, but not a common agent of PID. Most cases in average adults are manifested by mononucleosis-like symptoms. Lactose-fermenting gram-negative bacilli (choice E) would be consistent with Escherichia coli. Although this organism is the most common cause of urinary tract infections in women in the United States, it would not be expected to cause PID. Pear-shaped flagellated protozoa (choice F) refers to the protozoan parasite Trichomonas vaginalis, the only protozoan STD in the world. Infection would be characterized by a malodorous, cheesy exudate, and there would be more erythema of the external genitalia than of the cervix. Pleomorphic, gramnegative rods (choice G) are consistent with Hemophilus ducreyi, which causes chancroid, and presents in a different manner. Spirochetes on dark-field microscopy (choice H) refers to Treponema pallidum, the causative agent of syphilis, which would produce rash and/or chancre, depending on the stage of the infection. -------------------------------------------------------------------------------9>The correct answer is B. The principle host defense in mycobacterial infections (such as this patient's tuberculosis) is cell-mediated immunity, which causes formation of granulomas. Unfortunately, in tuberculosis and in many other infectious diseases characterized by granuloma formation, the organisms may persist intracellularly for years in the granulomas, only to be a source of activation of the infection up to decades later. While antibody-mediated phagocytosis (choice A) is a major host defense against many bacteria, it is not the principle defense against Mycobacteria. IgA-mediated hypersensitivity (choice C) is not involved in the body's defense against Mycobacteria. IgE-mediated hypersensitivity (choice D) is not involved in the body's defense against Mycobacteria. It is important in allergic reactions. Neutrophil ingestion of bacteria (choice E) is a major host defense against bacteria, but is not the principle defense against Mycobacteria. -------------------------------------------------------------------------------10>The correct answer is A. This patient has the classic symptoms of cat scratch disease caused by the bacillus Bartonella henselae. The disease is self-limited with the onset of symptoms occurring 3-10 days following an inoculating scratch. The organism can be isolated from kittens, typically less than 1 year of age, or from fleas. A history of a new kitten in the house and the papule at the site of a scratch with regional painful adenopathy

defines the classic scenario. Chickens (choice B) can harbor Salmonella spp. producing a gastroenteritis or enterocolitis. Chicken guano is also a favorable environment for the fungus Histoplasma capsulatum. The mycelial phase thrives in the rich soil. The human disease is a granulomatous infection involving the lungs and mimicking tuberculosis. Dogs or puppies (choice C) carry Capnocytophaga canimorsus as part of the normal flora

of the oral cavity. Infections from licking or biting range from a self-limited cellulitis to fatal septicemia. Patients at risk for more severe infections are those with asplenia, alcoholism, or hematologic malignancies. This organism is also associated with cat bites, but the patient develops cellulitis and fulminant septicemia, especially in asplenic patients. Pasteurella multocida is another pathogen that colonizes the nasopharynx and gastrointestinal tract of cats and dogs. Cats have the highest rate of colonization (5090%), followed by dogs (50%), swine (50%), and rats (14%). P. multocida most commonly causes a localized soft tissue infection or cellulitis after an animal bite, but systemic symptoms may be present in about 40% of the cases. These symptoms include osteomyelitis, septic arthritis, or tenosynovitis. Horses (choice D) and horse manure have been associated with a pulmonary opportunistic infection with cavitation caused by Rhodococcus equi that resembles tuberculosis in immunocompromised patients. Burkholderia mallei (the cause of glanders) is characterized by non-caseating granulomatous abscesses of skin, lymphadenopathy, and pronounced involvement of the lungs. Parrots (choice E) are associated with psittacosis caused by Chlamydia psittaci. Psittacosis is associated with a dry, hacking cough productive of scant sputum, an interstitial infiltrate in the lungs, severe headache, and myalgias. A pale macular rash is also seen. -------------------------------------------------------------------------------11>The correct answer is A. Bacillary angiomatosis is a disease that occurs primarily in AIDS patients, and is indicative of a defect in cell-mediated immunity. It is caused by either Bartonella henselae or Bartonella quintana. The domestic cat is the reservoir for these organisms and they are usually transmitted to humans via a cat scratch or cat bite. Patients with this illness usually have multiple skin lesions and extracutaneous manifestations involving liver and bone. Diagnosis is usually based on characteristic histopathologic findings including plump "epithelioid" endothelial cells and mitotic figures. A macrolide, such as erythromycin or azithromycin, is the drug of choice for the infection. Human papillomavirus (choice B) causes warts. Infection can present as a sessile wart or as condyloma acuminatum, which are fleshy soft growths that coalesce into large masses. When cellular immunity is depressed, as in AIDS, the condylomata acuminatum proliferate. Molluscum contagiosum virus (choice C) is a pox virus that is spread by close person-toperson contact. Infection produces a firm nodule that often becomes umbilicated, and may resolve by discharging its contents. In AIDS, the lesions do not resolve, but enlarge and spread. Rickettsia prowazekii (choice D) is the cause of epidemic typhus. It is spread by the human body louse, Pediculus humanis. Its reservoirs are humans and flying squirrels. Treponema pallidum (choice E) is the spirochete that causes syphilis. The characteristic primary lesion is a chancre (a painless, indurated ulcer) at the site of inoculation. -------------------------------------------------------------------------------12>The correct answer is E. This is the classic presentation of a psoas abscess. This clinical entity was formerly a fairly common complication of vertebral tuberculosis, but is now rare in clinical practice in this country. The psoas muscle is covered by a fibrous sheath known as the psoas fascia. This sheath is open superiorly, permitting an infection involving the soft tissues around the spine to enter the sheath, then track down to the groin. The adductor longus (choice A) is a muscle of the anterior thigh, and is not related to the lumbar portion of the back. The gluteus maximus (choice B) gluteus minimus (choice C) and piriformis (choice D) are muscles of the buttock with no relationship to the groin.

-------------------------------------------------------------------------------13>The correct answer is C. The patient has a fever and is extremely ill. The most important clue to the diagnosis is the presence of bulky vegetations on the tricuspid valve, indicating that he has infective endocarditis. On the basis of the rapid clinical course, this is likely a case of acute infective endocarditis. The diagnosis of this condition

must be confirmed by blood cultures, which are also necessary to determine bacterial antibiotic sensitivity. S. aureus (commonly present on the skin) is the most frequent etiologic agent of infective endocarditis in intravenous drug abusers. It commonly affects the tricuspid valve. Because of its high virulence, S. aureus-related endocarditis follows an acute course and may lead to death within a few days. The causative agents of infective endocarditis differ depending on host factors. Fungal organisms, such as Candida albicans (choice A), may cause infective endocarditis in severely immunosuppressed patients, such as those with AIDS. A minority of cases of infective endocarditis are caused by a number of normal commensals in the oral cavity, i.e., the "HACEK" group: Hemophilus (choice B), Actinobacillus, Cardiobacterium, Eikenella, and Kingella. S. epidermidis (choice D) and other coagulase-negative staphylococci tend to produce endocarditis in recipients of prosthetic valves. Viridans streptococci (choice E) are the most frequent agents causing endocarditis in previously abnormal valves, such as those damaged by rheumatic disease, or congenitally abnormal valves. Coagulasenegative staphylococci and viridans (ahemolytic) streptococci are less virulent than S. aureus and are thus associated with a subacute (more prolonged) clinical course and a better prognosis -------------------------------------------------------------------------------14>The correct answer is A. Campylobacter jejuni is a pathogen causing an invasive enteric infection associated with ingestion of raw or undercooked food products, or through direct contact with infected animals. In the U.S., ingestion of contaminated poultry that has not been sufficiently cooked is the most common means of acquiring the infection. The patients typically have bloody diarrhea, abdominal pain, and fever. The presence of fecal leukocytes indicates an invasive infection. The organism is a gram negative rod with a "comma-shape." Enterotoxigenic E. coli (choice B) causes the classic traveler's diarrhea. The infection is non-invasive and is acquired via the fecal-oral route through consumption of unbottled water or uncooked vegetables. The major manifestation is a copious outpouring of fluid from the GI tract presenting as explosive diarrhea. This is due to the action of one of two types of enterotoxins on the GI tract mucosa. Shigella sonnei (choice C) produces a syndrome very similar to C. jejuni. However, the organism appears as a gram-negative rod on Gram's stain. It does not have a comma shape. Transmission is from person to person via the fecal-oral route. Infection requires a low infective dose since the organism is fairly resistant to gastric acidity. Staphylococcus aureus (choice D) produces food poisoning due to the ingestion of a preformed enterotoxin. The organism is present in food that is high in salt content such as potato salad, custard, milk shakes, and mayonnaise. The patient presents with nausea, vomiting, and abdominal pain, followed by diarrhea beginning 1-6 hours after ingestion of the enterotoxin. Vibrio cholerae (choice E) produces a secretory diarrhea due to increases in cAMP in the intestinal cells. The organism is not invasive. The patient presents with the sudden onset of painless, watery diarrhea that becomes voluminous, followed by vomiting. The stool appears nonbilious, gray, and slightly cloudy with flecks of mucus, no blood, and a sweet odor. -------------------------------------------------------------------------------15>The correct answer is D. The lesions are characteristic of molluscum contagiosum, which is a typically benign and self-limited condition caused by a poxvirus. The disease can be transmitted either venereally or through non-venereal contact. The other viruses listed do not cause similar skin lesions. Patients with advanced HIV infection may develop a severe,

generalized, and persistent eruption, often involving the face and upper body. Cytomegalovirus (choice A) causes congenital infections and disseminated infections in immunosuppressed patients. Herpesvirus 6 (choice B)causes roseola (exanthem subitum). Parvovirus (choice C) causes aplastic crises in patients with hemolytic anemia. Variola (choice E) is the smallpox virus. --------------------------------------------------------------------------------

16>The correct answer is D. This patient has Wegener's granulomatosis, which is characterized by renal involvement, severe upper respiratory tract symptoms, and pulmonary involvement. Other organ systems may also be involved. The renal syndrome is a crescentic rapidly progressive glomerulonephritis leading to renal failure. The upper respiratory tract findings include sinus pain and drainage, and purulent or bloody nasal discharge with or without nasal ulcerations. Nasal septal perforation may follow. Pulmonary involvement may be clinically silent with only infiltrates present on x-ray, or it may present as cough and hemoptysis. c-ANCA is a marker for Wegener's granulomatosis, present in a high percentage of patients. Anti-centromere antibody (choice A) is associated with approximately 90% of cases with CREST syndrome (calcinosis, Raynaud's phenomenon, esophageal motility syndrome, sclerodactyly, and telangiectasia) which is also called limited scleroderma. Anti-Ro (choice B) is also called anti-SS-A and is associated with Sj gren's syndrome (7095%). Anti-SS-B (choice C) is associated with Sj gren's syndrome (60-90%). Decreased ESR (choice E) is not a marker of Wegener's. Instead, a markedly elevated ESR is seen. Additionally, mild anemia, thrombocytosis, leukocytosis, mild hypergammaglobulinemia (IgA), and mildly elevated rheumatoid factor are seen in this disorder. -------------------------------------------------------------------------------17>The correct answer is E. Parvoviruses are small single-stranded DNA viruses, of which only serotype B19 is pathogenic for humans. This virus causes three distinct syndromes: a childhood febrile rash known as erythema infectiosum ("Fifth disease"); aplastic crisis in individuals with chronic hemolytic diseases (sickle cell anemia, thalassemia, etc); and congenital infections that can present as stillbirth, hydrops fetalis (analogous to severe Rh incompatibility), or severe anemia. Coxsackie viruses (choice A) usually cause cold-like illness, but can cause herpangina, myocarditis, and meningitis. Echoviruses (choice B) can infect a variety of organ systems (GI, CNS, eyes, heart, respiratory, skin), but are not a cause of aplastic crises. Hepadnavirus (choice C) is the causative agent of hepatitis B. Herpes viruses (choice D) cause a variety of acute to chronic infections including herpes simplex types I and II, chicken pox, chronic herpes zoster, CMV infection, and Epstein Barr virus infections. -------------------------------------------------------------------------------18>The correct answer is A. The cavernous sinuses are located on either side of the body of the sphenoid bone, and become a potential route of infection because they receive blood both from the face (via the ophthalmic veins and sphenoparietal sinus) and some of the cerebral veins. The spread of infection, especially by Mucor sp., into the cavernous sinus, can produce either CNS infection or cavernous sinus thrombosis, both of which are potentially fatal. The route from the face to the brain is not arterial (choices B and C). The superior sagittal sinus (choice D) is located in the falx cerebri, and drains venous blood from the brain to other dural sinuses, from which it eventually drains into the jugular vein. Zygomycosis does not reach the brain by way of the superior sagittal sinus. The superior vena cava (choice E) drains blood from the upper part of the body into the heart. -------------------------------------------------------------------------------19>The correct answer is A. Streptococci are usually initially speciated by their hemolytic capacity on sheep blood agar. Beta-hemolytic streptococci include groups A, B, and D. S. agalactiae is the classic group B streptococcus. The non beta-hemolytic streptococci consist principally of the pneumococci and the viridans group. Both S. agalactiae and pneumococcus have a

carbohydrate capsule (choice B), an important virulence factor and means of subtyping streptococcal species. None of the streptococci utilize cytochrome enzymes (choice C). They derive all of their energy from the fermentation of sugars to lactic acid.

Neither pneumococcus nor S. agalactiae can grow in bile (choice D). This ability is specific for the enterococcus group (group D) of streptococci. Both pneumococci and S. agalactiae are usually treated with penicillin-type antibiotics, although group B streptococci require a penicillinase-resistant type such as oxacillin (choice E). -------------------------------------------------------------------------------20>The correct answer is A. Bartonella henselae is the infective agent of cat scratch disease, which generally presents as regional lymphadenopathy with or without low fevers and headaches. Bartonella is a gram-variable pleomorphic rickettsial organism that is introduced to the skin in a cat bite or scratch. It produces a self-limited granulomatous response in the draining lymph nodes. Borrelia burgdorferi (choice B) is a spirochetal organism that is transmitted by a tick bite (Ixodes spp.), producing Lyme disease. Lyme disease progresses from a skin rash to fevers, headache and pain over about one month. It may produce lymphadenopathy, but is not associated with granuloma formation. Chlamydia psittaci (choice C) infection occurs after contact with infected bird droppings and produces an atypical pneumonia. The central nervous system may also be involved, but lymph nodes are spared. Chlamydia trachomatis is the chlamydial species that typically produces suppurative nodal granulomas (lymphogranuloma venereum). Coxiella burnetii (choice D) infection is transmitted by inhaling dusts or drinking milk from infected mammals, especially sheep and cows. The disease in humans, Q fever, is marked by mild nonspecific symptoms or pneumonia, and may progress to myocarditis or hepatitis. Rickettsia prowazekii (choice E) produces epidemic (louse-borne) typhus, which is transmitted by body lice and produces a rash akin to Rocky Mountain spotted fever. Although the organism may reside in the lymph nodes in dormancy, it does not elicit granuloma formation. -------------------------------------------------------------------------------21>The correct answer is D. Parvovirus B19 causes erythema infectiosum, or Fifth disease. The 5-year-old boy has the classic "slapped cheek" appearance. Adults typically do not get the facial rash, but have arthralgias and arthritis. The symmetrical distribution of involved joints is similar to that in rheumatoid arthritis. The onset in adults is typically 3 to 4 weeks after exposure. Parvovirus infections may persist in immunosuppressed patients, resulting in red blood cell aplasia. Listeriosis (choice A) is caused by the gram-positive rod Listeria monocytogenes. Meningitis and bacteremia are common clinical manifestations. Elderly, neonates, pregnant women, and those taking steroids have the highest risk for infection. Lyme disease (choice B) is caused by Borrelia burgdorferi. It is the most common vectorborne disease (Ixodes ticks) in the U.S. The incidence is highest in the summer and fall. The white-footed mouse and the white-tailed deer are zoonotic reservoirs. From 3 to 32 days following a tick bite, the patient develops fever, lymphadenopathy, meningismus, and the characteristic rash (erythema migrans). The rash enlarges and resolves over 3 to 4 weeks. Sequelae include arthritis, carditis, and neurologic abnormalities. Mumps (choice C) is caused by a Paramyxovirus. The virus most commonly affects glandular tissue. Parotitis, pancreatitis, and orchitis are characteristic. Mumps meningoencephalitis is one of the most common viral meningitides. Mumps polyarthritis is most common in men between the ages of 20 and 30 years. Joint symptoms begin 1 to 2 weeks after the parotitis subsides and large joints are involved. Reiter's syndrome (choice E) is a seronegative, asymmetric arthropathy predominantly affecting the lower extremities. It may be triggered by a C. trachomatis infection. In addition to the arthritis, patients may

have urethritis (which is usually due to chlamydia), conjunctivitis, mucocutaneous disease such as balanitis, oral ulcerations, or keratoderma. Approximately 80% of patients are HLA-B27 po -------------------------------------------------------------------------------22>The correct answer is E. Embolization from infective endocarditis typically causes multiple, small parietal lobe abscesses. This "factoid" is worth knowing because some patients with infective endocarditis present with what clinically looks like multiple small

"strokes", and their treatable cardiac disease may be completely unsuspected. -------------------------------------------------------------------------------23>The correct answer is E. The most common bacteria implicated in communityacquired pneumonia is the pneumococcus, Streptococcus pneumoniae. Other organisms frequently implicated in patients less than age 60 without comorbidity include Mycoplasma pneumoniae, respiratory viruses, Chlamydia pneumoniae, and Haemophilus influenzae. When community-acquired pneumonia occurs in elderly patients or patients with comorbidity, aerobic gram-negative bacilli and Staphylococcus aureus are added to the list. The organisms listed in choices A, B, and C are important causes of community-acquired pneumonia, but are not the most frequent causes. Staphylococcus aureus (choice D) is an important cause of community-acquired pneumonia (particularly in the elderly and in patients with comorbidity), but is not the most frequent cause. -------------------------------------------------------------------------------24>The correct answer is A. The combination of a one week incubation period, the Eastern seaboard clue, and the intra-erythrocyte parasites strongly suggest Babesia microti as the infecting organism. The clinical disease is called babesiosis. The infecting protozoan is related to Plasmodium and is transmitted by the bite of the Ixodes dammini tick. Occasional cases have been transmitted by blood transfusions. Most patients are asymptomatic; symptomatic cases in reasonably healthy individuals have the features listed in the question stem. Rare severe cases, which may be fatal, may develop in severely debilitated or asplenic individuals and can be accompanied by severe hemolysis (up to 30% of RBCs may have the parasites) with subsequent hemoglobinuria, hemolysis, and renal failure. Leishmania donovania (choice B) is an intracellular tissue protozoan, not a blood protozoan. Plasmodium falciparum (choice C) and Plasmodium vivax (choice D) are malarial parasites not encountered on the Eastern seaboard. Trypanosoma cruzi (choice E) is an intracellular tissue protozoan, and does not infect blood cells. -------------------------------------------------------------------------------25>The correct answer is A. The patient's presentation is consistent with a simple urinary tract infection; there is a short history of dysuria, increased urinary frequency and the appearance of white blood cells and gram-negative rods in the urine. Urinary tract infections are common in women after they become sexually active. The infection is likely caused by urethral trauma during intercourse, which leads to bacterial contamination of the bladder. Since the majority of these infections are caused by Escherichia coli (a gramnegative rod), the most appropriate therapy would be ampicillin for around 10 days. Ceftriaxone (choice B) is the treatment of choice for uncomplicated infections with N. gonorrhoeae, now that most strains are resistant to penicillin. Intravenous ceftriaxone is a regimen reserved for the treatment of life-threatening infections. Fluconazole (choice C) is indicated for the treatment of vaginal candidiasis. Since there is no vaginal discharge and the patient has gram-negative rods in the urine, a diagnosis of vaginal candidiasis can be excluded. __________________ <26>An active intravenous drug abuser presents to the emergency department with fever of 5 days' duration, a cough occasionally productive of blood, and pleuritic chest pain. Petechiae are present in his mouth and conjunctivae, and splinter hemorrhages are visible under the fingernails. Which of the following test results would most likely confirm the identity of the causative agent? A. Antibodies to p24 capsid antigen

B. Antibodies to Trichinella spiralis antigen C. Blood culture of a catalase-positive, novobiocin-sensitive, gram-positive coccus D. Blood culture of a coagulase-positive, catalase positive, gram-positive coccus E. Blood culture of a gamma-hemolytic, grampositive coccus on bile-esculin agar F. Blood culture of an alpha-hemolytic, optochinresistant, gram-positive coccus G. Blood culture of an alpha-hemolytic, optochinsensitive, gram-positive coccus Answer -------------------------------------------------------------------------------<27>A 43-year-old executive presents to a physician with chronic, symmetric polyarthritis involving the knees. The man gives a history of having developed an extensive rash after a deer hunting trip in Connecticut several years earlier. He tells that he felt "sick" at the time, and developed knee pain that prevented him from climbing stairs for several months, but then partially resolved. Which of the following organisms is most likely etiologically related to the patient's arthritis? A. Fungus B. Gram-negative cocci C. Gram-negative rod D. Gram-positive cocci E. Spirochete Answer -------------------------------------------------------------------------------<28>A 34-year-old woman presents with fatigue, malaise, and swollen, tender joints. Physical examination is significant for a maculopapular eruption over sun-exposed areas, including the face. Examination of a peripheral blood smear reveals mild thrombocytopenia. Which of the following autoantibodies, if present, would be most specific for the diagnosis of the patient's disorder? A. Anti-centromere antibody B. Anti-IgG antibody C. Antinuclear antibody D. Anti-Sm (Smith antigen) antibody E. Anti-SS-A (Ro) antibody Answer -------------------------------------------------------------------------------<29>A 33-year-old single mother of two young children visits her physician because of an oral ulcer. A review of systems is significant for fatigue, myalgia, and joint pain. Laboratory results demonstrate leukopenia, and a high-titered antinuclear antibody. A speckled staining pattern due to anti-Sm is seen with immunofluorescence; urinary protein is elevated. Which of the following is the most likely diagnosis? A. Generalized fatigue

B. Goodpasture's syndrome C Mixed connective tissue disease D. Scleroderma E. Systemic lupus erythematosus Answer -------------------------------------------------------------------------------<30>A 67-year-old black man with a history of glucose-6-phosphate dehydrogenase (G-6-PD) deficiency presents with fever, irritative voiding symptoms, and perineal pain. Rectal examination is remarkable for a boggy, exquisitely tender prostate. A urine Gram's stain is positive for gram-negative rods. The risk for development of hemolytic anemia is highest if he receives high-dose, 21-day therapy with A. ampicillin B. cefaclor C. ciprofloxacin D. sulfamethoxazole/trimethoprim E. tetracycline Answer -------------------------------------------------------------------------------<31>A patient with colorectal cancer develops septicemia complicated by endocarditis. You would expect the blood cultures to grow A. Streptococcus agalactiae B. Streptococcus bovis C. Streptococcus pneumoniae D. Streptococcus pyogenes E. Streptococcus viridans Answer -------------------------------------------------------------------------------<32>A patient with a cavitary lung lesion coughs up sputum that contains thin, acid-fast positive rods. Which of the following features would most likely be associated with these bacteria? A. Nutritional requirement for factors V and X B. Streptokinase C. Toxic shock syndrome toxin D. Visible under dark field illumination E. Waxy envelope Answer -------------------------------------------------------------------------------<33>A 27-year-old woman presents to the emergency department complaining of 10-12 episodes of nonbloody diarrhea per day for the past 2 days, along with severe abdominal cramps, nausea, vomiting, and a low-grade fever. She states that she just returned from a vacation to Mexico. While in Mexico, she did not drink any of the local water and ate only cooked foods and a few fresh salads. If fecal leukocytes are present, the patient should most likely be empirically treated with A. acyclovir B. ciprofloxacin

C. mebendazole D. quinine E. tetracycline Answer -------------------------------------------------------------------------------<34>A 54-year-old woman suffering from influenza deteriorates and develops shaking chills and a high fever. Physical examination is remarkable for dullness to percussion at the left base and decreased breath sounds on the left. Chest x-ray confirms the diagnosis of lobar pneumonia, presumed to be caused by Streptococcus pneumoniae. The patient has no known drug allergies. Which of the following antibiotics would be most appropriate to treat the patient's condition? A. Cefotaxime B. Chloramphenicol C. Erythromycin D. Penicillin E. Vancomycin Answer -------------------------------------------------------------------------------<35>A 4-year-old boy is seen by his pediatrician for epistaxis. The patient has a history of multiple bacterial and viral respiratory tract infections and eczema. An uncle had similar problems. Physical examination is remarkable for multiple petechial lesions on the skin and mucous membranes. Serum IgE is increased, and platelets are decreased. Which of the following is the most likely diagnosis? A. Acquired hypogammaglobulinemia B. Ataxia telangiectasia C. DiGeorge syndrome D. Selective IgA deficiency E. Wiskott-Aldrich syndrome Answer -------------------------------------------------------------------------------<36>Which of the following organisms is most likely to be implicated as a cause of urethritis that persists after antibiotic therapy for gonorrhea? A. Actinomyces B. Chlamydia C. Mycobacteria D. Nocardia E. Rickettsia Answer -------------------------------------------------------------------------------<37>A 73-year-old woman with a history of diabetes presents with left ear pain and drainage of pus from the ear canal. She has swelling and tenderness over the left mastoid bone. Which of the following microorganisms is the most likely causative agent? A. Hemophilus influenzae

B. Klebsiella pneumoniae C. Mucor sp. D. Pseudomonas aeruginosa E. Streptococcus pyogenes Answer -------------------------------------------------------------------------------<38>: A 25-year-old female presents with a confluent maculopapular rash that began on her face, then spread downward over her trunk. She states that 3 days ago she started having a fever and headache, with bilateral pain associated with the front and back of her neck. She also complains of joint pain. Which of the following diseases does she most likely have? A. Infectious mononucleosis B. Lyme disease C. Roseola D. Rubella E. Rubeola Answer -------------------------------------------------------------------------------<39>A 24-year-old woman presents with a 3-day history of fever, chills, chest pain, and cough productive of rust-colored sputum. Past medical history includes a splenectomy 1 year ago. A chest x-ray film indicates consolidation of the right lower lobe. Blood cultures are positive for a-hemolytic gram-positive diplococci. Immunity to the causative organism is based on A. alternative complement pathway activation B. antibody to an ahelical coiled fimbria C. IgA antibodies to C carbohydrate D. IgG antibodies to C carbohydrate E. IgG antibodies to a surface acidic polysaccharide Answer -------------------------------------------------------------------------------<40>A 3-year-old male presents with a skin rash and epistaxis. He has had several, severe sinopulmonary infections. A careful history reveals that his maternal grandfather died of bleeding complications following an emergency cholecystectomy. What additional findings are likely in this case? A. A CD4/CD8 ratio of < 1.5:1 B. Cerebellar ataxia C. Elevated platelet count and high serum IgG, IgA, and IgE levels D. Low platelet count and low serum IgG levels E. Low platelet count and low serum IgM levels Answer -------------------------------------------------------------------------------<41> Which of the following characteristics most strongly suggests that the cell depicted above is a phagocytically active macrophage? A. Immunocytochemical detection of collagenase

B. Microvilli-covered surface C. Presence of receptors for IgG and complement D. Presence of secondary lysosomes throughout cytoplasm E. Shape of the nucleus Answer -------------------------------------------------------------------------------<42>A 27-year-old white male presents with a 3-week history of several swollen and painful toes and knees. He has a past history of conjunctivitis. He also describes some low back stiffness that is more severe in the morning. Which of the following is the most likely diagnosis? A. Gout B. Lyme disease C. Reiter's syndrome D. Rheumatoid arthritis E. Septic arthritis Answer -------------------------------------------------------------------------------------------------------------------------------------------------------------<43>A 1-year-old girl presents with a 2-day history of fever, vomiting, and watery, nonbloody diarrhea. On physical exam, she appears dehydrated. Which of the following best describes the most likely infecting organism? A. It has a complex double-stranded DNA genome B. It has a partially doublestranded circular DNA genome C. It has a segmented, double-stranded RNA genome D. It has a single-stranded circular RNA genome E. It has a single-stranded RNA genome Answer -------------------------------------------------------------------------------<44>A 46year-old woman visits her podiatrist to have several bunions removed from her right foot. She chooses conscious sedation rather than general anesthesia for this procedure. She is given intravenous midazolam to supplement the local anesthetics that are injected into her foot. Midway through the surgery, she suddenly becomes agitated, combative, and exhibits involuntary movements. The anesthesiologist determines that she is having a paradoxical reaction to the midazolam and immediately administers A. flumazenil B. glucagon C. naloxone D. nitrite E. protamine Answer -------------------------------------------------------------------------------<45>A 24-year-old woman attempts suicide by taking an overdose of diazepam. She is rushed to the emergency department, where the attending physician will most likely order which of the following treatments?

A. Acetylcysteine B. Atropine C. Bicarbonate D. CaNa2EDTA chelation E. Deferoxamine F. Ethanol G. Flumazenil H. Physostigmine I. Pralidoxime J. Protamine Answer -------------------------------------------------------------------------------<46>Which of the following enzymes is stimulated by glucagon? A. Acetyl-CoA carboxylase B. Glycogen phosphorylase C. Glycogen synthase D. HMG-CoA reductase E. Pyruvate kinase Answer -------------------------------------------------------------------------------<47>A 29-year-old epileptic sanitation engineer is maintained on primidone. Ultrastructural examination of a liver biopsy reveals increased amounts of smooth endoplasmic reticulum. This change is most closely related to increases in the activity of which of the following? A. P-450 system B. Purine degradation C. Pyrimidine synthesis D. Tricarboxylic acid (Krebs) cycle E. Urea cycle Answer -------------------------------------------------------------------------------<48> A pregnant southeast Asian immigrant presents for prenatal care. Her past medical history is significant for a severe illness 3 years ago characterized by fatigue, nausea, anorexia, vomiting, jaundice, joint pains, and generalized skin lesions that slowly disappeared. She has felt well recently. Which of the following laboratory tests should be ordered to investigate the patient's past illness? A. Hepatitis B surface antigen (HBsAg) B. IgG cytomegalovirus (CMV) antibody levels C. IgM antibody to HBsAg D. IgM antibody to hepatitis B core antigen E. Quantitation of hepatitis A virus (HAV) IgM antibody Answer -------------------------------------------------------------------------------<49>A research scientist is studying calcium fluxes in cultured cells

using confocal laser scanning microscopy. The magnitude of the signal (brightness) is proportional to the strength of the calcium flux. Stimulation of which of the following receptor types would be expected to produce the strongest signal? A. Alpha-1 adrenergic receptor B. Beta-1 adrenergic receptor C. Dopamine-1 receptor D. Muscarinic acetylcholine receptor E. Nicotinic acetylcholine receptor <50>A 10-year-old child is suspected of having pellagra because of chronic symptoms including diarrhea, a red scaly rash, and mild cerebellar ataxia. However, his diet is not deficient in protein and he appears to be ingesting adequate amounts of niacin. A sister has a similar problem. Chemical analysis of his urine demonstrates large amounts of free amino acids. Which of the following is the most likely diagnosis? A. Alkaptonuria B. Carcinoid syndrome C. Ehlers-Danlos syndrome D. Hartnup's disease E. Scurvy Part 3/6-50 mcqs 1>A 48-year-old white female secretary presents with progressive difficulty typing over the past month. She also notes that her hands begin to feel numb and weak after typing for long periods of time. Upon testing, which of the following deficits would be predicted? A. Difficulty in abducting the fifth finger B. Difficulty in adducting the thumb C. Difficulty in flexing digits two and three at the metacarpophalangeal joints D. Loss of sensation over the lateral half of the dorsum of the hand E. Loss of sensation over the lateral half of the palm F. Loss of sensation over the medial half of the dorsum of the hand G. Loss of sensation over the medial half of the palm Anwser -------------------------------------------------------------------------------2>The smooth part of the right atrium derives from which of the following embryonic structures? A. Bulbus cordis B. Primitive atrium C. Primitive ventricle D. Sinus venosus E. Truncus arteriosus Anwser -------------------------------------------------------------------------------3>A 66-yearold male presents with chronic fatigue. On examination, the patient is noted to have lymphadenopathy and an enlarged liver and spleen. Laboratory examination reveals a white blood cell count of 25,000/mL with

93% lymphocytes; the lymphocytes appear small and mature. Both the hematocrit and platelet counts are within normal limits; however, hypogammaglobulinemia is also noted. Which of the following agents is indicated for treatment of this patient's condition? A. Chlorambucil B. Cisplatin C. Dacarbazine D. Tamoxifen E. Vinblastine Anwser -------------------------------------------------------------------------------4>A patient who has suffered severe chest trauma in an automobile accident is found to have fluid in the right pleural space. A thoracentesis reveals the presence of chylous fluid in the pleural space, suggesting a rupture of the thoracic duct. In which regions of the thorax is the thoracic duct found? A. Anterior and middle mediastinum B. Anterior and superior mediastinum C. Middle and posterior mediastinum D. Middle and superior mediastinum E. Posterior and superior mediastinum Anwser -------------------------------------------------------------------------------5>A 15-year-old boy is evaluated by a clinician for failure to develop norma l male secondary sexual characteristics. Physical examination reveals small testes, a small penis, and gynecomastia. The boy has had some difficulties in school, and the parents say that the school psychometrist said he had an IQ of 90. This patient's condition is most likely to be related to which of the following? A. Deletion B. Nondisjunction of an autosomal chromosome C. Nondisjunction of a sex chromosome D. Non-Robertsonian translocation E. Robertsonian translocation Anwser -------------------------------------------------------------------------------6>Careful testing of the visual fields in a patient complaining of difficulty reading demonstrates a central scotoma involving one visual field. This defect is most likely due to a lesion involving which of the following structures? A. Macula B. Optic chiasm C. Optic radiations in the parietal lobe D. Optic radiations in the temporal lobe E. Optic tract Anwser -------------------------------------------------------------------------------7>A 70-year-old woman with a history of multiple small strokes reports to her physician that she has had multiple recent experiences that something or someone seemed very familiar, when in reality they were not familiar to her. This type of experience is called which of the following? A. Anterograde amnesia B. Confabulation

C. Deja vu D. Jamais vu E. Retrograde amnesia Anwser -------------------------------------------------------------------------------8>A 33-year-old single mother of two young children visits her physician because of an oral ulcer. A review of systems is significant for fatigue, myalgia, and joint pain. Laboratory results demonstrate leukopenia, and a high-titered antinuclear antibody. A speckled staining pattern due to anti-Sm is seen with immunofluorescence; urinary protein is elevated. Which of the following is the most likely diagnosis? A. Generalized fatigue B. Goodpasture's syndrome C. Mixed connective tissue disease D. Scleroderma E. Systemic lupus erythematosus Anwser -------------------------------------------------------------------------------9>Researchers are assessing the specificity of a screening procedure for breast cancer in a population of 3000 women whose mothers had the disease. The presence or absence of a malignant condition is established by conventional mammography, which is assumed to be definitive for the purposes of comparison. The following data are collected: Positive Negative Total Cancer 90 10 100 No cancer 360 2540 2900 Total 450 2550 3000 What is the specificity of the test? A. 90/100 B. 90/450 C. 2540/2550 D. 2540/2900 E. (90+2540)/3000 Anwser -------------------------------------------------------------------------------10>A patient develops fever, shortness of breath, and appears to be quite ill. X-ray demonstrates bilateral interstitial lung infiltrates. Bronchial washings demonstrate small "hat-shape" organisms visible on silver stain within alveoli. Which predisposing condition is most likely to be present in this patient? A. AIDS B. Congestive heart failure C. Pulmonary embolus D. Rheumatoid arthritis E. Systemic lupus erythematosus Anwser -------------------------------------------------------------------------------11>A 35-year-old retarded man with a strong history of mental retardation among male relatives undergoes genetic testing. His lymphocytes are cultured in a medium containing methotrexate and 4% of the metaphase chromosomes in the lymphocytes show a breakpoint at q27.3 on the X chromosome. This man is at increased risk for which of the following

cardiovascular disorders? A. Aortic stenosis B. Atrial septal defect C. Mitral valve prolapse D. Tricuspid atresia E. Ventricular septal defect Anwser -------------------------------------------------------------------------------12>While lying supine in bed eating, a child aspirates a peanut. Which of the following bronchopulmonary segments would this foreign object mos t likely enter? A. Apical segment of the left upper lobe B. Apical segment of the right upper lobe C. Medial segment of the right middle lobe D. Posterior basal segment of the left lower lobe E. Superior segment of the right lower lobe Anwser -------------------------------------------------------------------------------13>A 43-year-old man is brought to the general medicine clinic by his wife. She states that his memory has progressively deteriorated over the last several years, and that his personality has been changing. On examination, the physician notes abnormal, writhing movements of the man's limbs and hyperreactive patellar reflexes. An MRI of the head reveals a loss of volume in the neostriatum and cortex. A family history reveals that similar symptoms occurred in several members of the patient's family. Which of the following genetic mechanisms has been implicated in this disorder? A. Expanded trinucleotide tandem repeat B. Genomic imprinting C. Large deletion in one gene D. Single amino acid substitution E. Translocation Anwser -------------------------------------------------------------------------------14>During a routine physical examination of a 74-year-old man, a physician palpates a large, pulsating mass in the lower abdomen. To which of the following is this mass most likely etiologically related? A. Atherosclerosis B. Bacterial infection C. Congenital anomaly D. Cystic medial degeneration E. Syphilis Anwser -------------------------------------------------------------------------------15>The cell in the center of the electron micrograph above is important in wound healing and plays a role in the pathological process underlying Dupuytren's contracture. Which of the following cell types is depicted? A. Endothelial cell B. Myoepithelial cell C. Myofibroblast D. Pericyte E. Smooth muscle cell

Anwser -------------------------------------------------------------------------------16>Damage to which of the following structures might produce hair cell loss? A. Basilar membrane B. Organ of Corti C. Reissner's membrane D. Scala tympani E. Scala vestibuli Anwser -------------------------------------------------------------------------------17>A thyroid mass usually moves with swallowing because the thyroid gland is enclosed by which of the following fascia? A. Carotid sheath B. Investing layer of the deep cervical fascia C. Pretracheal fascia D. Prevertebral fascia E. Superficial fascia Anwser -------------------------------------------------------------------------------18>The blood from an 8-year-old boy was analyzed by flow cytometry . The exact number of B cells was counted. Which of the following cell surface markers was likely used to identify the B cells in this blood sample? A. CD3 B. CD4 C. CD8 D. CD19 E. CD56 Anwser -------------------------------------------------------------------------------19>A child who has had abnormal development of the membranous bones has a broad skull with associated facial and dental anomalies. Which other bones are most likely to also be affected? A. Clavicles B. Femurs C. Metatarsals D. Phalanges E. Tibias Anwser -------------------------------------------------------------------------------20>Which of the following drugs used in the treatment of noninsulin-dependent diabetes mellitus (NIDDM) has no effect on the secretion of insulin? A. Acetohexamide B. Chlorpropamide C. Glyburide D. Metformin E. Tolbutamide Anwser --------------------------------------------------------------------------------

21>A 57-year-old man presents with a steady, severe pain in the right hypochondrium, nausea, vomiting, and a temperature of 102 F. He states that his signs and symptoms began shortly after eating his favorite pizza with extra cheese, pepperoni, and sausage. Laboratory examination reveals a white blood cell count of 13,400/mL and a serum bilirubin value of 2.8 mg/dL. If hepatobiliary imaging reveals an obstructed cystic duct, which of the following agents would be the drug of choice for the treatment of this patient's pain? A. Meperidine B. Morphine C. Naproxen D. Oxycodone E. Propoxyphene Anwser -------------------------------------------------------------------------------22>After falling on his laterally outstretched arm, a patient suffered a dislocation of the glenohumeral joint. Which of the following nerves is most likely to have been injured from this dislocation? A. Axillary nerve B. Dorsal scapular nerve C. Lateral pectoral nerve D. Medial pectoral nerve E. Suprascapular nerve Anwser -------------------------------------------------------------------------------23>A 45-year-old woman presents to her physician because of a severe "sore throat." Physical examination demonstrates fever and an extremely tender, enlarged thyroid gland, but no throat erythema. Serum thyroid studies demonstrate a mild degree of hyperthyroidism. Two months later, the patient is asymptomatic, and thyroid function tests have returned to normal. She never again experiences difficulty with her thyroid function. Which of the following was the most likely cause of her hyperthyroidism? A. Diffuse nontoxic goiter B. Graves disease C. Hashimoto's thyroiditis D. Subacute granulomatous thyroiditis E. Subacute lymphocytic thyroiditis Anwser -------------------------------------------------------------------------------25>A patient has an insulin-secreting tumor that is localized to the tail of the pancreas. Which of the following would most likely be an associated finding during fasting? A. Glycosylated hemoglobin level is increased B. Plasma concentration of C peptide is decreased C. Plasma concentration of glucagon is decreased D. Plasma concentration of glucose is increased E. Plasma concentration of proinsulin is increased Anwser -------------------------------------------------------------------------------Answers Now you can see what's hidden1>The correct answer is C. This is a classic presentation of a patient with carpal tunnel syndrome, which typically affects females between the

ages of 40 and 60 who chronically perform repetitive tasks that involve movement of the structures that pass through the carpal tunnel. One important structure that passes through the carpal tunnel is the median nerve. Patients often note a tingling, a loss of sensation, or diminished sensation in the digits. There is also often a loss of coordination and strength in the thumb, because the median nerve also sends fibers to the abductor pollicis brevis, flexor pollicis brevis, and the opponens pollicis. A final function of the median nerve distal to the carpal tunnel is control of the first and second lumbricals which function to flex digits two and three at the metacarpophalangeal joints and extend interphalangeal joints of the same digits. Abduction of the fifth digit (choice A) is a function controlled by the ulnar nerve, which does not pass through the carpal tunnel. Adduction of the thumb (choice B) is a function of the adductor pollicis, which is the only short thumb muscle that is not innervated by the median nerve, but rather by the deep branch of the ulnar nerve. Sensation of the lateral half of the dorsum of the hand (choice D) is mediated by the radial nerve, which also does not pass through the carpal tunnel. Sensation over the lateral aspect of the palm (choice E) is mediated by the median nerve, however the branch innervating the palm (palmar cutaneous branch of the median nerve) passes superficial to the carpal tunnel. Sensation over the medial aspect of the dorsum of the hand (choice F) is mediated by the ulnar nerve. Sensation over the medial aspect of the palm (choice G) is mediated by the ulnar nerve. -------------------------------------------------------------------------------2> The correct answer is D. The smooth part of the right atrium (the sinus venarum) is derived from the sinus venosus. The coronary sinus and the oblique vein of the left atrium also derive from the sinus venosus. The bulbus cordis (choice A) gives rise to the smooth part of the right ventricle (conus arteriosus) and the smooth part of the left ventricle (aortic vestibule). The primitive atrium (choice B) gives rise to the trabeculated part of the right and left atria. The primitive ventricle (choice C) gives rise to the trabeculated part of the right and left ventricles. The truncus arteriosus (choice E) gives rise to the proximal part of the aorta and the proximal part of the pulmonary artery. -------------------------------------------------------------------------------3>The correct answer is A. Chronic lymphocytic leukemia (CLL) is typically a disease of the elderly, with 90% of cases occurring after the age of 50; the median age is 65. Patients will typically present with a complaint of chronic fatigue and/or lymphadenopathy. Approximately 50% of all patients with CLL present with an enlarged liver and/or spleen. CLL typically pursues an indolent course but can occasionally present as a rapidly progressive disease. The hallmark of CLL is the isolated lymphocytosis in which the white blood cell count is usually greater than 20,000/mL and between 75% and 98% of the circulating cells are small "mature" lymphocytes. Chlorambucil is classified as a nitrogen mustard, a subcategory of the alkylating agents. It is primarily used to treat chronic lymphocytic leukemia and ovarian carcinoma; it can also be used to treat Hodgkin's disease and various other lymphomas. Cisplatin (choice B) is an alkylating agent indicated for the treatment of metastatic testicular and ovarian tumors in combination with other agents. Dacarbazine (choice C) is a cytotoxic agent with alkylating properties. It is used as a single agent or in combination with other antineoplastics in the treatment of metastatic malignant melanoma, refractory Hodgkin's disease, and various sarcomas. Tamoxifen (choice D) is an antiestrogen hormone used in the palliative treatment of breast cancer in patients with estrogen-receptor-positive breast cancer. Vinblastine (choice E) is a mitotic

inhibitor antineoplastic agent indicated for the treatment of Hodgkin's disease and nonHodgkin's lymphomas, choriocarcinoma,

lymphosarcoma, and neuroblastoma, as well as various other types of cancer. -------------------------------------------------------------------------------4>The correct answer is E. The mediastinum is divided into four regions. The region above the manubriosternal junction (level of fourth thoracic vertebra) is the superior mediastinum. The region below the manubriosternal junction is divided into the anterior mediastinum (anterior to the pericardium), the middle mediastinum (within the pericardium), and the posterior mediastinum (posterior to the pericardium). The thoracic duct enters the thorax through the aortic hiatus of the diaphragm. At this point it lies in the posterior mediastinum, the region posterior to the pericardium. As it ascends through the thorax and passes the level of the fourth thoracic vertebra, it enters the superior mediastinum. The anterior mediastinum (choices A and B) contains the thymus and fatty connective tissue. The middle mediastinum (choices A, C, and D) contains the heart and the roots of the great vessels. In addition to the thoracic duct, the superior mediastinum (choices B, D, and E) contains the ascending aorta, aortic arch, branches of the aortic arch, descending aorta, superior vena cava, brachiocephalic veins, thymus, trachea and esophagus. In addition to the thoracic duct, the posterior mediastinum (choices C and E) contains the descending aorta, azygos vein, hemiazygos vein and the esophagus -------------------------------------------------------------------------------5The correct answer is C. The boy probably has Klinefelter's syndrome (47, XXY), which has the typical presentation described in the question. The condition arises as a result of failure of separation (nondisjunction) of the sex chromosomes, and can be related to either paternal nondisjunction (slightly more common) or maternal nondisjunction. Deletions (choice A) are a common form of genetic disease and contribute to many genetic recessive diseases. Examples of nondisjunction of autosomes (choice B) include trisomies such as most cases of Down's syndrome (trisomy 21), Edwards' syndrome (trisomy 1, and Patau syndrome (trisomy 13). There are two types of translocations: non-Robertsonian (choice D) and Robertsonian (choice E). Non-Robertsonian (reciprocal) translocations result when two nonhomologous chromosomes exchange genetic material. Robertsonian translocations are a special type of translocation that involve exchange of genetic material from the long arms of one acrocentric chromosome to the long arms of another acrocentric chromosome, with fusion of the centromeres. Four percent of cases of Down's syndrome are caused by this mechanism. -------------------------------------------------------------------------------6>The correct answer is A. The probable location of lesions producing visual defects is a favorite USMLE topic (and is also well worth knowing if you have occasion to work up such a patient). Here is a list that may help you sort through these problems: Central scotoma ~ macula Ipsilateral blindness ~ optic nerve Bitemporal hemianopia ~ optic chiasm (choice B) Homonymous hemianopia ~ optic tract (choice E) Upper homonymous quadrantanopia ~ temporal optic radiations (choice D) Lower homonymous quadrantanopia ~ parietal optic radiations (choice C) Also, cortical lesions produce defects similar to those of the optic radiations, but may spare the macula. --------------------------------------------------------------------------------7>The correct answer is C. This is deja vu ("seen before"), which is the experience of an event, person, or thing as familiar, even though it has never previously been experienced. Severe cases often accompany an underlying neurologic problem. Anterograde amnesia (choice A) is the

inability to learn new facts. Most commonly, it involves both verbal and nonverbal material, but it can involve one or the other.

In confabulation (choice B), a patient reports "memories" of events that did not take place at the time in question. In jamais vu (choice D), a patient fails to recognize familiar events that have been encountered before. In retrograde amnesia (choice E), a patient fails to remember facts or events that occurred before the onset of amnesia. --------------------------------------------------------------------------------8>The correct answer is E. Systemic lupus erythematosus (SLE) is a prototype connective tissue disease. The diagnosis requires four criteria to be met from a list of eleven possible criteria: malar rash, discoid rash, photosensitivity, oral ulcers, arthritis, serositis, renal disorder, neurologic disorder, hematologic disorder, immunologic disorder, and antinuclear antibody. This patient also has anti-Sm, which is pathognomonic for SLE, but is only found in 30% of the affected patients. Antinuclear antibodies (ANA) are present in 95100% of cases of SLE; anti-double-stranded DNA is found in 70% of the cases. Generalized fatigue (choice A) due to being a single working mother of two children could well be a possibility, but the presence of the other criteria make SLE more likely. Goodpasture's syndrome (choice B) is characterized by linear disposition of immunoglobulin, and often C3, along the glomerular basement membrane (GBM). Glomerulonephritis, pulmonary hemorrhage, and occasionally idiopathic pulmonary hemosiderosis occur. Mixed connective tissue disease (choice C) is an overlap syndrome characterized by a combination of clinical features similar to those of SLE, scleroderma, polymyositis, and rheumatoid arthritis. These patients generally have a positive ANA in virtually 100% of the cases. High titer anti-ribonucleoprotein (RNP) antibodies may be present, generating a speckled ANA pattern. Anti-RNP is not pathognomonic for mixed connective tissue disease, since it can be found in low titers in 30% of the patients with SLE. Scleroderma (choice D) is characterized by thickening of the skin caused by swelling and thickening of fibrous tissue, with eventual atrophy of the epidermis. ANA are often associated with the disease, but the staining pattern is generally nucleolar. --------------------------------------------------------------------------------9>The correct answer is D. Specificity refers to how well a test identifies persons who do not have the disease in question. In the example, 2900 women do not have breast cancer, and 2540 of these women have a negative test. Specificity is given by (true negatives)/ (true negatives + false positives) = 2540/(2540+360) = 2540/2900. Choice A, 90/100, represents the sensitivity of the test; that is, the proportion of women with the condition who test positive. Choice B, 90/450, gives the predictive value of a positive test. Of the 450 women who tested positive, 90 actually had the condition. Choice C, 2540/2550, gives the predictive value of a negative test. Of the 2550 who tested negative, 2540 did not have cancer. Choice E, (90+2540)/3000, refers to the efficiency of a test; that is, the proportion of all subjects who were correctly classified by the test. --------------------------------------------------------------------------------10>The correct answer is A. The disease is Pneumocystis pneumonia, which is caused by an agent now believed to be a fungus rather than a true bacteria. Pneumocystis carinii pneumonia is seen in immunocompromised patients, particularly in those with AIDS, cancer, and in malnourished children. It can be the AIDS-defining illness. Congestive heart failure (choice B) predisposes the patient to pulmonary edema. Pulmonary embolus (choice C) can cause pulmonary infarction or sudden death. Rheumatoid arthritis (choice D), particularly in miners, can cause formation of lung nodules similar to subcutaneous rheumatoid nodules. Systemic lupus erythematosus (choice E) can cause pleuritis, but is

not associated with a significantly increased incidence of pneumonia. --------------------------------------------------------------------------------11>The correct answer is C. The disease is Fragile X Syndrome, which is a familial form of mental retardation that is roughly as common as Down's syndrome. The phenotype has a variable expression, but can include large head circumference at birth, perinatal

complications (premature birth, asphyxia, seizures), and possibly increased incidence of sudden infant death syndrome. Later, mental retardation, particularly involving language, and symptoms suggestive of attention deficit disorder and/or autism may appear. Features suggestive of connective tissue disorder (lax skin and joints, flat feet, large ears) are common. After puberty, there may be a long narrow face with prominent jaw and nasal bridge. Macro-orchidism is also common after puberty. Mitral valve prolapse and aortic root dilatation, which may appear in late adolescence or adulthood, are among the most serious complications of this disorder. Aortic regurgitation related to aortic root dilatation, not stenosis (choice A), can be a problem in this population. Common congenital cardiac malformations such as atrial septal defect (choice B) or ventricular septal defect (choice E) are not features of Fragile X syndrome, but can be seen with Down's syndrome. Tricuspid atresia (choice D) is a serious congenital cardiac malformation limiting flow into the right ventricle, but it is not part of Fragile X syndrome. --------------------------------------------------------------------------------12>The correct answer is E. Because the right main bronchus is wider and more vertical than the left, foreign objects are more likely to be aspirated into the right main bronchus. The superior segmental bronchus of the lower lobar bronchus is the only segmental bronchus that exits from the posterior wall of the lobar bronchi. Therefore, if a patient is supine at the time of aspiration, the object is most likely to enter the superior segmental bronchus of the lower lobe. None of the segmental bronchi of the left lung (choices A and D) are likely to receive the object because the object is less likely to enter the left main bronchus. The apical segment of the right upper lobe (choice B) is not likely to receive the foreign object because of the sharp angle that the upper lobar bronchus makes with the right main bronchus and the sharp angle that the apical segmental bronchus makes with the lobar bronchus. The medial segmental bronchus of the right middle lobe (choice C) arises from the anterior wall of the right middle lobar bronchus. Therefore, when the patient is supine, the effect of gravity will tend to prevent the object from entering this segmental bronchus. --------------------------------------------------------------------------------13>The correct answer is A. The disease described is Huntington's disease, which is now known to be related to an expanded trinucleotide tandem repeat on the short arm of chromosome 4. The number of trinucleotide repeats frequently increases in succeeding generations during the process of spermatogenesis. Prader-Willi and Angelman syndromes are frequently cited examples of genetic diseases involving genomic imprinting (choice B). Duchenne's muscular dystrophy is an example of a disease that is frequently due to a large deletion in a single gene (choice C). Single amino acid substitutions (choice D) are common in recessive diseases such as sickle cell anemia. Translocations (choice E) occur in disorders such as chronic myelogenous leukemia (CML; Philadelphia chromosome) and some cases of Down's syndrome. --------------------------------------------------------------------------------14>The correct answer is A. The mass is an abdominal aortic aneurysm, typically found in older men. Such aneurysms are almost always related to the formation of complicated atherosclerotic plaques in the aorta. Associated coronary artery disease is commonplace. Bacterial infection (choice B) can cause "mycotic" aneurysms; these usually involve smaller vessels. The small berry aneurysms that can involve the circle of Willis are congenital anomalies (choice C). Cystic medial degeneration (choice D) is related to the

development of dissecting aneurysms (actually dissecting hematomas). Tertiary syphilis (choice E) typically causes aneurysms of the root and arch of the aorta, rather than the descending aorta.

--------------------------------------------------------------------------------15>The correct answer is C. The cell is spindle-shaped like a fibroblast; however, the difference is that the cytoplasm contains several bundles of microfilaments. These bundles are parallel to the long axis of the cell and are seen immediately beneath the cell membrane and within the cytoplasm. Densities, comparable to Z-lines, can be seen along some of these bundles. The microfilaments are responsible for the contractile properties of this cell. These contractile properties, in addition to the cell's ability to link with collagen, function in wound closure in the healing process. Dupuytren's contracture, which is a contracture of the palmar fascia, is caused by interaction of these cells with collagen fibrils of the fascia. The endothelial cell (choice A) lines vessels. There are no vessels in the photomicrograph. The myoepithelial cell (choice B) contains microfilaments and is contractile. However, it is closely associated with glandular epithelium (not apparent here). The pericyte (choice D) is a multipotential connective tissue cell found near or around blood vessels, but it does not contain microfilament bundles such as these. There are no vessels apparent in the photomicrograph. The smooth muscle cell (choice E) is joined by junctions to other smooth muscle cells, arranged in bundles. Microfilaments make up most of the cytoplasm of such cells, with the nucleus in a central location. --------------------------------------------------------------------------------16>The correct answer is B. The structure of the cochlea is complex. The organ of Corti contains hair cells from the cochlear branch of the vestibulocochlear nerve (CN VIII). These cells rest on the basilar membrane (choice A), which separates the scala tympani (choice D) from the scala media. The hairs of the hair cells are embedded in the tectorial membrane, and movement of the basilar membrane below the cells tends to bend the hairs, which generates action potentials by the hair cells. The tectorial membrane that lies on the hair cells does not form a boundary between the different scala; the membrane separating the scala media from the scala vestibuli (choice E) is Reissner's (vestibular) membrane (choice C). --------------------------------------------------------------------------------17>The correct answer is C. The pretracheal layer of the cervical fascia runs from the investing layers in both sides of the lateral neck and splits to enclose the thyroid gland. Superiorly, it attaches to the laryngeal cartilages; inferiorly, it fuses with the pericardium. As a result of these connections, the thyroid gland moves with laryngeal movements. The carotid sheath (choice A) contains the vagus nerve, internal jugular vein, carotid artery, and lymph nodes. The investing layer of the deep cervical fascia (choice B) splits to enclose the trapezius and sternocleidomastoid muscles. The prevertebral fascia (choice D) covers muscles arising from the vertebrae. The superficial fascia (choice E) is immediately deep to the platysma muscle --------------------------------------------------------------------------------18>The correct answer is D. The best markers for identification of B cells are CD19, CD20, and CD21. The CD21 marker is a receptor for EBV (Epstein-Barr Virus). The CD3 marker (choice A) is present on all T cells with either a CD4 or CD8 marker. This is the marker that is used to identify total T cell count in a blood sample. The CD3 marker is used for signal transduction in the different T cells. The CD4 marker (choice B) is used to identify T helper cells. These are the cells that recognize exogenous peptides presented on MHC class II molecules by macrophages. CD4+ T helper cells are also involved in cellmediated delayed hypersensitivity, production of cytokines for stimulation of antibody production by B cells, and stimulation of macrophages. The CD8 marker (choice C) is

used to identify cytotoxic T cells. These are the cells that recognize viral epitopes attached to the MHC class I molecules of a virally infected cell. The CD56 marker (choice E) is used to identify NK(natural killer) cells. These cells are important in innate host defense, specializing in killing virally infected cells and tumor cells by secreting granzymes and perforins.

--------------------------------------------------------------------------------19>The correct answer is A. In a syndrome called cleidocranial dysostosis, absence of part of the clavicles accompanies a broad skull, and facial and dental anomalies. Note that you could also have answered this question by noting that of the bones listed, only the clavicles form by intramembranous ossification. The femurs (choice B), metatarsals (choice C), phalanges (choice D), and tibias (choice E) are cartilaginous (formed by endochondral ossification) rather than membranous bones. --------------------------------------------------------------------------------20>The correct answer is D. Metformin is a drug that is often used in conjunction with oral hypoglycemic agents for the treatment of NIDDM. Its mechanism of action is two-fold: (1) it decreases the production of glucose in the liver; (2) it increases the uptake of glucose in the liver. Metformin has no effect on the secretion of pancreatic insulin. Acetohexamide (choice A) is an oral hypoglycemic agent that is a sulfonylurea derivative. It stimulates secretion of insulin from the pancreas. Chlorpropamide (choice B) is an oral hypoglycemic agent that is a sulfonylurea derivative. It stimulates secretion of insulin from the pancreas. Glyburide (choice C) is a sulfonylurea derivative that stimulates insulin secretion from the pancreas. Tolbutamide (choice E) is a sulfonylurea derivative that stimulates insulin secretion from the pancreas. -------------------------------------------------------------------------------21The correct answer is A. The patient is presenting with signs and symptoms of acute cholecystitis, which is associated with gallstones in more than 90% of all cases. This condition occurs when a stone becomes impacted in the cystic duct and inflammation develops behind the obstruction. The acute attack is often precipitated by a large fatty meal, and is characterized by the sudden appearance of severe, steady pain localized to the epigastrium or right hypochondrium. Laboratory findings often include elevated white blood cells (2,000 - 15,000/mL). Total serum bilirubin values of 1-4 mg/dL may be seen in some instances, and serum amylase may be elevated. In noncomplicated cases, treatment often includes IV alimentation, analgesics, and antibiotics, as well as withholding of oral feedings. Meperidine is the narcotic of choice since it is least likely to cause spasm of the sphincter of Oddi, probably because of its antimuscarinic properties. It is therefore preferred over morphine (choice B), oxycodone (choice D), and propoxyphene (choice E). Furthermore, propoxyphene is a narcotic agonist with mild analgesic properties; hence, it would most likely not be able to effectively treat this patient's severe pain. Naproxen (choice C) is a nonsteroidal anti-inflammatory drug (NSAID) indicated for the treatment of mild to moderate pain; this agent would most likely not provide sufficient pain control for this patient. --------------------------------------------------------------------------------22>The correct answer is A. When the head of the humerus dislocates from the glenohumeral joint, it exits inferiorly, where the joint capsule is the weakest. Immediately inferior to the glenohumeral joint, the axillary nerve exits from the axilla by passing through the quadrangular space. At this location, the downward movement of the head of the humerus can stretch the axillary nerve. The axillary nerve innervates the deltoid muscle after leaving the axilla. The dorsal scapular nerve (choice B) passes along the medial border of the scapula to innervate the rhomboid muscles. The nerve does not pass in the region of the glenohumeral joint. The lateral and medial pectoral nerves (choices C and D) branch from the lateral and medial cords of the brachial plexus, respectively, and exit

through the anterior wall of the axilla to innervate the pectoralis major and minor. These nerves do not pass in the region of the glenohumeral joint. The suprascapular nerve (choice E) is a branch of the upper trunk of the brachial plexus and passes over the superior border of the scapula to innervate the supraspinatus and infraspinatus muscles. This nerve does not pass in the region of the glenohumeral joint.

--------------------------------------------------------------------------------23>The correct answer is D. This patient most likely has subacute granulomatous (de Quervain's) thyroiditis, which frequently develops after a viral infection. Microscopically, it is characterized by microabscess formation within the thyroid, eventually progressing to granulomatous inflammation with multinucleated giant cells. Clinically, patients may experience fever, sudden painful enlargement of the thyroid, and/or symptoms of transient hyperthyroidism. The disease usually abates within 6 to 8 weeks. Diffuse nontoxic goiter (choice A) by definition does not produce hyperthyroidism. The hyperthyroidism of Graves disease (choice B) does not spontaneously remit. Hashimoto's thyroiditis (choice C) can cause transient hyperthyroidism, but then goes on to cause hypothyroidism. Subacute lymphocytic thyroiditis (choice E) can cause transient hyperthyroidism, but is characteristically painless. -------------------------------------------------------------------------------25> The correct answer is E. Hypersecretion of insulin by a pancreatic b cell tumor is a major cause of fasting hypoglycemia (plasma glucose is not increased, choice D). Symptoms are related to neuroglycopenia and could include recurrent central nervous system dysfunction during fasting or exercise. While proinsulin only makes up approximately 20% of plasma immunoreactive insulin in normal individuals, in patients with an insulinoma it contributes 30-90% of the immunoreactive insulin. Hence, plasma levels of proinsulin are increased. The increased secretion of insulin by the tumor will also lead to an increase in C peptide secretion (not decreased, choice B) since b cells secrete insulin and C peptide on a one-to-one molar ratio. Given the prolonged hypoglycemia, the amount of glycosylated hemoglobin may also be decreased, although this is not a universal finding. Certainly, an increase in glycosylated hemoglobin would not be expected (choice A). Glucagon secretion is increased by hypoglycemia and its plasma level in a patient with an insulinoma would be expected to be increased compared to normal (not decreased, choice C __________________ 26>A 14-year-old girl with a high fever and sore throat presents to the emergency room. A complete blood count with differential implies the presence of a viral infection. Which of the following best describe s the cells that indicate a viral etiology to her illness? A. They are basophilic with spherical dark-stained nuclei B. They are precursors of osteoclasts and liver Kupffer cells C. They contain a peripheral hyalomere and central granulomere D. They have azurophilic granules and multilobed nuclei E. They remain in the circulation approximately 120 days Anwser -------------------------------------------------------------------------------27>A 35-year-old woman notices a change in the appearance of a mole on her neck. Physical examination reveals that the lesion is an irregular, nodular, superficial mass with a variegated appearance. Biopsy demonstrates a primary malignant tumor. Which of the following factors is most predictive of the patient's long term prognosis? A. Circumference of lesion B. Darkness of lesion C. Degree of color variation D. Depth of lesion E. Sharpness of border between lesion and adjacent skin Anwser --------------------------------------------------------------------------------

28>A patient develops a swollen and tender lymph node in his popliteal fossa. An infected skin lesion in which of the following sites would most likely induce lymphadenopathy in this region? A. Lateral side of the dorsum of the foot B. Lateral side of the thigh C. Medial side of the leg below the knee D. Medial side of the sole of the foot E. Medial side of the thigh Anwser -------------------------------------------------------------------------------29>A 4-year-old child living in a slum is bitten by a rat while sleeping. Two days later, the child develops a rash characterized by discrete erythematous 1-4 mm macules on the extremities and face, most obvious on the palms and soles. Which of the following organisms is the most likely cause of this child's disease? A. Borrelia burgdorferi B. Pseudomonas mallei C. Pseudomonas pseudomallei D. Spirillium minus E. Streptobacillus moniliformis Anwser -------------------------------------------------------------------------------30>A 25-year-old woman develops extensive pruritic wheals following ingestion of seafood to which she was allergic. While these lesions are usually not biopsied, a biopsy would probably show which of the following features? A. Dilated superficial lymphatic channels B. Granular complement and IgG at dermal/epidermal junction C. Microscopic blisters D. Munro microabscesses E. Solar elastosis Anwser -------------------------------------------------------------------------------31>A 43-year-old man presents complaining of pain in the groin. On examination, his physician palpates a bulge in the region of the superficial inguinal ring, which he diagnoses as a direct inguinal hernia. The hernial sac most likely A. is covered by all three layers of the spermatic fascia B. passes medial to the inferior epigastric artery C. passes medial to the lateral border of the rectus abdominis muscle D. passes posterior to the inguinal ligament E. passes through the deep inguinal ring Anwser -------------------------------------------------------------------------------32>A 60-year-old alcoholic smoker abruptly develops high fever, shakes, a severe headache, and muscle pain. He initially has a dry, insignificant cough, but over the next few days he develops marked shortness of breath requiring assisted ventilation. Chest x-ray demonstrates homogeneous radiographic shadowing that initially involves the left lower lobe but continues to spread until both lungs are extensively involved. Culture of bronchoalveolar lavage fluid on buffered charcoal yeast extract (BCYE) demonstrates a coccobacillary pathogen. Which of the following is the most likely causative organism?

A. Legionella pneumophila B. Listeria monocytogenes C. Spirillium minus D. Staphylococcus aureus E. Streptococcus pneumoniae Answer -------------------------------------------------------------------------------33>An experimental serologic test is developed to detect the presence of HIV antibody. Epidemiological analysis reveals the results shown below. People with HIV antibody People without HIV antibody Positive test 100 50 Negative test 20 950 What is the sensitivity of this test? A. 11% B. 67% C. 83% D. 95% E. 98% Anwser -------------------------------------------------------------------------------34>A 30-year-old pregnant woman of Jewish descent presents to a physician with painful oral ulcers. Physical examination demonstrates widespread erosions of her mucous membranes. Close examination reveals a friable mucosa, but no well-defined aphthous ulcers. Biopsy of perilesional mucosa demonstrates acantholysis; direct immunofluorescence demonstrates an intraepidermal band of IgG and C3. Which of the following is the most likely diagnosis? A. Bullous pemphigoid B. Dermatitis herpetiformis C. Herpes simplex I D. Herpes simplex II E. Pemphigus vulgaris Anwser -------------------------------------------------------------------------------35>A 40-year-old woman presents to the emergency department because of hematuria. Laboratory analyses show significant proteinuria, bacteria and white cells in the urine, and a blood urea nitrogen (BUN) of 40 mg/dL with a creatinine of 4.0 mg/dL. Ultrasonography reveals enlarged kidneys, and she is given a provisional diagnosis of polycystic renal disease. Which of the following is likely to be decreased in this patient? A. Creatinine clearance B. Extracellular sodium concentration C. Glucose clearance D. Plasma creatinine levels E. Plasma inulin levels Anwser -------------------------------------------------------------------------------36>During development, the formation of the kidney is induced by which of the following structures?

A. Allantois B. Mesonephric duct C. Mesonephros D. Metanephric duct E. Urogenital ridge Anwser -------------------------------------------------------------------------------37>A premature infant develops progressive difficulty breathing over the first few days of life. Deficient surfactant synthesis by which of the following cell types may have contributed to the baby's respiratory problems? A. Alveolar capillary endothelial cells B. Bronchial mucous cells C. Bronchial respiratory epithelium D. Type I pneumocytes E. Type II pneumocytes Anwser -------------------------------------------------------------------------------38>A surgeon performs an exploratory laparotomy, producing a large incision in the patient's abdomen. Poor blood supply to which of the following is most likely to cause problems during the healing process? A. Adipose tissue B. Aponeuroses C. Loose connective tissue D. Muscle E. Skin Anwser -------------------------------------------------------------------------------39>Which of the following would be expected to be decreased in a patient with chronic hypertension secondary to renal artery stenosis? A. Atrial natriuretic peptide levels B. Blood urea nitrogen (BUN) levels C. Glomerular filtration rate in response to captopril D. Net acid excretion E. Potassium secretion Anwser -------------------------------------------------------------------------------40>A patient, who appears to be female, is found to be 46,XY. The patient's vagina is very shallow, ending in a blind pouch, and there are palpable masses in the labia. The diagnosis of testicular feminization syndrome is made. Which of the following was most likely present during the early fetal life of this individual? A. A streak ovary B. A uterus C. An oviduct D. Depressed levels of testosterone E. MIF (Mullerian inhibitory factor) Anwser -------------------------------------------------------------------------------41>A healthy 42-yearold woman with a history of anxiety attacks sits in the hospital waiting room as her 3year-old daughter undergoes open heart surgery for a septal malformation. The woman experiences a feeling of suffocation and is obviously hyperventilating. She informs her husband

that she feels faint and has blurred vision. Which of the following is most likely to relieve the symptoms caused by hyperventilation? A. Breathing a 10% oxygen/90% nitrogen mixture B. Breathing 100% nitrogen C. Breathing in and out of a plastic bag D. Intravenous administration of bicarbonate E. Lying down Anwser -------------------------------------------------------------------------------42>Which of the following occurs at the darkly stained band indicated by the arrow? A. Acrosome reaction B. Aromatase acts on testosterone C. Capacitation D. Implantation receptors are exhibited E. Meiosis is resumed Anwser -------------------------------------------------------------------------------43>A 38-year-old AIDS patient presents to his physician's office in Kansas City, Missouri, complaining of fever for the past week and an increasing headache. He also states that sunlight hurts his eyes and that he has been feeling nauseated and weak. His past medical history is significant for Pneumocystis pneumonia and a total CD4 count of 89. Current medications are trimethoprim/sulfa and indinavir. Cerebrospinal fluid (CSF) reveals 4 WBC, and budding encapsulated yeast forms grow on Sabouraud's agar. Which of the following is an accurate description of the morphology of the infectiou s form of the organism responsible for the man's illness? A. Broad-based, budding yeasts B. Budding yeasts in a "pilot's wheel" arrangement C. Cylindrical arthroconidia D. Encapsulated budding yeasts E. Filamentous molds F. Septate hyphae with microconidia and macroconidia Anwser -------------------------------------------------------------------------------44>To which of the following diseases is pyruvate kinase deficiency most similar clinically? A. a-thalassemia B. b-thalassemia C. Glucose-6-phosphate dehydrogenase deficiency D. Hereditary spherocytosis E. Iron deficiency anemia Anwser -------------------------------------------------------------------------------45>A 60-year-old male with angina comes to the emergency room with severe chest pain unresponsive to sublingual nitroglycerin. An EKG shows ST segment elevation in the anterolateral leads, and thrombolytic therapy is initiated. If streptokinase is given to this patient, it may produce thrombolysis after binding to which of the following proteins? A. Antithrombin III

B. Fibrin C. Plasminogen D. Protein C E. Thrombomodulin Anwser -------------------------------------------------------------------------------46>A child is brought to the pediatrician because of perianal itching, which has been disturbing the child's sleep. Physical examination reveals scaly skin in the perianal region. The physician instructs the parents to place transparent tape on the perianal region the next morning, and then retur n to the office. In the office, the tape is transferred to a microscope slide, and low-power examination of the tape reveals oval eggs that are flattened along one side. The microorganism most likely responsible for the child's symptoms is A. Ancylostoma duodenale B. Ascaris lumbricoides C. Enterobius vermicularis D. Necator americanus E. Trichuris trichiura Anwser -------------------------------------------------------------------------------47>A healthy, 37-yearold, recently divorced woman loses her job at the auto factory. She picks up her three young children from the factory day care center and gets into an automobile accident on the way home. Her 5-year-old son, who was not wearing a seat belt, sustains a severe head injury. The woman was not hurt in the accident, but is hyperventilating as she sits in the waiting room at the hospital. She complains of feeling faint and has blurred vision. Which of the following is decreased in this woman? A. Arterial oxygen content B. Arterial oxygen tension (PO2) C. Arterial pH D. Cerebral blood flow E. Cerebrovascular resistance Anwser -------------------------------------------------------------------------------48>A 66-year-old man with urinary retention secondary to prostatic hyperplasia develops a spiking fever and tachypnea. Physical exam reveals intercostal muscle retractions and bilateral inspiratory crackles. A chest x-ray exhibits bilateral interstitial and alveolar infiltrates. Arterial blood gases demonstrate severe hypoxemia. Blood cultures would most likely reveal A. gramnegative diplococci B. gram-negative rods C. gram-positive cocci D. gram-positive diplococci E. gram-positive rods Anwser -------------------------------------------------------------------------------49>A patient arrives in the emergency room after having attempted suicide by lacerating his wrist. No major artery was damaged, but the nerve that is immediately lateral to the flexor digitorum superficialis tendon is cut. Which of the following actions will no longer be possible?

A. Abduction of the second digit B. Adduction of the second digit C. Adduction of the thumb D. Flexion at the interphalangeal joint of the thumb. E. Opposition of the thumb Anwser -------------------------------------------------------------------------------50>A 48-yearold female is being treated for breast carcinoma. Over the past few days, she has been complaining of dysuria and frequency. Laboratory examination revealed the presence of microscopic hematuria. The next day the patient developed gross hematuria. Which of the following agents most likely caused the development of these signs and symptoms? A. Cyclophosphamide B. Mitomycin C. Paclitaxel D. Tamoxifen E. Vincristine Answers Now you can see what's hidden26>The correct answer is A. This question is really asking you two things. First, it tests whether you know that lymphocytosis is associated with viral infection. Second, it tests your knowledge of lymphocytes' histological appearance. Note that these cells are generally small and normally constitute 25%-33% of leukocytes. Two types of lymphocytes have been distinguished: T cells (involved in cell-mediated immunity) and B cells (involved in humoral immunity). Monocytes are precursors of osteoclasts and liver Kupffer cells (choice B) and also give rise to tissue macrophages and alveolar macrophages. Platelets contain a peripheral hyalomere and central granulomere (choice C). Neutrophils have azurophilic granules and multilobed nuclei (choice D). They increase in number in response to bacterial infection. Erythrocytes remain in the circulation for about 120 days (choice E). --------------------------------------------------------------------------------27>The correct answer is D. The lesion is a malignant melanoma. Melanomas can develop either de novo or in an existing mole. Sunlight exposure is a significant risk factor and fair-skinned persons are at increased risk of developing melanoma. The most significant factor for long term prognosis is the depth of the lesion, since the superficial dermis lies about 1 mm under the skin surface, and penetration to this depth is associated with a much higher incidence of metastasis than is seen with a more superficial location. The circumference of the lesion (choice A) is much less important than depth, since one form of melanoma (superficial spreading) can still have good prognosis despite large size, if it has not extended to the depth of the superficial dermal lymphatic bed. The darkness (choice B) or degree of variation in color (choice C) do not have prognostic significance once melanoma is diagnosed. Irregularity, or fuzziness at the border (choice E) of a mole-like lesion is a good clue to potential malignancy, but does not affect prognosis once a melanoma is diagnosed. --------------------------------------------------------------------------------28>The correct answer is A. Most of the venous drainage of the skin of the lower extremity is to the long saphenous vein, the accompanying lymphatics of which drain into the superficial group of the inguinal lymph nodes. However, the skin drained by the short saphenous vein, including that of the lateral aspect of the dorsum of the foot, is an exception to this rule. Lymphatic fluid from this area drains into lymphatics accompanying the short saphenous vein, then drains into lymph nodes behind the knee in the popliteal fossa. The lateral side of the thigh (choice B), the medial side of the leg below the knee (choice C), the medial side of the sole of the foot (choice D), and the medial side of the thigh (choice E) all drain to the superficial inguinal nodes. --------------------------------------------------------------------------------29>The correct

answer is E. The child has "rat-bite fever." This occurs in two forms with somewhat similar clinical manifestations. The form this child has is the Haverhill fever form, caused by Streptobacillus moniliformis and characterized by a short (often 1-3 days) incubation period. The Haverhill form is more common in the United States than the Sodoku form, which is caused by Spirillium minus, has a 1-4 week incubation period, and is most prevalent in Japan. One of the problems with diagnosing these diseases is that the victims are usually young children, and the bite site may be inapparent by the time the disease becomes severe enough for the child to be taken to a doctor. Borrelia burgdorferi (choice A) causes Lyme disease, which is characterized by an expanding erythematous rash, arthralgias, and eventual nervous system involvement. Pseudomonas mallei (choice B) causes glanders, which generally affects horses or humans in close contact with equines. Pseudomonas pseudomallei (choice C) causes melioidosis, a rare pulmonary disease found mostly in Southeast Asia. Spirillium minus (choice D) causes the Sodoku form of rat-bite fever. --------------------------------------------------------------------------------30>The correct answer is A. Urticaria (hives) are pruritic wheals that form after mast cells degranulate and trigger localized dermal edema with dilated superficial lymphatic channels. The mast cell degranulation is sometimes, but not always, triggered by IgEantigen interactions. Granular complement and IgG deposition at the dermal/epidermal junction (choice B) is a characteristic of systemic lupus erythematosus. Microscopic blisters (choice C) are a characteristic of dermatitis herpetiformis. Munro abscesses (choice D) are a characteristic of psoriasis. Solar elastosis (choice E) is found in actinic keratoses. --------------------------------------------------------------------------------31>The correct answer is B. Direct inguinal hernias enter the inguinal canal by tearing through the posterior wall of that structure. The typical location for this type of hernia is through the inguinal triangle, bounded laterally by the inferior epigastric artery, medially by the lateral border of the rectus abdominis, and inferiorly by the inguinal ligament. Direct inguinal hernias pass medial to the inferior epigastric artery, whereas indirect inguinal hernias pass lateral to the inferior epigastric artery because the deep inguinal ring is lateral to the artery. Indirect inguinal hernias are covered by all three layers of the spermatic fascia (choice A). Direct inguinal hernias are covered by fewer than all three layers because the direct inguinal hernia tears through one or more layers of fascia as it emerges though the abdominal wall. The lateral border of the rectus abdominis muscle (choice C) forms the medial border of the inguinal triangle. All inguinal hernias pass lateral to the rectus abdominis. Femoral hernias pass posterior to the inguinal ligament (choice D). Inguinal hernias emerge through the superficial inguinal ring, which is superior to the inguinal ligament. Inguinal hernias that descend below the inguinal ligament pass anterior to the ligament. Indirect inguinal hernias pass through the deep inguinal ring (choice E); direct inguinal hernias do not. Both types of inguinal hernias pass through the superficial inguinal ring. -------------------------------------------------------------------------------32>The correct answer is A. The patient has a severe, potentially fatal, pneumonia with prominent systemic symptoms. Culture on BCYE is the specific clue that the organism is Legionella pneumophila. The disease is respiratory Legionellosis, also known as Legionnaire's disease, because the disease was first described when it occurred in epidemic form following an American Legion convention at a Philadelphia hotel. Patients tend to be older (40-70 years old) and may have risk factors including cigarette use, alcoholism,

diabetes, chronic illness, or immunosuppressive therapy. Listeria monocytogenes (choice B) causes listeriosis and is not a notable cause of pneumonia. Spirillium minus (choice C) is a cause of rat-bite fever and is not a notable cause of pneumonia. Staphylococcus aureus (choice D) can cause pneumonia, but is easily cultured on routine media.

Streptococcus pneumoniae (choice E) can cause pneumonia, but is easily cultured on routine media. --------------------------------------------------------------------------------------------------------------------------------------------------------------33The correct answer is C. Sensitivity is defined as the ability of a test to detect the presence of a disease in those who truly have the disease. It is calculated as the number of people with a disease who test positive (true positive) divided by the total number of people who have the disease (true positive + false negative). In this case, sensitivity equals the number of people with HIV antibody who test positive (100) divided by the total number of people who have HIV antibody (120). This yields 100/120 = 83% (not a very sensitive test). 11% (choice A) corresponds to the prevalence of the disease in the tested population, which in this case equals the total number of people with HIV antibody (true positive + false negative = 100 + 20 = 120) divided by the total number of people tested (100 + 20 + 50 + 950 = 1120). This yields 120/1120 = 11%. 67% (choice B) corresponds to the positive predictive value of the test, which equals the number of people with the disease who test positive (true positives = 100) divided by the total number of people testing positive (all positives = 50). This yields 100/150 = 67%. 95% (choice D) corresponds to the specificity of the test, which equals the number of people without HIV antibody who test negative (950) divided by the total number of people without HIV antibody (10 __________________ Part 4/6-50 mcqs <1>A 72-year-old male is noted as having a 9-pound weight loss over the past few weeks. His past medical history is significant for oat cell carcinoma of the lung, without known metastases, for which he is currently undergoing treatment. The patient states that even though his wife is preparing his favorite meals, he is not hungry. Which of the following would be the best treatment option to improve his eating habits? A. Amitriptyline B. Megestrol acetate C. Methotrexate D. Neostigmine E. Prochlorperazine Answer -------------------------------------------------------------------------------<2>A 45-yearold homeless man has a chronic cough, a cavitary lesion of the lung, and is sputum positive for acid-fast bacilli. Which of the following is the principle form of defense by which the patient's body fights this infection? A. Antibody-mediated phagocytosis B. Cell-mediated immunity C. IgA-mediated hypersensitivity D. IgE-mediated hypersensitivity E. Neutrophil ingestion of bacteria Answer --------------------------------------------------------------------------------

<3>A researcher is examining the distribution of an ion channel protein in the kidney. She incubates slices of kidney tissue in a dilute solution of a specific antibody directed against the protein, then uses the immunoperoxidase method to localize the ion channel proteins. She notes the presence of brown pigment in a population of epithelial cells, which on closer examination, have a brush border. The researcher concludes that the protein is probably present in cells of the A. collecting duct B. deep portion of loop of Henle C. distal convoluted tubule D. glomerulus E. proximal convoluted tubule Answer -------------------------------------------------------------------------------<4>A 38-yearold pregnant woman with a past medical history significant for chronic hypertension presents with a blood pressure of 158/105 mm Hg. Which of the following antihypertensive agents would be most suitable for initial therapy in this patient? A. Bumetanide B. Fosinopril C. Hydrochlorothiazide D. Methyldopa E. Valsartan Answer -------------------------------------------------------------------------------<5>A 3 yearold boy is brought to a physician because the mother notices tha t the child is engaging in less active play and tires easily. During physical examination, the pediatrician notices that the child's thighs are larger than normal for age and that the child cannot stand up without using his arms to help. Further studies demonstrate a defective dystrophin gene in the boy. Which of the following people in the child's family is most likely to also have this disease? A. Father B. Father's brother C. Mother D. Mother's brother E. Sister Answer -------------------------------------------------------------------------------<6> The leukocyte pictured above stains intensely with acidic dyes such as eosin. Which of the following substances is contained in the crystalline core of the granule at the arrow? A. Lactoferrin B. Major basic protein C. Myeloperoxidase D. Histamine

E. Tartrate-resistant acid phosphatase Answer -------------------------------------------------------------------------------<7>A 7-year-old boy is brought to a physician because of a nearly confluent, fine, erythematous, macular rash that is most pronounced on his trunk. He has had a mild fever for 36 hours, but does not appear very sick, and is happily playing when the pediatrician enters the examining room. Physical examination demonstrates a reddened throat with tonsillar exudates, enlarged cervical nodes including the occipital node, and questionable splenomegaly. The mother says that the boy has not been coughing, and no Koplik spots are noted. Which of the following is the most likely diagnosis? A. Bullous pemphigoid B. Dermatitis herpetiformis C. Herpes simplex D. Measles E. Rubella Answer -------------------------------------------------------------------------------<8>A child who has had abnormal development of the membranous bones has a broad skull with associated facial and dental anomalies. Which other bones are most likely to also be affected? A. clavicles B. Femurs C. Metatarsals D. Phalanges E. Tibias Answer -------------------------------------------------------------------------------<9>A 33-year-old woman presents with fever, vomiting, severe irritative voiding symptoms, and pronounced costovertebral angle tenderness. Laboratory evaluation reveals leukocytosis with a left shift; blood cultures indicate bacteremia. Urinalysis shows pyuria, mild hematuria, and gram-negative bacteria. Which of the following drugs would best treat this patient's infection? A. Ampicillin and gentamicin B. Erythromycin C. Gentamicin and vancomycin D. Phenazopyridine and nitrofurantoin E. Tetracycline Answer -------------------------------------------------------------------------------<10>A surgical pathology specimen from a 24-year-old woman seen at a reproductive medicine clinic demonstrates a ciliated columnar epithelium. From which of the following locations in the female genital tract was the

biopsy obtained? A. Cervix B. Endometrium C. Fallopian tube D. Ovary E. Vagina Answer -------------------------------------------------------------------------------<11>In a genotypic male, the testes fail to develop, and do not secrete testosterone or Mllerian regression factor. Which of the following best describes the in utero reproductive system development of this individual ? A. Both male- and female-type internal reproductive tracts and male-type external genitalia B. Female-type internal reproductive tract and female-type external genitalia C. Female-type internal reproductive tract and male-type external genitalia D. Male-type internal reproductive tract and female-type external genitalia E. Male-type internal reproductive tract and male-type external genitalia Answer -------------------------------------------------------------------------------<12>A pulmonologist is testing a patient's lung volumes and capacities using simple spirometry. Which of the following lung volumes or capacities cannot be measured directly using this technique? A. Expiratory reserve volume B. Functional residual capacity C. Inspiratory reserve volume D. Tidal volume E. Vital capacity Answer -------------------------------------------------------------------------------<13>Contraction of which of the following muscles contributes most to the backward movement of the lower jaw during the process of mastication? A. Digastric B. Lateral pterygoid C. Medial pterygoid D. Mylohyoid E. Temporalis Answer -------------------------------------------------------------------------------<14>A 28-year-old female presents to the doctor complaining of syncopal episodes that last a few minutes. She is not taking any medications and has no previous medical history. EEG and EKG studies are performed and are unremarkable. An echocardiogram shows a single ballshaped mass dangling in the left atrium near the mitral valve. The most likely diagnosis is A. angiosarcoma B. mesothelioma C. myxoma D. rhabdomyoma E. rhabdomyosarcoma

Answer -------------------------------------------------------------------------------<15>A patient who has been exhibiting various endocrine abnormalities has an MRI scan of the head. This scan reveals a small tumor of the pituitary gland. If this tumor expands laterally, which of the following nerves will most likely be affected first? A. Abducens nerve B. Oculomotor nerve C. Optic nerve D. Trigeminal nerve E. Trochlear nerve Answer -------------------------------------------------------------------------------<16>A liver biopsy from a 54-year-old man shows many Mallory bodies. This finding is most suggestive of which of the following diseases? A. Alcohol abuse B. Alpha1-antitrypsin deficiency C. Hepatitis A D. Hepatitis B E. Wilson's disease Answer -------------------------------------------------------------------------------<17>A 27-year-old man has been arrested by the police for hitting, cursing at, and verbally berating his wife of 8 years. The wife tells the police he also regularly physically whips his 7-year-old son with a leather belt and often strikes the boy with his hand. When asked why he does this, he responds that this is "how my father treated me, it's how men should act." This represents which of the following types of learning? A. Classical conditioning B. Cognitive learning C. Imprinting D. Operant conditioning E. Social learning Answer -------------------------------------------------------------------------------<18>A 64-year-old man presents to his doctor with aching, burning pain afte r meals. He had been selfmedicating for several months with antacids, but he found this to be increasingly ineffective. His physician decides to take him off the antacids and instead places him on a combination of ranitidine and sucralfate. Why is this combination a bad idea? A. Ranitidine increases the toxicity of sucralfate B. Ranitidine inhibits the action of sucralfate C. Sucralfate and ranitidine coprecipitate D. Sucralfate increases the toxicity of ranitidine E. Sucralfate inhibits the action of ranitidine Answer --------------------------------------------------------------------------------

<19>A 42-year-old female presents with a recent onset of fatigue, malaise, constipation, and a 12-pound weight gain. On examination, her thyroid is firm and enlarged. What laboratory test is most likely to confirm the expected diagnosis? A. Antithyroid antibodies B. Serum thyroid-stimulating hormone (TSH) measurement C. Serum thyroxine (T4) measurement D. Serum triiodothyronine (T3) measurement E. T3 resin uptake Answer -------------------------------------------------------------------------------<20>A 70-year-old man with a history of atrial fibrillation is started on an oral anticoagulant. His prothrombin time is monitored on a regular basis. A few months into his therapy, he begins treatment for a duodenal ulcer and he develops symptoms of a bleeding diathesis. Which of the following ulcer medications is most likely responsible for this change in his hemostatic status? A. Cimetidine B. Famotidine C. Misoprostol D. Omeprazole E. Ranitidine Answer -------------------------------------------------------------------------------<21>An elderly woman with a history of multiple oral ulcers presents with flaccid bullae on her scalp, face, and trunk. Manual pressure on the skin produces separation of the upper layer of the epidermis, followed by eventual sloughing of the skin. The patient has been in relatively good health until recently, and denies taking any medications. A biopsy of one of the skin lesions reveals separation of epithelial cells above the basal layer. Autoantibodies to which of the following components would most likely be found in this patient? A. Epidermal basement membrane proteins B. Glycoprotein IIb/IIIa C. Intercellular junctions of epidermal cells D. Intrinsic factor E. Type IV collagen Answer -------------------------------------------------------------------------------<22>A 35-year-old woman notices a change in the appearance of a mole on her neck. Physical examination reveals that the lesion is an irregular, nodular, superficial mass with a variegated appearance. Biopsy demonstrates a primary malignant tumor. Which of the following factors is most predictive of the patient's long term prognosis? A. Circumference of lesion B. Darkness of lesion C. Degree of color variation D. Depth of lesion E. Sharpness of border between lesion and adjacent skin Answer

-------------------------------------------------------------------------------<23>During embryological development, hematopoiesis occurs in different organs at different times. Which of the following are the correct organs, in the correct sequence, at which hematopoiesis occurs embryologically? A. Amnion, yolk sac, placenta, bone marrow B. Placenta, liver and spleen, yolk sac, bone marrow C. Placenta, spleen and lymphatic organs, bone marrow D. Yolk sac, bone marrow, liver and spleen E. Yolk sac, liver, spleen and lymphatic organs, bone marrow Answer -------------------------------------------------------------------------------<24>A 6-month-old boy is brought to the pediatrician by his parents, who are first cousins. This is their first child. Physical examination reveals a small, thin, lethargic infant with slightly misshapen long bones. His features are somewhat coarse. Joint movements are restricted, his corneas are clouded, and his gums are underdeveloped. His liver is not enlarged. Serum levels of acid hydrolases are found to be elevated. The child most likely has a defect in which of the following metabolic activities? A. Degradation of dermatan sulfate and heparan sulfate B. Degradation of gangliosides C. Degradation of glycogen D. Degradation of sphingomyelin E. Phosphorylation of mannose moieties F. Phosphorylation of tyrosine moieties Answer -------------------------------------------------------------------------------<25>A particular association determines membership on the basis of members' IQ scores. Only those persons who have documented IQ scores at least 2 standard deviations above the mean on the Wechsler Adult Intelligence Scale (WAIS) are eligible for admission. Of a group of 200 people randomly selected from the population at large, how many would be eligible for membership to this society? A. 1 B. 2 C. 3 D. 4 E. 5 Answers Now you can see what's hidden1>The correct answer is B. One of the most common side effects of any antineoplastic therapy is weight loss secondary to decreased appetite and/or nausea and vomiting. Furthermore, weight loss due to decreased food intake tends to occur more frequently in elderly patients receiving antineoplastic therapy. One medication that has consistently helped to increase appetite in such patients is megestrol acetate. This agent is a progestational hormone with antineoplastic properties used in the treatment of advanced carcinoma of the breast and endometrium. Megestrol, when given in relatively high doses, can substantially increase the appetite in most individuals, even those with advanced cancer. Amitriptyline (choice A) is a tricyclic antidepressant used in the treatment of depression. There is nothing mentioned in the case study to suggest that the patient is clinically depressed; hence, this agent would provide no benefit.

Methotrexate (choice C) is an antimetabolite and folic acid antagonist commonly used in various neoplastic disorders and in the treatment of rheumatoid arthritis. Since nausea, vomiting, and ulcerative stomatitis are common side effects of this medication, its usage in this patient would not be recommended. Neostigmine (choice D) is a carbamylating acetylcholinesterase inhibitor that would not increase appetite. Prochlorperazine (choice E) is a phenothiazine derivative used primarily to control severe nausea and vomiting. This patient is not experiencing nausea. Furthermore, this agent does not possess appetite-stimulating properties. -------------------------------------------------------------------------------2>The correct answer is B. The principle host defense in mycobacterial infections (such as this patient's tuberculosis) is cell-mediated immunity, which causes formation of granulomas. Unfortunately, in tuberculosis and in many other infectious diseases characterized by granuloma formation, the organisms may persist intracellularly for years in the granulomas, only to be a source of activation of the infection up to decades later. While antibody-mediated phagocytosis (choice A) is a major host defense against many bacteria, it is not the principle defense against Mycobacteria. IgA-mediated hypersensitivity (choice C) is not involved in the body's defense against Mycobacteria. IgE-mediated hypersensitivity (choice D) is not involved in the body's defense against Mycobacteria. It is important in allergic reactions. Neutrophil ingestion of bacteria (choice E) is a major host defense against bacteria, but is not the principle defense against Mycobacteria. -------------------------------------------------------------------------------3>The correct answer is E. The immunoperoxidase method uses horseradish peroxidase to produce a visible pigment when a specific antibody binds to antigenic sites in the tissue. Binding of the antibody to the epithelial cells indicates that the protein in question is being expressed by those cells. The positive immunoperoxidase results obtained here suggests that the channel protein is expressed in proximal convoluted tubular epithelium, since this is the only site in the kidney at which the epithelial cells have a "brush border." The brush border is made of microvilli, which enhance the proximal tubules' ability to reabsorb plasma constituents filtered at the glomeruli. -------------------------------------------------------------------------------4>The correct answer is D. Pregnant women with chronic hypertension "require" antihypertensive therapy when the diastolic pressure is greater than 100 mm Hg; however, some clinicians may decide to treat patients with diastolic blood pressures less than 100 mm Hg. For the initiation of therapy, methyldopa is still considered to be the agent of choice. Methyldopa is converted intraneuronally to a-methylnorepinephrine, an alpha-2 adrenergic agonist, which is subsequently released. Release of amethylnorepinephrine in the medulla leads to a decrease in sympathetic outflow, thus lowering blood pressure. Methyldopa has been safely used in the treatment of hypertension during pregnancy; this agent is not associated with the development of teratogenic or other fetal abnormalities. Diuretics, such as bumetanide (choice A) and hydrochlorothiazide (choice C), are often avoided since these agents can produce hypovolemia, leading to reduced uterine blood flow. Although these agents can be used during pregnancy, methyldopa and hydralazine are the drugs of choice for hypertension during pregnancy. Fosinopril (choice B) is an angiotensin-converting enzyme (ACE) inhibitor that should not be administered to pregnant women, especially in the second or third trimesters. These agents have been associated with severe fetal and neonatal injury, such as hypotension, neonatal skull hypoplasia, anuria, renal failure, and

death. Along the same lines, the use of the angiotensin II receptor antagonists, such as valsartan (choice E), is not recommended since these agents cause fetal complications similar to the ACE inhibitors. -------------------------------------------------------------------------------5>The correct answer is D. The disease is Duchenne muscular dystrophy, an X-linked recessive muscular disease usually caused by a deletion involving the dystrophin gene.

This defect produces accelerated muscle breakdown leading initially to proximal muscle weakness, then later to generalized weakness that typically begins before age 5. A feature of X-linked recessive diseases is that carrier mothers pass the disease to half their sons; affected fathers can have carrier daughters but not affected sons. Since the mother is presumably normal (because the disease is X-linked), she must be a carrier to have an affected son, and the grandmother must also be a carrier, therefore the mother's brother (maternal uncle) may also have the disease. The father's (choice A) side of the family, including the father's brother (choice B), most likely does not carry the defective gene (since they themselves would be affected, and furthermore since the father cannot pass the gene on to a son). It would be extremely unlikely for a carrier female to marry an affected male (and the question does not mention any similar symptoms in the father). The mother (choice C) and possibly the sister (choice E) are carriers of, but not affected by, the defective gene. -------------------------------------------------------------------------------6>The correct answer is B. The cell pictured is an eosinophil, a member of the granulocytic lineage of white blood cells. The crystalline core of the granule contains a protein called the major basic protein, which appears to function in the destruction of parasites. Major basic protein also has deleterious effects on epithelial cells in patients with asthmatic reactions. The light component around the dense crystalline core contains products such as histaminase, arylsulfatase, and other enzymes. Lactoferrin (choice A) is found in the specific granules of the neutrophil. It inhibits the growth of bacteria by interfering with iron metabolism. Myeloperoxidase (choice C) is found in the azurophilic (large) granule of the neutrophil. This enzyme is also destructive to bacteria, destroying their cell walls. Histamine (choice D) is produced by the basophil and the mast cell. The histaminase of the eosinophil regulates the inflammatory reaction of these two cell types. Tartrate-resistant acid phosphatase (choice E) is a marker for hairy cell leukemia, a neoplasm of the B lymphocyte line. -------------------------------------------------------------------------------7>The correct answer is E. This presentation (fine, nonblotchy, truncal rash in a not-veryill child) is characteristic of rubella, or German measles. IgM specific for rubella can often be detected in serum within 1-2 days of developing the rash. The principal significance of this disease is that it can cause a devastating congenital infection characterized by ocular problems (cataracts, retinopathy, microphthalmos, glaucoma), cardiovascular problems (patent ductus arteriosus, ventricular septal defect, pulmonary stenosis), deafness, thrombocytopenic purpura, hepatosplenomegaly, CNS problems, and bony lesions. Bullous pemphigoid (choice A) produces large, tense blisters. Dermatitis herpetiformis (choice B) causes recurrent crops of small vesicles or papules. Herpes simplex (choice C) is characterized by crops of vesicles on oral or genital sites. Measles (choice D) causes a blotchy, maculopapular erythematous rash that begins on the face and spreads downward. Patients with measles are usually much sicker than those with German measles, and Koplik spots maybe seen on the buccal mucosa. -------------------------------------------------------------------------------8>The correct answer is A. In a syndrome called cleidocranial dysostosis, absence of part of the clavicles accompanies a broad skull, and facial and dental anomalies. Note that you could also have answered this question by noting that of the bones listed, only the clavicles form by intramembranous ossification. The femurs (choice B), metatarsals (choice C), phalanges (choice D), and tibias (choice E) are cartilaginous (formed by endochondral ossification)

rather than membranous bones. -------------------------------------------------------------------------------9>The correct answer is A. Acute pyelonephritis is an infectious disease involving the kidney parenchyma and the renal pelvis. Gram-negative bacteria, such as Escherichia coli, Proteus, Klebsiella, and Enterobacter, are the most common causative organisms in acute pyelonephritis. Laboratory evaluation will often reveal leukocytosis with a left shift, and urinalysis typically shows pyuria, varying degrees of hematuria, and white cell casts.

Since bacteremia is present, the patient should be hospitalized and empirically started on IV ampicillin and gentamicin. This regimen may be need to be changed, however, once the sensitivity results are available. Erythromycin (choice B) and tetracycline (choice E) are both bacteriostatic antibiotics and would not be recommended in a patient with a severe infection, such as acute pyelonephritis with bacteremia. Vancomycin (choice C) is primarily used in the treatment of severe gram-positive infections. Phenazopyridine (choice D) is a urinary analgesic, and nitrofurantoin (choice D) is a urinary tract antiinfective. Although nitrofurantoin is indicated for the treatment of "mild" cases of pyelonephritis, as well as cystitis, this patient's condition is severe and should be treated with appropriate antibiotics. -------------------------------------------------------------------------------10>The correct answer is C. The fallopian tube is the only structure in the female genital tract with a ciliated columnar epithelium; the beating of the cilia helps move the egg into the uterus. This fact is also sometimes clinically helpful since dilated and deformed fallopian tubes can be microscopically distinguished from cystic ovarian tumors by the presence of the cilia. The cervix (choice A) and vagina (choice E) are lined by squamous epithelium. The endometrium (choice B) is lined by columnar epithelium (although a few ciliated cells may be present). The covering of the ovary (choice D) is cuboidal epithelium, and cysts within the ovary can be lined by cuboidal or non-ciliated columnar epithelium. -------------------------------------------------------------------------------11>The correct answer is B. The description is that of gonadal dysgenesis. In the absence of testosterone, the Wolffian ducts will regress and fail to differentiate into normal male internal reproductive tracts. In the absence of Mllerian regression factor, the Mllerian ducts will automatically differentiate into oviducts and a uterus. Differentiation of the male external genitals is dependent on adequate dihydrotestosterone (via an action of 5 a-reductase on testosterone). In the absence of testosterone, female-type external genitalia will develop. Selective dysgenesis of the Sertoli cells could produce the situation described in choice A. Normal Leydig cells would secrete testosterone and produce normal male-type internal and external tracts. However, the absence of Mllerian regression factor, which is secreted by the Sertoli cells, would allow formation of female-type internal structures as well. Female-type internal reproductive tract and male-type external genitalia (choice C) would not be likely to occur under any circumstances. The situation described in choice D could occur with 5 a-reductase deficiency. Normal male-type internal tracts can form because there is no requirement for dihydrotestosterone. Mllerian regression factor will prevent differentiation of femaletype internal tracts. Since differentiation of the normal male external genitals requires dihydrotestosterone, 5 a-reductase deficiency will lead to feminization. The situation described in choice E is normal, and would not occur in the individual described who has testicular dysgenesis. -------------------------------------------------------------------------------12>The correct answer is B. The functional residual capacity is the amount of air left in the lungs after a normal expiration. Because this volume cannot be expired in its entirety, it cannot be measured by spirometry. Essentially, lung volume that contains the residual volume, which is the amount of air remaining after maximal expiration (e.g., functional residual capacity and total lung capacity), cannot be measured by spirometry. These volumes can be determined using helium dilution techniques coupled with spirometry or body plethysmography. The expiratory reserve volume (choice A) is the volume of air that can

be expired after expiration of a tidal volume. The inspiratory reserve volume (choice C) is the volume of air that can be inspired after inspiration of a tidal volume.

Tidal volume (choice D) is the amount of air inspired or expired with each normal breath. Vital capacity (choice E) is the volume of air expired after a maximal inspiration -------------------------------------------------------------------------------13>The correct answer is E. Mastication is a complex process involving alternating elevation, depression, forward movement, and backward movement of the lower jaw. The backward movement step is accomplished by the posterior fibers of the temporalis muscle. The digastric (choice A) helps to depress the lower jaw during chewing. The lateral pterygoid (choice B) helps to move the lower jaw forward during chewing. The medial pterygoid (choice C) helps to elevate the lower jaw during chewing. The mylohyoid (choice D) helps to depress the lower jaw during chewing. -------------------------------------------------------------------------------14>The correct answer is C. The vignette illustrates a typical presentation for a tumor of the heart. Primary cardiac tumors are rare and usually require an intensive work-up to pinpoint the diagnosis. 75% of primary cardiac tumors are benign and among these, myxoma is the most common. The tumors are usually single; the most common location is the left atrium. They may cause syncopal episodes or even shock and death due to obstruction by a "ball valve" mechanism. Angiosarcoma (choice A) is a malignant tumor of vascular origin that can occur as a primary cardiac tumor. It is the most common malignant primary cardiac tumor, but it is still very rare. Angiosarcoma usually affects the right side of the heart. Mesothelioma (choice B) is a benign tumor of mesothelial origin that can rarely present as a primary cardiac tumor. It is usually a small intramyocardial tumor that presents with disturbances of the conduction system of the heart. Rhabdomyoma (choice D) is a benign tumor of muscle origin. It can occur as a primary cardiac tumor, typically in infants and children, in whom it may be associated with tuberous sclerosis. It usually occurs in the ventricles. Rhabdomyosarcoma (choice E) is a malignant neoplasm that can also occur as a rare primary cardiac tumor. It is of muscle origin and usually affects the right heart. -------------------------------------------------------------------------------15>The correct answer is A. The pituitary gland is located in the pituitary fossa within the skull. The floor of this fossa is formed by the sella turcica. The lateral walls of the fossa are formed by the cavernous sinuses. The abducens nerve passes through the cavernous sinus along with the internal carotid artery. As the tumor expands laterally, the first nerve that will be encountered is the abducens nerve, producing a lateral rectus palsy. The oculomotor nerve (choice B) lies in the lateral wall of the cavernous sinus. It is further from the pituitary gland than is the abducens nerve. The optic nerve (choice C) is anterosuperior to the pituitary gland. Upward expansion of the tumor may compress the optic chiasm. The trigeminal nerve (choice D) is found posterior to the cavernous sinus. Two of its three divisions (ophthalmic and maxillary divisions) pass through the lateral wall of the cavernous sinus and are further from the pituitary gland than is the abducens nerve. The trochlear nerve (choice E) is also in the lateral wall of the cavernous sinus, and would be affected later if the tumor continued to expand. -------------------------------------------------------------------------------16>The correct answer is A. Mallory bodies are eosinophilic cytoplasmic inclusions ("alcoholic hyaline") that are found in the largest numbers in alcoholic hepatitis. They were originally considered to be pathognomic of alcohol abuse, but have since been found (in much smaller numbers) in many other liver conditions. Alpha1-antitrypsin deficiency (choice B) involvement of the liver is characterized by periodic acid Schiff (PAS)-positive cytoplasmic granules in

hepatocytes. Hepatitis A (choice C) and hepatitis B (choice D) infections are definitively established with serologic markers. In Wilson's disease (choice E), there is excess copper deposition in the liver -------------------------------------------------------------------------------17>The correct answer is E. In social learning, also known as modeling, behavior is acquired by watching other persons and assimilating their actions into the behavioral

repertoire. There is no verbal or cognitive process (choice B) that is involved, no reinforcement (as in operant conditioning; choice D), no pairing of stimuli to get stimulus substitution (as in classical conditioning; choice A), nor any early-life bonding or imprinting (choice C) involved in this type of process. Because behaviors such as spousal abuse, child abuse, and elder abuse are all based on observing and incorporating behaviors from significant others, the person displaying the behaviors does not realize the behaviors are inappropriate and is typically very resistant to change. The fact that the learning is nonverbal and not dependent upon reinforcement contributes to the resistance to change. -------------------------------------------------------------------------------18>The correct answer is B. Sucralfate is a promising drug that is not presently in widespread use because it is incompatible with H2 antagonists such as cimetidine, ranitidine, famotidine and nizatidine. Sucralfate is aluminum sucrose sulfate, a sulfated disaccharide, which polymerizes and binds to ulcerated tissue. It forms a protective coating against acid, pepsin and bile, giving the tissue a chance to heal. Unfortunately, a low gastric pH is required for polymerization, meaning that sucralfate is incompatible with drugs that reduce gastric acidity, such as H2 blockers and antacids. The moral of the story is that you cannot assume that two medications that are individually helpful in a medical condition will be synergistic. Learning the mechanisms by which the drugs work will help you spot potential interactions and earn you points on the USMLE. -------------------------------------------------------------------------------19>The correct answer is B. The patient's presentation is consistent with hypothyroidism. Serum thyroidstimulating hormone (TSH) measurement (choice B) is most likely to confirm the empiric diagnosis. TSH levels usually rise above normal before serum thyroxine (T4; choice C) and serum triiodothyronine (T3; choice D) levels do, even in mild cases of hypothyroidism. Therefore, TSH measurement would be the most accurate test to determine the presence of hypothyroidism regardless of the severity. A high titer of antithyroid antibodies (choice A) is characteristic of chronic thyroiditis, which is the most common cause of hypothyroidism. However, detection of these antibodies would not indicate if hypothyroidism was present. T3 resin uptake (choice E) measurement is not an accurate test of thyroid function; it is primarily used to exclude various abnormalities in the thyroid-hormone binding proteins. -------------------------------------------------------------------------------20>The correct answer is A. Warfarin is the oral anticoagulant the patient was most likely taking. This drug is commonly prescribed to patients with atrial fibrillation to prevent the formation of atrial thrombi. Warfarin increases prothrombin time (PT) because it interferes with the synthesis of the vitamin K clotting factors of the liver (II, VII, IX, and X) and therefore necessitates regular monitoring of the PT. Cimetidine is an H2-blocker that inhibits hepatic enzymes, including those that metabolize warfarin. Consequently, coadministration of warfarin and cimetidine results in enhanced warfarin activity, producing pronounced anticoagulation and the bleeding diathesis in the patient in question. Cimetidine has one of the worst side effect profiles of all the H2-blockers and may also result in gynecomastia in men. Famotidine (choice B) is an H2-blocker that does not affect liver metabolism. Misoprostol (choice C) is a prostaglandin E1 analog used in peptic ulcer disease. It does not affect hepatic metabolism. Omeprazole (choice D) is a proton-pump inhibitor used to decrease acid production in patients with peptic ulcer disease or reflux. It does not affect drug metabolism by the liver. Ranitidine (choice

E) is another H2-blocker. It does not inhibit liver enzymes as strongly as cimetidine does. -------------------------------------------------------------------------------21>The correct answer is C. Bullae with the cleavage plane above the basal layer of the epidermis suggests pemphigus vulgaris, which is caused by autoantibodies to intercellular junctions of epidermal cells. The autoantibodies decrease the ability of the keratinocytes to adhere to one another, permitting formation of vesicles and bullae. Oral involvement is common, and often precedes the characteristic skin lesions. Separation of the epidermis

upon manual stroking of the skin is known as Nikolsky's sign. This sign is present in other disorders such as Stevens-Johnson syndrome, but we are told the woman is not taking any medications, a typical cause of Stevens-Johnson syndrome in the adult population. Antibodies to epidermal basement membrane proteins (choice A) are seen in bullous pemphigoid, which is a bullous disease characterized by blisters with a cleavage line between the epidermis and dermis. Antibodies to glycoprotein IIb/IIIa (choice B) are seen in autoimmune thrombocytopenic purpura. Antibodies to intrinsic factor (choice D) are seen in pernicious anemia. Antibodies to Type IV collagen (choice E) are seen in Goodpasture's syndrome. -------------------------------------------------------------------------------22>The correct answer is D. The lesion is a malignant melanoma. Melanomas can develop either de novo or in an existing mole. Sunlight exposure is a significant risk factor and fair-skinned persons are at increased risk of developing melanoma. The most significant factor for long term prognosis is the depth of the lesion, since the superficial dermis lies about 1 mm under the skin surface, and penetration to this depth is associated with a much higher incidence of metastasis than is seen with a more superficial location. The circumference of the lesion (choice A) is much less important than depth, since one form of melanoma (superficial spreading) can still have good prognosis despite large size, if it has not extended to the depth of the superficial dermal lymphatic bed. The darkness (choice B) or degree of variation in color (choice C) do not have prognostic significance once melanoma is diagnosed. Irregularity, or fuzziness at the border (choice E) of a mole-like lesion is a good clue to potential malignancy, but does not affect prognosis once a melanoma is diagnosed. -------------------------------------------------------------------------------23>The correct answer is E. By the third week of development, hematopoiesis begins in the blood islands of the yolk sac. Beginning at 1 month of age and continuing until 7 months of age, blood elements are also formed in the liver. Hematopoiesis occurs in the spleen and lymphatic organs between 2 and 4 months, and in the bone marrow after 4 months. -------------------------------------------------------------------------------24>The correct answer is E. The patient has I-cell disease, also known as mucolipidosis II, which is due to a defective UDP-N-acetylglucosamine-1-phosphotransferase, the enzyme that phosphorylates mannose on enzymes destined for lysosomes. Proteins coded by nuclear DNA are synthesized on cytoplasmic ribosomes, which may be either "free" or associated with the endoplasmic reticulum to form the rough endoplastic reticulum (RER). Proteins synthesized on the RER are transferred into the Golgi apparatus, where they undergo further modifications that determine whether they remain part of the Golgi apparatus, become part of the plasma membrane, or are shipped to lysosomes or mitochondria. Proteins not marked for transport to a specific intracellular site follow the default pathway and are exported into the extracellular compartment. The signal for transport of the acid hydrolases (and probably other enzymes) to the lysosomes is phosphorylation of a terminal mannose moiety on an N-linked oligosaccharide to form mannose 6phosphate. In I-cell disease, this terminal mannose moiety is not phosphorylated, and the acid hydrolases follow the default pathway and are secreted. Deficiency of alpha-Liduronidase results in lysosomal accumulation of dermatan sulfate and heparan sulfate (choice A) in several conditions such as mucopolysaccharidosis I, Hurler's disease, or Hurler's/Scheie disease. Hexosaminidase A deficiency (Tay-Sachs disease) is one

example of a condition in which ganglioside accumulation occurs (choice B). There are a number of diseases in which glycogen degradation (choice C) is defective. These are collectively termed glycogen storage diseases since they result in abnormal cellular accumulation of glycogen. In Pompe's disease, or type II glycogen storage disease, a lysosomal glucosidase is deficient, resulting in lysosomal glycogen accumulation.

Deficiency of sphingomyelinase (choice D), an enzyme involved in degradation of sphingomyelin, results in Niemann-Pick disease. Phosphorylation of tyrosine moieties (choice F) is unrelated to lysosomes or lysosomal enzymes; however, decreased ability to phosphorylate tyrosine moieties might be associated with diabetes or dwarfism. -------------------------------------------------------------------------------25>The correct answer is E. 95% of a normally distributed population will fall between plus or minus 1.96 standard deviations from the mean. Since the population is normally distributed with regard to IQ, this means that approximately 2.5% of the population will have IQ scores 2 standard deviations or more above the mean, and 2.5% of the population will have IQ scores 2 standard deviations or more below the mean. 2.5% of 200 people is 5 people. __________________ <26>5 mL of synovial fluid is aspirated from an inflamed knee joint. The fluid is yellow-white and cloudy and contains 200,000 WBC/mm3 (85% neutrophils). Needle-shaped, strongly negatively birefringent crystals are seen both within and outside neutrophils. These crystals most likely have which of the following compositions? A. Basic calcium phosphate B. Calcium oxalate C. Calcium pyrophosphate dihydrate D. Cholesterol E. Monosodium urate Answer -------------------------------------------------------------------------------<27>A patient has a painful ulcer on the tip of his tongue. Which of the following cranial nerves carries the pain sensation he experiences? A. V2 B. V3 C. VII D. IX E. X Answer -------------------------------------------------------------------------------<28>A 30-year-old veterinarian on a cattle ranch presents with a 1-to-2-month history of malaise, chills, drenching malodorous sweats, fatigue, and weakness. He has anorexia and has lost 15 pounds. He has intermittent fevers that range up to 103 F (39.4 C). He complains of visual blurring. A physical examination reveals mild lymphadenopathy, petechiae, and a cardiac murmur consistent with aortic insufficiency. What is the most likely etiologic agent? A. Bacillus anthracis B. Brucella abortus C. Coccidioides immitis D. Erysipelothrix rhusiopathiae E. Trichinella spiralis Answer --------------------------------------------------------------------------------

<29>The parents of a 6-month-old child who was normal at birth bring her into the clinic. Since their emigration to the U.S. from Eastern Europe soon after her birth, the child has developed diminished responsiveness , progressive blindness and deafness, and recently, seizures. Serum levels of which of the following compounds would be expected to be decreased in both of the parents? A. Dystrophin B. Hexosaminidase A C. Hypoxanthineguanine phosphoribosyltransferase (HGPRT) D. Phenylalanine hydroxylase E. Vitamin D3 Answer -------------------------------------------------------------------------------<30>A 60year-old male with angina comes to the emergency room with severe chest pain unresponsive to sublingual nitroglycerin. An EKG shows ST segment elevation in the anterolateral leads, and thrombolytic therapy is initiated. If streptokinase is given to this patient, it may produce thrombolysis after binding to which of the following proteins? A. Antithrombin III B. Fibrin C. Plasminogen D. Protein C E. Thrombomodulin Answer -------------------------------------------------------------------------------<31>The left adrenal vein drains directly into which of the following veins? A. Hemiazygos vein B. Inferior vena cava C. Left renal vein D. Splenic vein E. Superior mesenteric vein Answer -------------------------------------------------------------------------------<32>A premature infant develops progressive difficulty breathing over the first few days of life. Deficient surfactant synthesis by which of the following cell types may have contributed to the baby's respiratory problems? A. Alveolar capillary endothelial cells B. Bronchial mucous cells C. Bronchial respiratory epithelium D. Type I pneumocytes E. Type II pneumocytes Answer -------------------------------------------------------------------------------<33>A surgeon performs an exploratory laparotomy, producing a large incision in the patient's abdomen. Poor blood supply to which of the following is most likely to cause problems during the healing process? A. Adipose tissue B. Aponeuroses C. Loose connective tissue

D. Muscle E. Skin Answer -------------------------------------------------------------------------------<34>A 1-year-old child develops voluminous watery diarrhea and vomiting. She is brought to the pediatrician by her parents and evaluated, then sent home with instructions for the parents to give the child an electrolyte replacement solution. Which of the following viruses is the most likely cause of the child's diarrhea? A. Coronavirus B. Lymphocytic choriomeningitis virus C. Norwalk agent D. Orbivirus E. Rotavirus <35>A 2-year-old child presents to the pediatrician with hematuria. Examination reveals hypertension and an abdominal mass. A tumor is localized to the right kidney and biopsy reveals a stroma containing smooth and striated muscle, bone, cartilage, and fat, with areas of necrosis. The gene for this disorder has been localized to which of the following chromosomes? A. 5 B. 11 C. 13 D. 17 E. 22 Answer -------------------------------------------------------------------------------<36>A newborn baby has a prominent defect at the base of his spine through which his meninges and spinal cord protrude. A failure of which of the following processes is the most common cause of this type of defect? A. Development of primary vertebral ossification centers B. Development of the body C. Development of the pedicle D. Development of the superior articular process E. Fusion of the vertebral arches Answer -------------------------------------------------------------------------------<37>A 45-year-old male presents to the physician with muscle cramps, perioral numbness, and irritability over the past 3 to 4 months. Lab results reveal hypocalcemia, normal albumin level, and hyperphosphatemia . Parathyroid hormone level is decreased. Alkaline phosphatase level is normal. Which of the following is most likely causing this clinical scenario? A. Bone metastases B. Hashimoto's thyroiditis C. Hypervitaminosis D D. Hypomagnesemia E. Previous subtotal thyroidectomy Answer --------------------------------------------------------------------------------

<38>In which of the following organs are fenestrated endothelial cells common? A. Heart B. Liver C. Lungs D. Pancreas E. Stomach Answer -------------------------------------------------------------------------------<39>A macroscopic hepatic change known as nutmeg liver is indicative of A. acute left-sided heart failure B. acute right-sided heart failure C. alcohol toxicity D. chronic congestive heart failure E. liver cirrhosis Answer -------------------------------------------------------------------------------<40>A 26-year-old man is admitted through the emergency department to the hospital for a heroin overdose. His heart rate is 45 beats/min, and his blood pressure is 75/40 mm Hg. Which of the following best depicts the results from an arterial blood sample ? pH PaCO2 (mm Hg) HCO3- (mEq/L) A. 7.22 66 26 B. 7.34 29 15 C. 7.40 40 24 D. 7.47 20 14 E. 7.49 48 35 Answer -------------------------------------------------------------------------------<41> The maximum expiratory flow-volume curve shown above is created when the patient inhales as much air as possible and then expires with maximum effort until no more air can be expired. What is the forced vital capacity of this patient? A. 1.5 Liters B. 2.5 Liters C. 3.5 Liters D. 4.5 Liters E. 6.0 Liters Answer -------------------------------------------------------------------------------<42>A 25-year-old male gets into a brawl outside a bar. During the altercation, someone pulls out a gun and shoots him in the head. The bullet enters the man's temple and severs his right optic nerve completely. He is quickly transported to a nearby emergency room and an emergency physician tests his pupillary response by shining a light in the right eye. What will the physician most likely find? A. No pupillary constriction in the right eye, and no pupillary constriction in the left eye

B. No pupillary constriction in the right eye, but pupillary constriction in the left eye C. Pupillary constriction followed by pupillary dilatation in both eyes D. Pupillary constriction in the right eye, and no pupillary constriction in the left eye E. Pupillary constriction in both eyes F. Pupillary dilatation in both eyes Answer -------------------------------------------------------------------------------<43>A patient complaining of chest pain with exercise is evaluated by cardiac catheterization. The left anterior descending (LAD) branch of the coronary artery is visualized but the contrast angiography is poor . A Doppler-tipped catheter is inserted and the blood velocity is observed to increase transiently from 10 cm/sec to 70 cm/sec and then decrease bac k to 10 cm/sec as the probe passes a particular location in the artery . What was the cause of these changes in velocity measurements? A. A coronary artery aneurysm with a crosssectional area 1/7th the size of the native artery B. A coronary artery aneurysm with a cross-sectional area 7 times greater than the native artery C. A coronary artery obstruction with a cross-sectional area 1/7th of the size of the native artery D. A coronary artery obstruction with a cross-sectional area 7 times greater than the native artery Answer -------------------------------------------------------------------------------<44>Type of blood vessel Fall in blood pressure (mm Hg) (% of total peripheral resistance) Aorta and large arteries <1 Small arteries 10-20 Arterioles 50 Capillaries 25 Venules and small veins 9 Vena cave <1 The table above shows the fall in blood pressure that occurs for the various types of blood vessels as blood flows from the aorta (100 mm Hg) to the right atrium (0 mm Hg). Which of the following types of blood vessel is likely to have the highest ratio of wall cross-sectional area to lumen cross-sectional area? A. Aorta and large arteries B. Small arteries C. Arterioles D. Capillaries E. Venules and small veins F. Vena cavae Answer -------------------------------------------------------------------------------<45>Evaluation of an infant with a variety of congenital abnormalities reveals hypocalcemia due to a lack of parathyroid hormone. On x-ray, the thymic shadow is absent. A failure of development and differentiation of

which of the following embryonic structures would most likely be responsible for the observed presentation? A. Second pharyngeal arch B. Second pharyngeal cleft C. Second pharyngeal pouch D. Third pharyngeal arch E. Third pharyngeal pouch Answer -------------------------------------------------------------------------------<46>A person lifts one foot prior to taking a step. Which of the following nerves innervates the muscle group that allows the person to maintain balance by holding the weight of his body over the foot remaining on the ground? A. Femoral nerve B. First and second sacral nerves C. Obturator nerve D. Superior gluteal nerve E. Tibial nerve Answer -------------------------------------------------------------------------------<47>A 21-year-old college student from Connecticut with a past history of Lyme disease presents with chronic pain and swelling in his right knee. He states that he has had problems with the knee for the past two years. Which of the following HLA alleles would you expect to be present in this individual? A. HLA-B9 B. HLA-B17 C. HLA-B27 D. HLA-DR3 E. HLADR4 Answer -------------------------------------------------------------------------------<48>Which of the following metabolic processes occurs exclusively in the mitochondria? A. Cholesterol synthesis B. Fatty acid synthesis C. Gluconeogenesis D. Glycolysis E. Hexose monophosphate shunt F. Ketone body synthesis G. Urea cycle Answer -------------------------------------------------------------------------------49>An alert pediatric intern notices that a neonate with dysmorphic facies is twitching abnormally. As he watches, the baby experiences a seizure. Stat laboratories indicate a glucose of 90, serum sodium of 140, serum potassium of 4.2 and serum calcium of 3.9. Over the next several months, the child is admitted to the hospital twice for Candida infections, and once for a viral exanthem. Which of the following is the most likely diagnosis? A. Ataxia telangiectasia

B. Bruton's hypogammaglobulinemia C. DiGeorge syndrome D. Severe combined immunodeficiency E. Wiskott-Aldrich syndrome Answer -------------------------------------------------------------------------------50>A 2-month-old boy is evaluated for failure to thrive. As the pediatrician is examining the patient, she witnesses a seizure. Physical examination is remarkable for hepatomegaly, a finding later confirmed by CT scan, which also reveals renomegaly. Serum chemistries demonstrate severe hypoglycemia, hyperlipidemia, lactic acidosis, and ketosis. Which of the following diseases best accounts for this presentation? A. Gaucher's disease B. McArdle's disease C. Niemann-Pick disease D. Pompe's disease E. von Gierke's disease Answer -------------------------------------------------------------------------------Answer Now you can see what's hidden26>The correct answer is E. All the compounds listed can produce crystals in joint fluid, but only monosodium urate (associated with gout) and calcium pyrophosphate dihydrate (associated with CPPD crystal deposition disease, also called pseudogout), and to lesser degree basic calcium phosphate (apatite-associated arthropathy), have a high likelihood of being encountered on a step 1 USMLE exam. The crystals described are those of monosodium urate. Be careful not to answer "uric acid" if that is listed as an alternative choice on an exam, since the sodium salt is the predominant species in vivo. Basic calcium phosphate (choice A) is seen in apatite-associated arthropathy and produces spherical clumps of nonbirefringent submicroscopic crystals. Calcium oxalate crystals (choice B) are seen in primary oxalosis and are bipyramidal, positively birefringent crystals. Calcium pyrophosphate dihydrate crystals (choice C) are a feature of pseudogout and are rod-to-rhomboidal-shaped, weakly positively birefringent crystals. Cholesterol crystals (choice D) are seen in chronic and chylous effusions in inflammatory and degenerative arthritis, where they form large, flat, rhomboidal plates with notched corners. -------------------------------------------------------------------------------27>The correct answer is B. The innervation of the tongue is complex. The mandibular division of the trigeminal nerve (V3) carries general somatic sensation from the anterior two-thirds of the tongue. The maxillary division (V2, choice A) carries somatic sensation from the palate, upper gums, and upper lip. The facial nerve (VII, choice C) carries taste from the anterior twothirds of the tongue. The glossopharyngeal nerve (IX, choice D) carries sensation and taste from the posterior one-third of the tongue. The vagus nerve (X, choice E) carries sensation from the lower pharynx. -------------------------------------------------------------------------------28>The correct answer is B. Brucella abortus produces a chronic, granulomatous disease with caseating granulomas. Most cases occur in four states (Texas, California, Virginia, and Florida), and are associated with cattle, in which it produces spontaneous septic abortions. Most cases of brucellosis produce mild disease or fevers of unknown origin. However, Brucella spp. can infect the cardiovascular system and cause a localized infection. B. abortus is the most common species to cause endocarditis. The aortic valve is most commonly involved, followed by the mitral valve, and then both valves. Most cases of brucellosis are associated with occupational exposure, in persons such as

veterinarians, ranchers, and those who handle carcasses. Bacillus anthracis (choice A) is the causative agent for anthrax. It usually produces cutaneous disease (malignant pustule or eschar) at the site of inoculation in handlers of animal skins. It can also produce a severe hemorrhagic pneumonia (Woolsorter's disease) and septicemia. At-risk groups include those who handle animal carcasses or skins. Coccidioides immitis (choice C) is a dimorphic fungal disease producing a granulomatous pulmonary syndrome that is more severe in dark-skinned individuals. Disseminated disease occurs most often in Filipinos, Mexicans, and Africans. The infective form is the arthrospore; the diagnostic form in tissue is the spherule containing endospores. The disease is endemic in the San Joaquin River Valley. At-risk groups include military personnel, agricultural workers, construction workers, oil field workers, archaeology students, participants in outdoor sports, and sightseers. Remote infections from fomites (cotton harvested in the Southwestern U.S.) have been reported. Erysipelothrix rhusiopathiae (choice D) is a pleomorphic, gram-negative rod that causes a localized skin infection. It is an occupational disease of fishermen, fish handlers, butchers, meat-processing workers, poultry workers, farmers, veterinarians, abattoir workers, and housewives. Trichinella spiralis (choice E) is a nematode infection caused by the ingestion of larvae found in undercooked meat. Pork is the most common contaminated meat. However, outbreaks in the northern parts of the U.S. have been associated with eating undercooked infected bear meat. Symptoms include diarrhea, periorbital edema, myositis, fever, and eosinophilia. -------------------------------------------------------------------------------29>The correct answer is B. This patient has Tay-Sachs disease, an autosomal recessive disorder caused by the deficiency of hexosaminidase A, which leads to the accumulation of ganglioside GM2 in neurons, producing a degenerative neurologic disease. Children appear normal at birth, but then begin to suffer from diminished responsiveness, deafness, blindness, loss of neurologic function, and seizures. A cherry-red spot on the macula may be seen by ophthalmoscopic examination. Death usually occurs by 4 to 5 years of age. There is no therapy. The incidence is higher among Jews of Eastern European descent. Since the parents must be heterozygotes for the mutant hexosaminidase A allele, they would be expected to have diminished levels of the enzyme. A defect in the dystrophin (choice A) gene produces Duchenne muscular dystrophy, characterized by onset of weakness in early childhood. A severe deficiency in HGPRT (choice C) will lead to Lesch-Nyhan syndrome, characterized by excessive uric acid production, mental retardation, spasticity, selfmutilation, and aggressive, destructive behavior. Deficiency of phenylalanine hydroxylase (choice D) results in classic phenylketonuria, a disease in which phenylalanine, phenylpyruvate, phenylacetate, and phenyllactate accumulate in plasma and urine. Clinically, there is a musty body odor and mental retardation. Hypophosphatemic rickets is an X-linked dominant condition causing abnormal regulation of vitamin D3 (choice E) metabolism and defects in renal tubular phosphate transport. Symptoms include growth retardation, osteomalacia, and rickets. -------------------------------------------------------------------------------30>The correct answer is C. The fibrinolytic activity of streptokinase is due to its ability to bind and cleave plasminogen, producing plasmin. Plasmin directly cleaves fibrin, both between and within the fibrin polymers, thus breaking up thrombi and potentially restoring blood flow to ischemic cardiac muscle. This same mechanism of fibrinolysis is shared by urokinase and tissue-plasminogen activator (tPA). Antithrombin III (choice A) is a coagulation

inhibitor that binds to and inactivates thrombin. Antithrombin III is anticoagulant, not fibrinolyti __________________

Part 5/6-25 mcqs 1>Which of the following conditions may lead to development of an exudate within the pleural cavity? A. Bacterial pleuritis B. Cirrhosis of the liver C. Congestive heart failure D. Nephrotic syndrome E. Protein-losing enteropathy Answer -------------------------------------------------------------------------------2>In which of the following sites do myxopapillary ependymomas most frequently occur? A. Cerebellum B. Conus medullaris C. 4th ventricle D. Lateral ventricles E. Midbrain Answer -------------------------------------------------------------------------------3>A patient has longstanding severe hemolytic anemia characterized by hypochromic cells. Electrophoresis studies demonstrate a near complete absence of beta chains. Several years later, the patient develops cardiac failure. Intracardiac deposition of which of the following would be most likely to contribute to the cardiac failure? A. Calcium B. Iron C. Magnesium D. Potassium E. Sodium Answer -------------------------------------------------------------------------------4>Before being approved by the FDA, a chemical is tested for carcinogenicity by examining its mutagenic effects on bacterial cells in culture. Which of the following tests is used to make this determination? A. Ames test B. Nitroblue tetrazolium test C. Watson-Schwartz test D. Widal test E. Woellner enzyme test Answer -------------------------------------------------------------------------------5.A sharp instrument passing through the superior orbital fissure would most likely sever the A. abducens nerve B. facial nerve C. mandibular nerve D. maxillary nerve E. middle meningeal artery F. ophthalmic artery G. optic nerve

Answer -------------------------------------------------------------------------------6>An elderly nursing home patient has had multiple small strokes. On several occasions she has aspirated food, and neurological examination reveals that her gag reflex is absent. These findings suggest involvement of the nucleus of which of the following cranial nerves? A. Facial (VII) B. Glossopharyngeal (IX) C. Hypoglossal (XII) D. Spinal accessory (XI) E. Vestibulocochlear (VIII) Answer -------------------------------------------------------------------------------7>A new antifungal medication is being tested in Phase I clinical trials. Examination of the pharmacokinetic properties of the drug reveals that the half-life of the drug is 6 hours. If a continuous intravenous infusion of this drug were started on a research subject, how long would it take to reach 75% of steady state? A. 3 hours B. 6 hours C. 9 hours D. 12 hours E. 18 hours F. 24 hours Answer -------------------------------------------------------------------------------8>A leukemia patient who has undergone multiple courses of chemotherapy develops herpes simplex encephalitis. Which of the following would you expect a CT scan of the patient's brain to show? A. Generalized volume loss B. Volume loss selectively in the basal ganglia C. Volume loss selectively in the brainstem D. Volume loss selectively in the cerebellum E. Volume loss selectively in the temporal and frontal lobes Answer -------------------------------------------------------------------------------9>An 18-year-old male takes an employment physical examination that is remarkable for a systolic murmur preceded by a distinct click on auscultation. The patient has unique body features, with unusually long legs and long, tapering fingers. An ocular examination reveals dislocation of the lens. An abnormality of which of the following gene products is thought to underlie this condition? A. Collagen B. Dystrophin C. Elastin D. Fibrillin E. Myosin bchain Answer -------------------------------------------------------------------------------10>A third-world patient develops muscle weakness and atrophy with fasciculations and hyporeflexia. The patient's condition continues

to deteriorate, and his legs are eventually permanently paralyzed. The virus that can cause these problems is usually spread by which of the following routes? A. Dirt contact with open wound B. Injection C. Mosquitoes D. Oral-fecal E. Puncture wound of the foot Answer -------------------------------------------------------------------------------11>Physical examination of a 45-year-old diabetic patient demonstrates a pulsatile abdominal mass. Radiographic studies demonstrate a 10 cm diameter aneurysm of the abdominal aorta with foci of calcification in the walls. Which of the following is the most likely etiology for this aneurysm? A. Atherosclerosis B. Congenital weakness C. Cystic medial necrosis D. Syphilis E. Vasculitis Answer -------------------------------------------------------------------------------12>A 40-year-old woman is being seen by a physician for the 10th time this year for evaluation of vague aches and pains. An extensive prior evaluation has excluded the possibility of serious disease. During the interview with this patient, she makes repeated statements along the lines of, "What I want doesn't matter. Do what you want." And, "I'm afraid you won't have time to see me anymore." The traits this patient is exhibiting are most consistent with which of the following personality disorders? A. Dependent B. Histrionic C. Obsessive-compulsive D. Paranoid E. Schizoid Answer -------------------------------------------------------------------------------13>A 60-year-old man suddenly becomes completely blind in one eye, and angiography demonstrates occlusion of the central retinal artery. Which of the following is the most likely cause of the occlusion? A. Atheroma or embolism B. Cranial (temporal) arteritis C. Hypertension D. Polycythemia vera E. Tumor Answer -------------------------------------------------------------------------------14 A mailman gets a severe bite wound from a pit bull guarding a junkyard. The wound is cleansed and he receives a booster injection of tetanus toxoid and an injection of penicillin G. Several days later, the wound is inflamed and purulent. The exudate is cultured on blood agar and yields gram-negative rods. Antibiotic sensitivity tests are pending. The most likely agent to be isolated is

A. Bartonella henselae B. Brucella canis C. Clostridium tetani D. Pasteurella multocida E. Toxocara canis Answer -------------------------------------------------------------------------------15.A 7-year-old boy is referred to a specialty clinic because of digestive problems. He often experiences severe abdominal cramps after eating a high fat meal. He is worked up and diagnosed with a genetic defect resulting in a deficiency of lipoprotein lipase. Which of the following substances would most likely be elevated in this patient's plasma following a fatty meal? A. Albumin-bound free fatty acids B. Chylomicrons C. HDL D. LDL E. Unesterified fatty acids Answer -------------------------------------------------------------------------------16>A 49-year-old woman is in an automobile accident and sustains a closed head injury. A CT scan does not show any intracranial hemorrhage, but reveals a small tumor at the cerebellopontine angle of the brain. Which of the following nerves is most likely to be affected by this tumor? A. Facial nerve B. Glossopharyngeal nerve C. Optic nerve D. Trigeminal nerve E. Vagus nerve Answer -------------------------------------------------------------------------------17>A 47-year-old male patient presents with painful arthritis in the right big toe and uric acid renal stones. He has been taking allopurinol for his condition. What biochemical defect would likely be found in this patient? A. A defect in urea synthesis B. An abnormality of the purine degradation pathway C. An inability to synthesize non-essential amino acids D. Defective topoisomerases E. Increased levels of leukotrienes Answer -------------------------------------------------------------------------------18>A 3-year-old boy is brought to the emergency department after the acute onset of headache, vomiting, nuchal rigidity, and impaired mental status. MRI reveals a posterior fossa tumor that fills the 4th ventricle. Surgery is immediately started, and intraoperative consultation leads to a "frozen section" diagnosis of medulloblastoma. Which of the following pathologic mechanisms most likely accounts for this child's clinical presentation? A. Acute hemorrhage into the 4th ventricle B. Alteration of medullary function C. Increased intracranial pressure D. Infiltration of the cerebellar vermis by the neoplasm

E. Spread of tumor to the subarachnoid space Answer -------------------------------------------------------------------------------19>A diabetic mother gives birth to a baby who dies in the first week of life. Autopsy reveals a severe cardiac malformation. Which of the following is the most likely diagnosis? A. Atrial septal defect B. Coarctation of the aorta C. Eisenmenger's syndrome D. Tetralogy of Fallot E. Transposition of the great arteries Answer -------------------------------------------------------------------------------20>There is a classic zoo story about a cage with three monkeys in it. The largest monkey steals the middlesized monkey's banana. The middle-sized monkey then screams with rage, hits the smallest monkey on the head, and then steals his banana. The middle-sized monkey is using which of the following mechanisms of defense? A. Displacement B. Projection C. Reaction formation D. Regression E. Repression Answer -------------------------------------------------------------------------------21>A 10-year-old boy has a small tumor in the wall of the right lateral ventricle. A biopsy of this tumor is consistent with subependymal giant cell astrocytoma. Which of the following lesions may also be present in this patient? A. Caf-au-lait spots B. Cortical tubers C. Hemangioblastoma D. Lisch nodules E. Schwannoma of the 8th cranial nerve Answer -------------------------------------------------------------------------------22>Physical examination of a 45-year-old diabetic patient demonstrates a pulsatile abdominal mass. Radiographic studies demonstrate a 10-cm diameter aneurysm of the abdominal aorta with foci of calcification in the walls. Which of the following is the most likely etiology for the aneurysm? A. Atherosclerosis B. Congenital weakness C. Cystic medial necrosis D. Syphilis E. Vasculitis Answer -------------------------------------------------------------------------------23>A 19-year-old female, who recently moved from her family's home in another state, is hospitalized for attempting suicide by taking an overdose of antidepressant medications. On the third day of her hospital stay, she insists, under threat of a lawsuit, that her medications be stopped and

that she be discharged from the hospital so she "can go home and finish the job." Her sensorium is clear. Her physician should A. discharge her against medical advice (AMA) B. honor her request and release her immediately C. obtain an emergency order of detention D. release her to go back to her parents' home E. sedate her Answer -------------------------------------------------------------------------------24>Which of the following could prevent an allergen from reacting with a specific IgE molecule present on the mast cell membrane? A. Antihistamine B. Blocking antibody C. Cromolyn sodium D. Epinephrine E. Theophylline Answer -------------------------------------------------------------------------------25>During a fight, a 32-year-old man is hit on the back of the neck with a chair. A CT scan reveals a bony fragment that penetrated the lateral portion of the dorsal columns. Which of the following functions would most likely be affected by a lesion at this site? A. Fine motor control of the ipsilateral fingers B. Motor control of the contralateral foot C. Sweating of the ipsilateral face D. Proprioception from the ipsilateral leg E. Vibratory sense from the ipsilateral arm Answer -------------------------------------------------------------------------------Answers Now you can see what's hidden11>The correct answer is A. An exudate results from leakage of protein-rich fluid from the plasma into the interstitium. It is usually the result of increased vascular permeability caused by inflammation. Exudates also contain numerous acute or chronic inflammatory cells, depending on the inciting event. Of the above choices, only bacterial pleuritis would produce an exudate. If pleuritis is caused by pyogenic organisms, the exudate is purulent (neutrophil-rich). If pleural inflammation is due to mycobacterial infection or neoplastic infiltration, the resulting exudate will contain chronic inflammatory cells. In contrast, a transudate contains less protein and few inflammatory cells. There are two main mechanisms of transudate formation: 1) decreased oncotic pressure, such as that which occurs in cirrhosis of the liver, nephrotic syndrome, and protein-losing enteropathy (choices B, D, and E); and 2) increased hydrostatic pressure, which may result from congestive heart failure (choice C). -------------------------------------------------------------------------------2>The correct answer is B. Myxopapillary ependymoma is a variant of ependymoma, a tumor arising from ependymal cells. Histologically, myxopapillary ependymoma contains a myxoid (mucusrich) intercellular matrix, in which spindly neoplastic ependymal cells are arranged in a fascicular and papillary pattern (hence its designation). It is a benign tumor that almost always occurs in the distal segment of the spinal cord, ie, the conus medullaris. Once excised, the patient is cured. The cerebellum (choice A) is the favorite site for pilocytic astrocytomas, medulloblastomas, and hemangioblastomas, but not ependymomas. In general, classic ependymomas occur in close proximity to the ventricular cavities, specifically, the 4th ventricle (choice C) in children and the lateral ventricles (choice D) in adults. The myxopapillary variant does not occur in either location.

A midbrain location (choice E) would be truly exceptional for any type of ependymoma. -------------------------------------------------------------------------------3>The correct answer is B. The disease is beta thalassemia major, which is a severe hemolytic anemia characterized by a failure to produce the beta chains of hemoglobin (some HbF, the fetal form of hemoglobin, is produced). The excess alpha chains are insoluble, leading to intraand extravascular hemolysis. These patients require large numbers of transfusions, and iron overload with resulting secondary hemochromatosis can contribute to eventual cardiac failure. The heart is also damaged by the chronic high output state needed to compensate for the anemia. Calcium (choice A) deposition is seen in damaged tissues and states with high serum calcium, such as hyperparathyroidism. Magnesium (choice C), potassium (choice D), and sodium (choice E) are highly soluble and do not usually precipitate in tissues. -------------------------------------------------------------------------------4>The correct answer is A. The test described is the Ames test, which measures damage to DNA and correlates well with carcinogenicity in vitro. It is relatively inexpensive to perform, compared to other tests of carcinogenicity, and is frequently used as a screening test for potential carcinogens. The nitroblue tetrazolium test (choice B) is used to examine the ability of neutrophils to undergo a respiratory burst, and is used in the diagnosis of hereditary immunodeficiencies. The Watson-Schwartz test (choice C) detects porphobilinogen in urine, and is used in the The correct answer is C. Any tumor "filling the 4th ventricle" blocks the circulation of cerebrospinal fluid (CSF). This blockage leads to increased intracranial pressure, which manifests with nausea, vomiting, headache, nuchal rigidity, and mental status changes. If surgery is not performed promptly, cerebellar tonsillar herniation and rapid death will ensue. In children, medulloblastoma and ependymoma are the most frequent neoplasms presenting in this manner. There is no evidence in this case suggesting that acute hemorrhage into the 4th ventricular cavity (choice A) has occurred, nor is medulloblastoma typically associated with this complication. CNS tumors that frequently bleed are metastases from melanoma, renal cell carcinoma, and choriocarcinoma. Alterations in medullary function (choice B) lead to cardiorespiratory instability and may be caused by direct tumor compression or infiltration of the medulla, neither of which is supported by MRI findings in this case. Infiltration of the cerebellar vermis (choice D) is certainly seen in many cases of medulloblastoma, a tumor that arises from this midline cerebellar structure. However, this would lead to truncal ataxia and gait instability, not symptoms of increased intracranial pressure. Medulloblastoma characteristically spreads to the subarachnoid space (choice E), from which the neoplasm may metastasize to distant sites such as spinal cord. Plaques of medulloblastoma are often found on the cerebellar surface, creating a characteristic sugar coating, but this would not cause any significant blockage of CSF circulation. The correct answer is C. Any tumor "filling the 4th ventricle" blocks the circulation of cerebrospinal fluid (CSF). This blockage leads to increased intracranial pressure, which manifests with nausea, vomiting, headache, nuchal rigidity, and mental status changes. If surgery is not performed promptly, cerebellar tonsillar herniation and rapid death will ensue. In children, medulloblastoma and ependymoma are the most frequent neoplasms presenting in this manner. There is no evidence in this case suggesting that acute hemorrhage into the 4th ventricular cavity (choice A) has occurred, nor is medulloblastoma typically associated with this complication. CNS tumors that frequently bleed are metastases from melanoma,

renal cell carcinoma, and choriocarcinoma. Alterations in medullary function (choice B) lead to cardiorespiratory instability and may be caused by direct tumor compression or infiltration of the medulla, neither of which is supported by MRI findings in this case. Infiltration of the cerebellar vermis (choice D) is certainly seen in many cases of

medulloblastoma, a tumor that arises from this midline cerebellar structure. However, this would lead to truncal ataxia and gait instability, not symptoms of increased intracranial pressure. Medulloblastoma characteristically spreads to the subarachnoid space (choice E), from which the neoplasm may metastasize to distant sites such as spinal cord. Plaques of medulloblastoma are often found on the cerebellar surface, creating a characteristic sugar coating, but this would not cause any significant blockage of CSF circulation. diagnosis of porphyrias. The Widal test (choice D) is used to diagnose typhoid fever. The Woellner enzyme test (choice E) detects heterophil antibodies in patients with EpsteinBarr virus infection, such as infectious mononucleosis. -------------------------------------------------------------------------------5>The correct answer is A. A good way to remember what goes through the superior orbital fissure is that everything that innervates the eye, other than the optic nerve, passes through this fissure. This includes the oculomotor nerve (CN III), the trochlear nerve (CN IV), the ophthalmic nerve (V1), and the abducens nerve (CN VI). The facial nerve (CN VII; choice B) passes through the internal auditory meatus. The mandibular nerve (V3; choice C) passes through the foramen ovale. The maxillary nerve (V2; choice D) passes through the foramen rotundum. The middle meningeal artery (choice E) passes through the foramen spinosum. The ophthalmic artery (choice F) passes through the optic canal. The optic nerve (choice G) passes through the optic canal. -------------------------------------------------------------------------------6>The correct answer is B. Cranial nerve IX is the glossopharyngeal nerve, which has a nucleus in the medulla and is necessary for the gag reflex. The gag reflex is elicited by touching either side of the posterior pharynx with a tongue blade, producing bilateral elevation of the palate and bilateral contraction of the pharyngeal muscles. The afferent of this reflex arc consists of the ipsilateral glossopharyngeal nerve, while the vagus nerve, bilaterally, supplies the efferent limb. While the glossopharyngeal nerve may seem to be one of the less important cranial nerves, you should remember to test for its function, as a loss of gag reflex can lead to the patient's death secondary to an aspiration pneumonia. This form of pneumonia can be difficult to treat, as it commonly is due to a mixed flora, which may include a variety of anaerobes. Cranial nerve VII (choice A) is the facial nerve, which supplies motor function to the face, but does not supply the oropharynx. Cranial nerve XII (choice C) is the hypoglossal nerve, which supplies the tongue. It is not involved in the gag reflex. Cranial nerve XI (choice D) is the spinal accessory nerve, which supplies the trapezius and sternocleidomastoid. Cranial nerve VIII (choice E) is the vestibulocochlear nerve, responsible for hearing and equilibrium. -------------------------------------------------------------------------------7>The correct answer is D. The rule of thumb is that the plasma concentration will reach 50% in one half-life, 75% in two half-lives, 87.5% in three half-lives, etc., so that the difference between the current drug level and 100% halves with each half-life. In this instance, it takes two halflives to reach 75%. The half-live of this drug is 6 hours, so two half-lives is 12 hours. -------------------------------------------------------------------------------8>The correct answer is E. Herpes simplex can cause a necrotizing, hemorrhagic acute encephalitis that may rapidly produce death. The encephalitis characteristically involves the lower portions of the cerebral cortex, notably the temporal lobes and the base of the frontal lobes, possibly because the infection spreads from the oropharynx. -------------------------------------------------------------------------------9>The correct answer is

D. The genetic condition is Marfan syndrome, which is characterized by skeletal, ocular, and cardiovascular abnormalities. Typically, Marfan patients are tall with very long legs and tapering fingers. Laxity of joints is present, so that the thumb can be extended back to the wrist. Chest and spinal column deformities

may be present as well. The most frequent cardiovascular anomalies are incompetence of the aortic valve, aortic dissection, and mitral valve prolapse ("floppy valve"). The latter may give rise to mitral regurgitation with the typical auscultatory phenomenon of a systolic click followed by a murmur. The most characteristic ocular change is ectopia lentis, i.e., dislocation of the lens. Most deaths are due to rupture of aortic dissections. The gene mutated in Marfan syndrome encodes fibrillin, a 350-kD protein that serves as scaffolding for the deposition of elastin and formation of elastic fibers. Elastin (choice C) is a 70-kD protein that constitutes the central core of elastic fibers. Abundant elastin is found in the walls of large arteries, uterus, skin, and ligaments. Although elastic fibers are disrupted by mutations of the fibrillin gene, the structure of elastin protein is intact in Marfan syndrome. Collagen (choice A) is affected in a different set of genetic diseases, including EhlersDanlos syndromes and osteogenesis imperfecta. Dystrophin (choice B) is a large transmembrane protein whose function is essential in maintaining the structural integrity of striated muscle fibers. Mutations of the dystrophin gene, located on X chromosome, are responsible for muscular dystrophy. Mutations of the gene for myosin b-chain (choice E) account for more than one third of cases of hypertrophic cardiomyopathy, a condition characterized by hypertrophy of the left ventricle. Hypertrophic cardiomyopathy is not associated with mitral valve prolapse. -------------------------------------------------------------------------------10>The correct answer is D. The disease is poliomyelitis, which is caused by the poliovirus, a picorna virus. The virus is spread via the fecal-oral route and can then cause paralysis by infecting the alphamotor neurons of the anterior horn of the spinal cord. Early symptoms include malaise, headache, fever, nausea, abdominal pain, and sore throat. Bacterial and fungal infections can be spread by dirt contact with an open wound (choice A). AIDS is an example of a disease spread by injection (choice B) or exchange of body fluids. Malaria is an example of a disease spread by mosquitoes (choice C). Tetanus is the classic example of disease spread by a puncture wound of the foot (choice E). 11>The correct answer is A. This is a typical presentation of an abdominal aortic aneurysm, which is almost always due to severe atherosclerosis. The foci of calcification described occur within the atherosclerotic plaques, and indicate severe atherosclerotic disease. Congenital weakness of vessels (choice B) can produce berry aneurysms, especially in cerebral vessels in the circle of Willis. Cystic medial necrosis (choice C) can produce dissecting aneurysms, especially in Marfan's syndrome. Syphilitic aneurysms (choice D) typically involve the aortic root as it leaves the heart. Vasculitis (choice E) can produce aneurysms in small arteries. -------------------------------------------------------------------------------12>The correct answer is A. This scenario is classic for "dependent" personality. Look for reliance on others, subordination of own needs, and fear of abandonment. Note that in real life, patients may show symptoms of more than one personality disorder. Histrionic personality disorder (choice B) is characterized by theatricality, suggestibility, a strong desire for attention, and shallowness. Obsessive-compulsive personality disorder (choice C), also called anancastic personality disorder, is characterized by obsessions, perfectionism, rigidity, and self-doubt. Paranoid personality disorder (choice D) is characterized by suspiciousness, oversensitivity, querulousness, and an unforgiving character. Schizoid personality disorder (choice E) is characterized by emotional coldness, solitude, and social insensitivity.

-------------------------------------------------------------------------------13>The correct answer is A. The point of this question is that sometimes the obvious

explanation is the correct one. Occlusion of the central retinal artery rapidly causes irreversible blindness with loss of the inner retinal layers. (The photoreceptor rod and cone cells are maintained by the pigment epithelium.) The site of occlusion is typically just posterior to the cribriform plate. A garden-variety atheroma or embolism is overwhelmingly the most common cause of central retinal artery occlusion. Despite all of the teaching about the risk of blindness in temporal arteritis (choice B), this disorder causes only 10% of central retinal artery occlusions. Hypertension (choice C) is more apt to cause bleeding than thrombosis. Polycythemia vera (choice D) could (rarely) cause occlusion because of increased blood viscosity and a tendency for thrombosis. Tumor (choice E) might also cause retinal artery thrombosis, but this would be far rarer than atheroma. -------------------------------------------------------------------------------14>B -------------------------------------------------------------------------------15>C -------------------------------------------------------------------------------16>The correct answer is A. The facial nerve and the vestibulocochlear nerve emerge from the brain stem at the cerebellopontine angle. These are the two nerves which will be initially affected by a tumor in this region. The glossopharyngeal nerve and vagus nerve (choices B and E) emerge from the brain stem at the post-olivary sulcus. This is caudal to the cerebellopontine angle. The optic nerve (choice C) exits from the optic chiasm on the ventral surface of the diencephalon. This is rostral to the cerebellopontine angle. The trigeminal nerve (choice D) emerges from the brain stem at the anterolateral surface of the pons. This is rostral and ventral to the cerebellopontine angle. -------------------------------------------------------------------------------17>The correct answer is B. This patient has gout, characterized by painful joints due to the precipitation of uric acid crystals caused by excessive production of uric acid (a minority of cases are associated with underexcretion of uric acid). Kidney disease is also seen due to accumulation of uric acid in the tubules. The disease mostly affects males, and is frequently treated with allopurinol, an inhibitor of xanthine oxidase. Xanthine oxidase catalyzes the sequential oxidation of hypoxanthine to xanthine to uric acid. A defect in urea synthesis (choice A) would result in the accumulation of ammonia. Phenylketonuria is a disease in which tyrosine cannot be produced from phenylalanine (choice C). It is characterized by a musty body odor and mental retardation. Defective topoisomerases (choice D) would affect DNA unwinding, and therefore replication. Leukotrienes (choice E) are potent constrictors of smooth muscle and would more likely lead to bronchoconstriction. -------------------------------------------------------------------------------18>The correct answer is C. Any tumor "filling the 4th ventricle" blocks the circulation of cerebrospinal fluid (CSF). This blockage leads to increased intracranial pressure, which manifests with nausea, vomiting, headache, nuchal rigidity, and mental status changes. If surgery is not performed promptly, cerebellar tonsillar herniation and rapid death will ensue. In children, medulloblastoma and ependymoma are the most frequent neoplasms presenting in this manner. There is no evidence in this case suggesting that acute hemorrhage into the 4th ventricular cavity (choice A) has occurred, nor is medulloblastoma typically associated with this complication. CNS tumors that frequently bleed are metastases from melanoma, renal cell carcinoma, and choriocarcinoma. Alterations in medullary function (choice B) lead to cardiorespiratory instability and may be caused by direct tumor compression or infiltration of the medulla, neither of which is supported by MRI findings

in this case. Infiltration of the cerebellar vermis (choice D) is certainly seen in many cases of medulloblastoma, a tumor that arises from this midline cerebellar structure. However,

this would lead to truncal ataxia and gait instability, not symptoms of increased intracranial pressure. Medulloblastoma characteristically spreads to the subarachnoid space (choice E), from which the neoplasm may metastasize to distant sites such as spinal cord. Plaques of medulloblastoma are often found on the cerebellar surface, creating a characteristic sugar coating, but this would not cause any significant blockage of CSF circulation. -------------------------------------------------------------------------------19The correct answer is E. Maternal diabetes is best known for causing large but immature-forage babies. There is also a specific association between maternal diabetes and transposition of the great vessels. In transposition of the great vessels, the aorta takes off from the anterior part of the right ventricle and the pulmonary trunk takes off from the posterior part of the left ventricle. This produces a complete separation of the systemic and pulmonary circulations. Without surgical correction, most affected infants die within the first months of life, although a patent ductus arteriosus, patent foramen ovale, or ventricular septal defect may allow enough mixing of blood to temporarily sustain life. In atrial septal defect (choice A) blood can pass from one atrium to the other. Associate coarctation of the aorta (choice B) with Turner syndrome. Eisenmenger's syndrome (choice C) is a shift from a left-to-right shunt to a right-to-left shunt secondary to developing pulmonary hypertension. Tetralogy of Fallot (choice D) consists of a ventricular septal defect, an overriding aorta, pulmonic stenosis, and right ventricular hypertrophy. It is the most common cause of early cyanosis. -------------------------------------------------------------------------------20The correct answer is A. This is an example of displacement. In this defense mechanism, there is a transfer of emotion from a person, object, or situation with which it is appropriately associated to another that causes less distress. Displacement is common and often destructive to other individuals, such as when a man is fired from his job and subsequently beats his wife or children. In the medical setting, the hospital staff is a frequent target of displacement when family members react to their own feelings of guilt about someone's death. Projection (choice B) occurs when someone attributes his or her own thoughts to a different person. Reaction formation (choice C) is the unconscious adoption of behavior opposite to one's true feelings. Regression (choice D) is the adoption of behavior more appropriate to a younger age. Repression (choice E) is the deeply subconscious blocking of memories or emotions. -------------------------------------------------------------------------------21The correct answer is B. This item tests your knowledge of neurocutaneous syndromes, a group of hereditary conditions characterized by concomitant neoplastic or hamartomatous lesions predominantly affecting the skin and nervous system. Subependymal giant cell astrocytoma is a peculiar astrocytic tumor that grows from the walls of the lateral ventricles. It is pathognomonic of tuberous sclerosis, which is caused by mutations of TS1 or TS2 genes. Tuberous sclerosis manifests with multiple hamartomatous lesions in the skin, CNS, and visceral organs. Cortical tubers are malformed (hamartomatous) nodules of the cortex, probably resulting from faulty cortical development. Other lesions include shagreen patches and ash-leaf spots on the skin, cardiac myomas, and renal angiomyolipomas. Caf-au-lait spots (choice A) are found in both types of neurofibromatosis. Lisch nodules (choice D) are small pigmented nodular lesions of hamartomatous nature that are present in the iris of patients with neurofibromatosis type 1. Schwannomas of the 8th cranial nerve (choice E), especially when bilateral, are

typically associated with neurofibromatosis type 2. Hemangioblastoma (choice C) is a vascular tumor of unknown histologic origin that frequently develops in the cerebellum of patients with von Hippel-Lindau syndrome. Renal cell carcinomas are also common in this disease. Von Hippel-Lindau syndrome is caused by autosomal dominant mutations of the VHL gene, a tumor suppressor gene.

You may recall that mutations of VHL gene are also found in the majority of sporadic renal cell carcinomas --------------------------------------------------------------------------------22The correct answer is A. This is a typical presentation of an abdominal aortic aneurysm, which is almost always due to severe atherosclerosis. The foci of calcification described occur within the atherosclerotic plaques, and indicate severe atherosclerotic disease. Congenital weakness of vessels (choice B) can produce berry aneurysms. Cystic medial necrosis (choice C) can produce dissecting aneurysms (e.g., in Marfan's syndrome). Syphilitic aneurysms (choice D) typically involve the aortic root as it leaves the heart. Vasculitis (choice E) can produce aneurysms in small arteries. -------------------------------------------------------------------------------23The correct answer is C. The physician should obtain an emergency order of detention, regardless of her threats of a lawsuit. The woman clearly still has suicidal intent, demonstrated by her expressed verbalizations, and is therefore a danger to herself. Choices A, B, and D clearly place her in a position where she can carry out her plans to terminate her life. Sedating her (choice E) is the second best choice since it will prevent her from taking her life; however, sedation does not give therapists the opportunity to address the underlying motivations for her suicidal ideation. -------------------------------------------------------------------------------24The correct answer is B. Blocking antibody is generally an IgG antibody against the allergen. It is induced in the allergic patient by administering small amounts of allergen over a period of time. When the person is again exposed to the allergen, the IgG reacts with the allergen before it can reach the IgE-coated mast cell. Antihistamines (choice A) would block histamine receptors but would not react with the allergen before it could reach the IgE-coated mast cell. Cromolyn sodium (choice C) is a drug that stabilizes mast cell membranes, thus inhibiting degranulation, but would have no effect on allergen binding to IgE on mast cells. Epinephrine (choice D) is the mainstay of therapy in severe cases of immediate hypersensitivity (anaphylaxis). It increases intracellular cyclic AMP, thus decreasing mast cell degranulation, and causes smooth muscle dilation in the airways. Epinephrine would have no effect on the binding of the allergen to IgE on mast cells. Cyclic AMP is degraded in cells by the enzyme phosphodiesterase. Theophylline (choice E) inhibits this enzyme, thereby increasing mast cell cyclic AMP levels and making degranulation less likely without affecting the binding of the allergen to IgE on mast cells. -------------------------------------------------------------------------------25The correct answer is E. At this level, the lateral portion of the dorsal columns (funiculus) is comprised of the fasciculus cuneatus. Axons carrying tactile, proprioceptive, and vibratory information from the ipsilateral arm enter the spinal cord via the dorsal root, ascend the cord in the fasciculus cuneatus, and synapse in the nucleus cuneatus of the caudal medulla. Secondary neurons from this nucleus give rise to internal arcuate fibers, which decussate and ascend to the thalamus (ventral posterolateral nucleus, VPL) as the medial lemniscus. Tertiary neurons from the VPL project to the ipsilateral somatosensory cortex. Therefore, damage to the fasciculus cuneatus would result in a deficit in tactile, proprioceptive, and vibratory sense in the ipsilateral arm, because the fibers that carry this information do not cross until they reach the medulla. Fine motor control of the fingers (choice A) would be carried principally by the ipsilateral lateral corticospinal tract in the lateral funiculus of the cord. Motor control of the contralateral foot (choice B) is carried by the ipsilateral

corticospinal tract in the lateral funiculus of the cord. Hemianhidrosis (lack of sweating) over half of the face (choice C) could be produced by interruption of sympathetic innervation to the face. The hypothalamospinal tract projects from the hypothalamus to the intermediolateral cell column at levels T-1 to T-2. It descends in the lateral funiculus of the cord. Interruption of this tract results in Horner's syndrome (miosis, ptosis, hemianhidrosis).

Proprioception from the ipsilateral leg (choice D) is carried by the fasciculus gracilis in the medial part of the dorsal columns __________________ Part 6/6 - 25 mcqs 1>Which of the following conditions may lead to development of an exudate within the pleural cavity? A. Bacterial pleuritis B. Cirrhosis of the liver C. Congestive heart failure D. Nephrotic syndrome E. Protein-losing enteropathy Answer -------------------------------------------------------------------------------2>In which of the following sites do myxopapillary ependymomas most frequently occur? A. Cerebellum B. Conus medullaris C. 4th ventricle D. Lateral ventricles E. Midbrain Answer -------------------------------------------------------------------------------3>A patient has longstanding severe hemolytic anemia characterized by hypochromic cells. Electrophoresis studies demonstrate a near complete absence of beta chains. Several years later, the patient develops cardiac failure. Intracardiac deposition of which of the following would be most likely to contribute to the cardiac failure? A. Calcium B. Iron C. Magnesium D. Potassium E. Sodium Answer -------------------------------------------------------------------------------4>Before being approved by the FDA, a chemical is tested for carcinogenicity by examining its mutagenic effects on bacterial cells in culture. Which of the following tests is used to make this determination? A. Ames test B. Nitroblue tetrazolium test C. Watson-Schwartz test D. Widal test E. Woellner enzyme test Answer

-------------------------------------------------------------------------------5.A sharp instrument passing through the superior orbital fissure would most likely sever the A. abducens nerve B. facial nerve C. mandibular nerve D. maxillary nerve E. middle meningeal artery F. ophthalmic artery G. optic nerve Answer -------------------------------------------------------------------------------6>An elderly nursing home patient has had multiple small strokes. On several occasions she has aspirated food, and neurological examination reveals that her gag reflex is absent. These findings suggest involvement of the nucleus of which of the following cranial nerves? A. Facial (VII) B. Glossopharyngeal (IX) C. Hypoglossal (XII) D. Spinal accessory (XI) E. Vestibulocochlear (VIII) Answer -------------------------------------------------------------------------------7>A new antifungal medication is being tested in Phase I clinical trials. Examination of the pharmacokinetic properties of the drug reveals that the half-life of the drug is 6 hours. If a continuous intravenous infusion of this drug were started on a research subject, how long would it take to reach 75% of steady state? A. 3 hours B. 6 hours C. 9 hours D. 12 hours E. 18 hours F. 24 hours Answer -------------------------------------------------------------------------------8>A leukemia patient who has undergone multiple courses of chemotherapy develops herpes simplex encephalitis. Which of the following would you expect a CT scan of the patient's brain to show? A. Generalized volume loss B. Volume loss selectively in the basal ganglia C. Volume loss selectively in the brainstem D. Volume loss selectively in the cerebellum E. Volume loss selectively in the temporal and frontal lobes Answer

-------------------------------------------------------------------------------9>An 18-year-old male takes an employment physical examination that is remarkable for a systolic murmur preceded by a distinct click on auscultation. The patient has unique body features, with unusually long legs and long, tapering fingers. An ocular examination reveals dislocation of the lens. An abnormality of which of the following gene products is thought to underlie this condition? A. Collagen B. Dystrophin C. Elastin D. Fibrillin E. Myosin bchain Answer -------------------------------------------------------------------------------10>A third-world patient develops muscle weakness and atrophy with fasciculations and hyporeflexia. The patient's condition continues to deteriorate, and his legs are eventually permanently paralyzed. The virus that can cause these problems is usually spread by which of the following routes? A. Dirt contact with open wound B. Injection C. Mosquitoes D. Oral-fecal E. Puncture wound of the foot Answer -------------------------------------------------------------------------------11>Physical examination of a 45-year-old diabetic patient demonstrates a pulsatile abdominal mass. Radiographic studies demonstrate a 10 cm diameter aneurysm of the abdominal aorta with foci of calcification in the walls. Which of the following is the most likely etiology for this aneurysm? A. Atherosclerosis B. Congenital weakness C. Cystic medial necrosis D. Syphilis E. Vasculitis Answer -------------------------------------------------------------------------------12>A 40-year-old woman is being seen by a physician for the 10th time this year for evaluation of vague aches and pains. An extensive prior evaluation has excluded the possibility of serious disease. During the interview with this patient, she makes repeated statements along the lines of, "What I want doesn't matter. Do what you want." And, "I'm afraid you won't have time to see me anymore." The traits this patient is exhibiting are most consistent with which of the following personality disorders?

A. Dependent B. Histrionic C. Obsessive-compulsive D. Paranoid E. Schizoid Answer -------------------------------------------------------------------------------13>A 60-year-old man suddenly becomes completely blind in one eye, and angiography demonstrates occlusion of the central retinal artery. Which of the following is the most likely cause of the occlusion? A. Atheroma or embolism B. Cranial (temporal) arteritis C. Hypertension D. Polycythemia vera E. Tumor Answer -------------------------------------------------------------------------------14 A mailman gets a severe bite wound from a pit bull guarding a junkyard. The wound is cleansed and he receives a booster injection of tetanus toxoid and an injection of penicillin G. Several days later, the wound is inflamed and purulent. The exudate is cultured on blood agar and yields gram-negative rods. Antibiotic sensitivity tests are pending. The most likely agent to be isolated is A. Bartonella henselae B. Brucella canis C. Clostridium tetani D. Pasteurella multocida E. Toxocara canis Answer -------------------------------------------------------------------------------15.A 7-year-old boy is referred to a specialty clinic because of digestive problems. He often experiences severe abdominal cramps after eating a high fat meal. He is worked up and diagnosed with a genetic defect resulting in a deficiency of lipoprotein lipase. Which of the following substances would most likely be elevated in this patient's plasma following a fatty meal? A. Albumin-bound free fatty acids B. Chylomicrons C. HDL D. LDL E. Unesterified fatty acids Answer -------------------------------------------------------------------------------16>A 49-year-old woman is in an automobile accident and sustains a closed head injury. A CT scan does not show any intracranial hemorrhage, but reveals a small tumor at the cerebellopontine angle of the brain. Which of the following nerves is most likely to be affected by

this tumor? A. Facial nerve B. Glossopharyngeal nerve C. Optic nerve D. Trigeminal nerve E. Vagus nerve Answer -------------------------------------------------------------------------------17>A 47-year-old male patient presents with painful arthritis in the right big toe and uric acid renal stones. He has been taking allopurinol for his condition. What biochemical defect would likely be found in this patient? A. A defect in urea synthesis B. An abnormality of the purine degradation pathway C. An inability to synthesize non-essential amino acids D. Defective topoisomerases E. Increased levels of leukotrienes Answer -------------------------------------------------------------------------------18>A 3-year-old boy is brought to the emergency department after the acute onset of headache, vomiting, nuchal rigidity, and impaired mental status. MRI reveals a posterior fossa tumor that fills the 4th ventricle. Surgery is immediately started, and intraoperative consultation leads to a "frozen section" diagnosis of medulloblastoma. Which of the following pathologic mechanisms most likely accounts for this child's clinical presentation? A. Acute hemorrhage into the 4th ventricle B. Alteration of medullary function C. Increased intracranial pressure D. Infiltration of the cerebellar vermis by the neoplasm E. Spread of tumor to the subarachnoid space Answer -------------------------------------------------------------------------------19>A diabetic mother gives birth to a baby who dies in the first week of life. Autopsy reveals a severe cardiac malformation. Which of the following is the most likely diagnosis? A. Atrial septal defect B. Coarctation of the aorta C. Eisenmenger's syndrome D. Tetralogy of Fallot E. Transposition of the great arteries Answer -------------------------------------------------------------------------------20>There is a classic zoo story about a cage with three monkeys in it. The largest monkey steals the middlesized monkey's banana. The middle-sized monkey then screams with rage, hits the smallest monkey on the head, and

then steals his banana. The middle-sized monkey is using which of the following mechanisms of defense? A. Displacement B. Projection C. Reaction formation D. Regression E. Repression Answer -------------------------------------------------------------------------------21>A 10-year-old boy has a small tumor in the wall of the right lateral ventricle. A biopsy of this tumor is consistent with subependymal giant cell astrocytoma. Which of the following lesions may also be present in this patient? A. Caf-au-lait spots B. Cortical tubers C. Hemangioblastoma D. Lisch nodules E. Schwannoma of the 8th cranial nerve Answer -------------------------------------------------------------------------------22>Physical examination of a 45-year-old diabetic patient demonstrates a pulsatile abdominal mass. Radiographic studies demonstrate a 10-cm diameter aneurysm of the abdominal aorta with foci of calcification in the walls. Which of the following is the most likely etiology for the aneurysm? A. Atherosclerosis B. Congenital weakness C. Cystic medial necrosis D. Syphilis E. Vasculitis Answer -------------------------------------------------------------------------------23>A 19-year-old female, who recently moved from her family's home in another state, is hospitalized for attempting suicide by taking an overdose of antidepressant medications. On the third day of her hospital stay, she insists, under threat of a lawsuit, that her medications be stopped and that she be discharged from the hospital so she "can go home and finish the job." Her sensorium is clear. Her physician should A. discharge her against medical advice (AMA) B. honor her request and release her immediately C. obtain an emergency order of detention D. release her to go back to her parents' home E. sedate her Answer --------------------------------------------------------------------------------

24>Which of the following could prevent an allergen from reacting with a specific IgE molecule present on the mast cell membrane? A. Antihistamine B. Blocking antibody C. Cromolyn sodium D. Epinephrine E. Theophylline Answer -------------------------------------------------------------------------------25>During a fight, a 32-year-old man is hit on the back of the neck with a chair. A CT scan reveals a bony fragment that penetrated the lateral portion of the dorsal columns. Which of the following functions would most likely be affected by a lesion at this site? A. Fine motor control of the ipsilateral fingers B. Motor control of the contralateral foot C. Sweating of the ipsilateral face D. Proprioception from the ipsilateral leg E. Vibratory sense from the ipsilateral arm Answer -------------------------------------------------------------------------------Answers -------------------------------------------------------------------------------Now you can see what's hidden1>The correct answer is A. An exudate results from leakage of protein-rich fluid from the plasma into the interstitium. It is usually the result of increased vascular permeability caused by inflammation. Exudates also contain numerous acute or chronic inflammatory cells, depending on the inciting event. Of the above choices, only bacterial pleuritis would produce an exudate. If pleuritis is caused by pyogenic organisms, the exudate is purulent (neutrophil-rich). If pleural inflammation is due to mycobacterial infection or neoplastic infiltration, the resulting exudate will contain chronic inflammatory cells. In contrast, a transudate contains less protein and few inflammatory cells. There are two main mechanisms of transudate formation: 1) decreased oncotic pressure, such as that which occurs in cirrhosis of the liver, nephrotic syndrome, and protein-losing enteropathy (choices B, D, and E); and 2) increased hydrostatic pressure, which may result from congestive heart failure (choice C). -------------------------------------------------------------------------------2>The correct answer is B. Myxopapillary ependymoma is a variant of ependymoma, a tumor arising from ependymal cells. Histologically, myxopapillary ependymoma contains a myxoid (mucusrich) intercellular matrix, in which spindly neoplastic ependymal cells are arranged in a fascicular and papillary pattern (hence its designation). It is a benign tumor that almost always occurs in the distal segment of the spinal cord, ie, the conus medullaris. Once excised, the patient is cured. The cerebellum (choice A) is the favorite site for pilocytic astrocytomas, medulloblastomas, and hemangioblastomas, but not ependymomas. In general, classic ependymomas occur in close proximity to the ventricular cavities, specifically, the 4th ventricle (choice C) in children and the lateral

ventricles (choice D) in adults. The myxopapillary variant does not occur in either location. A midbrain location (choice E) would be truly exceptional for any type of ependymoma. -------------------------------------------------------------------------------3>The correct answer is B. The disease is beta thalassemia major, which is a severe hemolytic anemia characterized by a failure to produce the beta chains of hemoglobin (some HbF, the fetal form of hemoglobin, is produced). The excess alpha chains are insoluble, leading to intra- and extravascular hemolysis. These patients require large numbers of transfusions, and iron overload with resulting secondary hemochromatosis can contribute to eventual cardiac failure. The heart is also damaged by the chronic high output state needed to compensate for the anemia. Calcium (choice A) deposition is seen in damaged tissues and states with high serum calcium, such as hyperparathyroidism. Magnesium (choice C), potassium (choice D), and sodium (choice E) are highly soluble and do not usually precipitate in tissues. -------------------------------------------------------------------------------4>The correct answer is A. The test described is the Ames test, which measures damage to DNA and correlates well with carcinogenicity in vitro. It is relatively inexpensive to perform, compared to other tests of carcinogenicity, and is frequently used as a screening test for potential carcinogens. The nitroblue tetrazolium test (choice B) is used to examine the ability of neutrophils to undergo a respiratory burst, and is used in the diagnosis of hereditary immunodeficiencies. The Watson-Schwartz test (choice C) detects porphobilinogen in urine, and is used in the The correct answer is C. Any tumor "filling the 4th ventricle" blocks the circulation of cerebrospinal fluid (CSF). This blockage leads to increased intracranial pressure, which manifests with nausea, vomiting, headache, nuchal rigidity, and mental status changes. If surgery is not performed promptly, cerebellar tonsillar herniation and rapid death will ensue. In children, medulloblastoma and ependymoma are the most frequent neoplasms presenting in this manner. There is no evidence in this case suggesting that acute hemorrhage into the 4th ventricular cavity (choice A) has occurred, nor is medulloblastoma typically associated with this complication. CNS tumors that frequently bleed are metastases from melanoma, renal cell carcinoma, and choriocarcinoma. Alterations in medullary function (choice B) lead to cardiorespiratory instability and may be caused by direct tumor compression or infiltration of the medulla, neither of which is supported by MRI findings in this case. Infiltration of the cerebellar vermis (choice D) is certainly seen in many cases of medulloblastoma, a tumor that arises from this midline cerebellar structure. However, this would lead to truncal ataxia and gait instability, not symptoms of increased intracranial pressure. Medulloblastoma characteristically spreads to the subarachnoid space (choice E), from which the neoplasm may metastasize to distant sites such as spinal cord. Plaques of medulloblastoma are often found on the cerebellar surface, creating a characteristic sugar coating, but this would not cause any significant blockage of CSF circulation. The correct answer is C. Any tumor "filling the 4th ventricle" blocks the

circulation of cerebrospinal fluid (CSF). This blockage leads to increased intracranial pressure, which manifests with nausea, vomiting, headache, nuchal rigidity, and mental status changes. If surgery is not performed promptly, cerebellar tonsillar herniation and rapid death will ensue. In children, medulloblastoma and ependymoma are the most frequent neoplasms presenting in this manner. There is no evidence in this case suggesting that acute hemorrhage into the 4th ventricular cavity (choice A) has occurred, nor is medulloblastoma typically associated with this complication. CNS tumors that frequently bleed are metastases from melanoma, renal cell carcinoma, and choriocarcinoma. Alterations in medullary function (choice B) lead to cardiorespiratory instability and may be caused by direct tumor compression or infiltration of the medulla, neither of which is supported by MRI findings in this case. Infiltration of the cerebellar vermis (choice D) is certainly seen in many cases of medulloblastoma, a tumor that arises from this midline cerebellar structure. However, this would lead to truncal ataxia and gait instability, not symptoms of increased intracranial pressure. Medulloblastoma characteristically spreads to the subarachnoid space (choice E), from which the neoplasm may metastasize to distant sites such as spinal cord. Plaques of medulloblastoma are often found on the cerebellar surface, creating a characteristic sugar coating, but this would not cause any significant blockage of CSF circulation. diagnosis of porphyrias. The Widal test (choice D) is used to diagnose typhoid fever. The Woellner enzyme test (choice E) detects heterophil antibodies in patients with Epstein-Barr virus infection, such as infectious mononucleosis. -------------------------------------------------------------------------------5>The correct answer is A. A good way to remember what goes through the superior orbital fissure is that everything that innervates the eye, other than the optic nerve, passes through this fissure. This includes the oculomotor nerve (CN III), the trochlear nerve (CN IV), the ophthalmic nerve (V1), and the abducens nerve (CN VI). The facial nerve (CN VII; choice B) passes through the internal auditory meatus. The mandibular nerve (V3; choice C) passes through the foramen ovale. The maxillary nerve (V2; choice D) passes through the foramen rotundum. The middle meningeal artery (choice E) passes through the foramen spinosum. The ophthalmic artery (choice F) passes through the optic canal. The optic nerve (choice G) passes through the optic canal. -------------------------------------------------------------------------------6>The correct answer is B. Cranial nerve IX is the glossopharyngeal nerve, which has a nucleus in the medulla and is necessary for the gag reflex. The gag reflex is elicited by touching either side of the posterior pharynx with a tongue blade, producing bilateral elevation of the palate and bilateral contraction of the pharyngeal muscles. The afferent of this reflex arc consists of the ipsilateral glossopharyngeal nerve, while the vagus nerve, bilaterally, supplies the efferent limb. While the glossopharyngeal nerve may seem to be one of the less important cranial nerves, you should remember to test for its function, as a loss of gag reflex can lead to the patient's death secondary to an

aspiration pneumonia. This form of pneumonia can be difficult to treat, as it commonly is due to a mixed flora, which may include a variety of anaerobes. Cranial nerve VII (choice A) is the facial nerve, which supplies motor function to the face, but does not supply the oropharynx. Cranial nerve XII (choice C) is the hypoglossal nerve, which supplies the tongue. It is not involved in the gag reflex. Cranial nerve XI (choice D) is the spinal accessory nerve, which supplies the trapezius and sternocleidomastoid. Cranial nerve VIII (choice E) is the vestibulocochlear nerve, responsible for hearing and equilibrium. -------------------------------------------------------------------------------7>The correct answer is D. The rule of thumb is that the plasma concentration will reach 50% in one half-life, 75% in two half-lives, 87.5% in three halflives, etc., so that the difference between the current drug level and 100% halves with each half-life. In this instance, it takes two halflives to reach 75%. The half-live of this drug is 6 hours, so two half-lives is 12 hours. -------------------------------------------------------------------------------8>The correct answer is E. Herpes simplex can cause a necrotizing, hemorrhagic acute encephalitis that may rapidly produce death. The encephalitis characteristically involves the lower portions of the cerebral cortex, notably the temporal lobes and the base of the frontal lobes, possibly because the infection spreads from the oropharynx. -------------------------------------------------------------------------------9>The correct answer is D. The genetic condition is Marfan syndrome, which is characterized by skeletal, ocular, and cardiovascular abnormalities. Typically, Marfan patients are tall with very long legs and tapering fingers. Laxity of joints is present, so that the thumb can be extended back to the wrist. Chest and spinal column deformities may be present as well. The most frequent cardiovascular anomalies are incompetence of the aortic valve, aortic dissection, and mitral valve prolapse ("floppy valve"). The latter may give rise to mitral regurgitation with the typical auscultatory phenomenon of a systolic click followed by a murmur. The most characteristic ocular change is ectopia lentis, i.e., dislocation of the lens. Most deaths are due to rupture of aortic dissections. The gene mutated in Marfan syndrome encodes fibrillin, a 350-kD protein that serves as scaffolding for the deposition of elastin and formation of elastic fibers. Elastin (choice C) is a 70-kD protein that constitutes the central core of elastic fibers. Abundant elastin is found in the walls of large arteries, uterus, skin, and ligaments. Although elastic fibers are disrupted by mutations of the fibrillin gene, the structure of elastin protein is intact in Marfan syndrome. Collagen (choice A) is affected in a different set of genetic diseases, including Ehlers-Danlos syndromes and osteogenesis imperfecta. Dystrophin (choice B) is a large transmembrane protein whose function is essential in maintaining the structural integrity of striated muscle fibers. Mutations of the dystrophin gene, located on X chromosome, are responsible for muscular dystrophy. Mutations of the gene for myosin b-chain (choice E) account for more than one third of cases of hypertrophic cardiomyopathy, a condition characterized by hypertrophy of the left ventricle. Hypertrophic cardiomyopathy is not associated with mitral valve prolapse.

-------------------------------------------------------------------------------10>The correct answer is D. The disease is poliomyelitis, which is caused by the poliovirus, a picorna virus. The virus is spread via the fecal-oral route and can then cause paralysis by infecting the alphamotor neurons of the anterior horn of the spinal cord. Early symptoms include malaise, headache, fever, nausea, abdominal pain, and sore throat. Bacterial and fungal infections can be spread by dirt contact with an open wound (choice A). AIDS is an example of a disease spread by injection (choice B) or exchange of body fluids. Malaria is an example of a disease spread by mosquitoes (choice C). Tetanus is the classic example of disease spread by a puncture wound of the foot (choice E). 11>The correct answer is A. This is a typical presentation of an abdominal aortic aneurysm, which is almost always due to severe atherosclerosis. The foci of calcification described occur within the atherosclerotic plaques, and indicate severe atherosclerotic disease. Congenital weakness of vessels (choice B) can produce berry aneurysms, especially in cerebral vessels in the circle of Willis. Cystic medial necrosis (choice C) can produce dissecting aneurysms, especially in Marfan's syndrome. Syphilitic aneurysms (choice D) typically involve the aortic root as it leaves the heart. Vasculitis (choice E) can produce aneurysms in small arteries. -------------------------------------------------------------------------------12>The correct answer is A. This scenario is classic for "dependent" personality. Look for reliance on others, subordination of own needs, and fear of abandonment. Note that in real life, patients may show symptoms of more than one personality disorder. Histrionic personality disorder (choice B) is characterized by theatricality, suggestibility, a strong desire for attention, and shallowness. Obsessive-compulsive personality disorder (choice C), also called anancastic personality disorder, is characterized by obsessions, perfectionism, rigidity, and self-doubt. Paranoid personality disorder (choice D) is characterized by suspiciousness, oversensitivity, querulousness, and an unforgiving character. Schizoid personality disorder (choice E) is characterized by emotional coldness, solitude, and social insensitivity. -------------------------------------------------------------------------------13>The correct answer is A. The point of this question is that sometimes the obvious explanation is the correct one. Occlusion of the central retinal artery rapidly causes irreversible blindness with loss of the inner retinal layers. (The photoreceptor rod and cone cells are maintained by the pigment epithelium.) The site of occlusion is typically just posterior to the cribriform plate. A garden-variety atheroma or embolism is overwhelmingly the most common cause of central retinal artery occlusion. Despite all of the teaching about the risk of blindness in temporal arteritis (choice B), this disorder causes only 10% of central retinal artery occlusions. Hypertension (choice C) is more apt to cause bleeding than thrombosis.

Polycythemia vera (choice D) could (rarely) cause occlusion because of increased blood viscosity and a tendency for thrombosis. Tumor (choice E) might also cause retinal artery thrombosis, but this would be far rarer than atheroma. -------------------------------------------------------------------------------14>B -------------------------------------------------------------------------------15>C -------------------------------------------------------------------------------16>The correct answer is A. The facial nerve and the vestibulocochlear nerve emerge from the brain stem at the cerebellopontine angle. These are the two nerves which will be initially affected by a tumor in this region. The glossopharyngeal nerve and vagus nerve (choices B and E) emerge from the brain stem at the post-olivary sulcus. This is caudal to the cerebellopontine angle. The optic nerve (choice C) exits from the optic chiasm on the ventral surface of the diencephalon. This is rostral to the cerebellopontine angle. The trigeminal nerve (choice D) emerges from the brain stem at the anterolateral surface of the pons. This is rostral and ventral to the cerebellopontine angle. -------------------------------------------------------------------------------17>The correct answer is B. This patient has gout, characterized by painful joints due to the precipitation of uric acid crystals caused by excessive production of uric acid (a minority of cases are associated with underexcretion of uric acid). Kidney disease is also seen due to accumulation of uric acid in the tubules. The disease mostly affects males, and is frequently treated with allopurinol, an inhibitor of xanthine oxidase. Xanthine oxidase catalyzes the sequential oxidation of hypoxanthine to xanthine to uric acid. A defect in urea synthesis (choice A) would result in the accumulation of ammonia. Phenylketonuria is a disease in which tyrosine cannot be produced from phenylalanine (choice C). It is characterized by a musty body odor and mental retardation. Defective topoisomerases (choice D) would affect DNA unwinding, and therefore replication. Leukotrienes (choice E) are potent constrictors of smooth muscle and would more likely lead to bronchoconstriction. -------------------------------------------------------------------------------18>The correct answer is C. Any tumor "filling the 4th ventricle" blocks the circulation of cerebrospinal fluid (CSF). This blockage leads to increased intracranial pressure, which manifests with nausea, vomiting, headache, nuchal rigidity, and mental status changes. If surgery is not performed promptly, cerebellar tonsillar herniation and rapid death will ensue. In children, medulloblastoma and ependymoma are the most frequent neoplasms presenting in this manner. There is no evidence in this case suggesting that acute hemorrhage into the 4th ventricular cavity (choice A) has occurred, nor is medulloblastoma typically associated with this complication. CNS tumors that frequently bleed are metastases from melanoma, renal cell carcinoma, and choriocarcinoma.

Alterations in medullary function (choice B) lead to cardiorespiratory instability and may be caused by direct tumor compression or infiltration of the medulla, neither of which is supported by MRI findings in this case. Infiltration of the cerebellar vermis (choice D) is certainly seen in many cases of medulloblastoma, a tumor that arises from this midline cerebellar structure. However, this would lead to truncal ataxia and gait instability, not symptoms of increased intracranial pressure. Medulloblastoma characteristically spreads to the subarachnoid space (choice E), from which the neoplasm may metastasize to distant sites such as spinal cord. Plaques of medulloblastoma are often found on the cerebellar surface, creating a characteristic sugar coating, but this would not cause any significant blockage of CSF circulation. -------------------------------------------------------------------------------19The correct answer is E. Maternal diabetes is best known for causing large but immature-for-age babies. There is also a specific association between maternal diabetes and transposition of the great vessels. In transposition of the great vessels, the aorta takes off from the anterior part of the right ventricle and the pulmonary trunk takes off from the posterior part of the left ventricle. This produces a complete separation of the systemic and pulmonary circulations. Without surgical correction, most affected infants die within the first months of life, although a patent ductus arteriosus, patent foramen ovale, or ventricular septal defect may allow enough mixing of blood to temporarily sustain life. In atrial septal defect (choice A) blood can pass from one atrium to the other. Associate coarctation of the aorta (choice B) with Turner syndrome. Eisenmenger's syndrome (choice C) is a shift from a left-to-right shunt to a right-to-left shunt secondary to developing pulmonary hypertension. Tetralogy of Fallot (choice D) consists of a ventricular septal defect, an overriding aorta, pulmonic stenosis, and right ventricular hypertrophy. It is the most common cause of early cyanosis. -------------------------------------------------------------------------------20The correct answer is A. This is an example of displacement. In this defense mechanism, there is a transfer of emotion from a person, object, or situation with which it is appropriately associated to another that causes less distress. Displacement is common and often destructive to other individuals, such as when a man is fired from his job and subsequently beats his wife or children. In the medical setting, the hospital staff is a frequent target of displacement when family members react to their own feelings of guilt about someone's death. Projection (choice B) occurs when someone attributes his or her own thoughts to a different person. Reaction formation (choice C) is the unconscious adoption of behavior opposite to one's true feelings. Regression (choice D) is the adoption of behavior more appropriate to a younger age. Repression (choice E) is the deeply subconscious blocking of memories or emotions. -------------------------------------------------------------------------------21The correct answer is B. This item tests your knowledge of neurocutaneous syndromes, a group of hereditary conditions characterized by concomitant

neoplastic or hamartomatous lesions predominantly affecting the skin and nervous system. Subependymal giant cell astrocytoma is a peculiar astrocytic tumor that grows from the walls of the lateral ventricles. It is pathognomonic of tuberous sclerosis, which is caused by mutations of TS1 or TS2 genes. Tuberous sclerosis manifests with multiple hamartomatous lesions in the skin, CNS, and visceral organs. Cortical tubers are malformed (hamartomatous) nodules of the cortex, probably resulting from faulty cortical development. Other lesions include shagreen patches and ash-leaf spots on the skin, cardiac myomas, and renal angiomyolipomas. Caf-au-lait spots (choice A) are found in both types of neurofibromatosis. Lisch nodules (choice D) are small pigmented nodular lesions of hamartomatous nature that are present in the iris of patients with neurofibromatosis type 1. Schwannomas of the 8th cranial nerve (choice E), especially when bilateral, are typically associated with neurofibromatosis type 2. Hemangioblastoma (choice C) is a vascular tumor of unknown histologic origin that frequently develops in the cerebellum of patients with von Hippel-Lindau syndrome. Renal cell carcinomas are also common in this disease. Von HippelLindau syndrome is caused by autosomal dominant mutations of the VHL gene, a tumor suppressor gene. You may recall that mutations of VHL gene are also found in the majority of sporadic renal cell carcinomas --------------------------------------------------------------------------------22The correct answer is A. This is a typical presentation of an abdominal aortic aneurysm, which is almost always due to severe atherosclerosis. The foci of calcification described occur within the atherosclerotic plaques, and indicate severe atherosclerotic disease. Congenital weakness of vessels (choice B) can produce berry aneurysms. Cystic medial necrosis (choice C) can produce dissecting aneurysms (e.g., in Marfan's syndrome). Syphilitic aneurysms (choice D) typically involve the aortic root as it leaves the heart. Vasculitis (choice E) can produce aneurysms in small arteries. -------------------------------------------------------------------------------23The correct answer is C. The physician should obtain an emergency order of detention, regardless of her threats of a lawsuit. The woman clearly still has suicidal intent, demonstrated by her expressed verbalizations, and is therefore a danger to herself. Choices A, B, and D clearly place her in a position where she can carry out her plans to terminate her life. Sedating her (choice E) is the second best choice since it will prevent her from taking her life; however, sedation does not give therapists the opportunity to address the underlying motivations for her suicidal ideation. -------------------------------------------------------------------------------24The correct answer is B. Blocking antibody is generally an IgG antibody against the allergen. It is induced in the allergic patient by administering small amounts of allergen over a period of time. When the person is again exposed to the allergen, the IgG reacts with the allergen before it can reach the IgEcoated mast cell. Antihistamines (choice A) would block histamine receptors but would not react with the allergen before it could reach the IgE-coated mast cell.

Cromolyn sodium (choice C) is a drug that stabilizes mast cell membranes, thus inhibiting degranulation, but would have no effect on allergen binding to IgE on mast cells. Epinephrine (choice D) is the mainstay of therapy in severe cases of immediate hypersensitivity (anaphylaxis). It increases intracellular cyclic AMP, thus decreasing mast cell degranulation, and causes smooth muscle dilation in the airways. Epinephrine would have no effect on the binding of the allergen to IgE on mast cells. Cyclic AMP is degraded in cells by the enzyme phosphodiesterase. Theophylline (choice E) inhibits this enzyme, thereby increasing mast cell cyclic AMP levels and making degranulation less likely without affecting the binding of the allergen to IgE on mast cells. -------------------------------------------------------------------------------25The correct answer is E. At this level, the lateral portion of the dorsal columns (funiculus) is comprised of the fasciculus cuneatus. Axons carrying tactile, proprioceptive, and vibratory information from the ipsilateral arm enter the spinal cord via the dorsal root, ascend the cord in the fasciculus cuneatus, and synapse in the nucleus cuneatus of the caudal medulla. Secondary neurons from this nucleus give rise to internal arcuate fibers, which decussate and ascend to the thalamus (ventral posterolateral nucleus, VPL) as the medial lemniscus. Tertiary neurons from the VPL project to the ipsilateral somatosensory cortex. Therefore, damage to the fasciculus cuneatus would result in a deficit in tactile, proprioceptive, and vibratory sense in the ipsilateral arm, because the fibers that carry this information do not cross until they reach the medulla. Fine motor control of the fingers (choice A) would be carried principally by the ipsilateral lateral corticospinal tract in the lateral funiculus of the cord. Motor control of the contralateral foot (choice B) is carried by the ipsilateral corticospinal tract in the lateral funiculus of the cord. Hemianhidrosis (lack of sweating) over half of the face (choice C) could be produced by interruption of sympathetic innervation to the face. The hypothalamospinal tract projects from the hypothalamus to the intermediolateral cell column at levels T-1 to T-2. It descends in the lateral funiculus of the cord. Interruption of this tract results in Horner's syndrome (miosis, ptosis, hemianhidrosis). Proprioception from the ipsilateral leg (choice D) is carried by the fasciculus gracilis in the medial part of the dorsal columns.

Vous aimerez peut-être aussi